Вы находитесь на странице: 1из 181

NIMHANS - DM Neurology -

Exam Title :
MO...
Email : shubham.saha452@gmail.com
Contact : 9932160514

QUESTION 1. NDE3MCtTSFVCSEFNICBTQUhBK3NodWJoYW0uc2FoYTQ1MkBnbWFpbC5jb20rOTkzMjE2MD
USU9OIDA=
A 50 year old man presented to the emergency having woken up blind in his right eye after the
night’s sleep.Three months previously he had been evaluated for headache .An MRI done then
had revealed no abnormality.He was a known hypertensive since 10 years on regular
medications.On examination he had no perception of light in the right eye. Right pupil was
dilated and had no direct light reflex but consensual response to light shown in left eye was
normal .Fundus revealed generalised retinal pallor, swelling of the optic nerve head and
multiple small disc hemorrhages.Field charting was reported to show an altitudinal field
defect.The most likely cause of visual loss in this gentleman is

a) Anterior ischemic optic neuropathy


b) Posterior ischemic optic neuropathy
c) Acute evolved papilledema
d) Central retinal artery occlusion
Correct Answer: A
Your Answer: Unanswered
Explanation

The presentation of this man suggests a vascular etiology for blindness .The visual loss is
neverprominent in papilledema .While central retinal artery occlusion can produce can
produce visual loss of sudden onset ,fundus examination reveals cherry red spot in this
condition unlike what was seen in this patient .Occlusion of opthalmic artery and infarction of
optic nerve head and retina can result from local arteritis.The patient in question probably had
giant cell arteritis a condition commonly associated with anterior ischemic optic
neuropathy.Prognosis for recovery of vision is poor once visual loss sets in.

MORE NIMHANS FACTS- NIMHANS 2000

Medial medullary syndrome results due to occlusion of. ANTERIOR SPINAL ARTERY *****

Medial medullary syndrome is also known as inferior alternating syndrome , hypoglossal


alternating hemiplegia , lower alternating hemiplegia ,or Dejerine syndrome ,

Medial medullary syndrome is due to infarction of the pyramid and contralateral hemiparesis of
the arm and leg, sparing the face, also associated with contralateral loss of joint position sense
and ipsilateral tongue weakness occur.

On side opposite
On side of lesion
lesion

Paralysis of arm and


leg, sparing face;
Paralysis with
impaired tactile and
atrophy of one-half
proprioceptive sense
half the tongue:
over one-half the body:
Ipsilateral twelfth
Contralateral
nerve
pyramidal tract and
medial lemniscus

POSITIVE MED PG
Web: http://www.positivemedpg.com/ Score:
Email: manohrajan@yahoo.co.in 0.00 / 100
Page 1
NIMHANS - DM Neurology -
Exam Title :
MO...
Email : shubham.saha452@gmail.com
Contact : 9932160514

Pulmonary wedge pressure is indirectly a Left atrial pressure

The substance found primarily found in tendons is collagen

The specifie substrate for oxidative phosphorylation is ADP

METALLOPORPHYRINS

The important roles these tetrapyrrolic macrocycles play in vital biological processes, in
particular photosynthesis (chlorophyll), oxygen transport (hemoglobin), oxygen activation
(cytochrome), have led to their characterization as ‘pigments of life’.

QUESTION 2. NDE3MCtTSFVCSEFNICBTQUhBK3NodWJoYW0uc2FoYTQ1MkBnbWFpbC5jb20rOTkzMjE2MD
USU9OIDE=
A 70 year old man presented with limb weakness of 6 months duration .He reported
difficulty in getting up from squatting and climbing stairs and in combing.He also
reported dysphagia with occasional choking.There was no h/o ptosis or diplopia .He
had been recently tested for ‘lung cancer’ one year ago.He was a chronic asthmatic
since childhood and had been on steroid therapy(oral and inhailers) for several
years.On examination, there was no ptosis.He had proximal weakness of all four
limbs.There was no muscle tenderness.He had impaired touch sensation distally and
deep tendon reflexes were absent.Plantar was flexor. The most likely diagnosis

a) Lambert Eaton myasthenic syndrome


b) Polymyositis
c) Myasthenia gravis
d) Steroid induced myopathy
Correct Answer: B
Your Answer: Unanswered
Explanation

The presence of proximal weakness with dysphagia suggests polymyositis as the most likely
diagnosis in this patient.There is no ptosis or diplopia (AI-2006*) and there is no mention of
fatiguability which are characteris of myasthenia gravis.Limb girdle presentation is seen in
10% of myasthenics but again there is no fatiguability in this patient,rather there is a fixed
proximal weakness a feature seen in myasthenics of a very long duration. Limb girdle
presentation is more common in Lambert –Eaton myasthenic syndrome but again there
is distinct fatiguability in such patients.Steroid myopathy can be difficult to differentiate from
polymyositis.However the dysphagia seen in the patient suggests polymyositis rather than
steroid myopathy.Malignancies can predispose to the development of polymyositis as was seen
in this patient .The depressed deep tendon reflex are unusual for polymyositis and indicates
additional neuropathy that may be due to malignancy per se or due to drugs used to treat the
malignancy.

MORE NIMHANS FACTS-NIMHANS 2000

Tumour thought to arise from the chorionic villi

POSITIVE MED PG
Web: http://www.positivemedpg.com/ Score:
Email: manohrajan@yahoo.co.in 0.00 / 100
Page 2
NIMHANS - DM Neurology -
Exam Title :
MO...
Email : shubham.saha452@gmail.com
Contact : 9932160514

GTD is the common name for five closely related tumours (one benign tumour , and four malign
ant tumours ):

• The benign tumour


◦ Hydatidiform mole
• The four malignant tumours
◦ Invasive mol e
◦ Choriocarcinoma
◦ Placental site trophoblastic tumour
◦ Epithelioid trophoblastic tumour
◦ The moles were classified as benign or potentially malignant, accordin. to the
criteria of Hertig, while the chorionepitheliomas were divided into
choriocarcinoma, chorioadenoma and syncitioma****** , according to Ewing.
• Endodermal sinus tumor ( EST ), also known as yolk sac tumor ( YST ), is a member
of the germ cell tumor group of cancers . It is the most common testicular tumor in
children under 3, and is also known as infantile embryonal carcinoma .
• The process whereby WBC can squeeze through pores in capillary walls is DIAPE
DESIS
• The fungus which causes the vascular lesion with thrombus formation - MUCOR
[VORICONAZOLE-not useful*****}
• NON ENCAPSULATED BENIGN TUMORS -LEIOMYOMA,CAVERNOUS
HEMANGIOMA,RECTAL ADENOMA

QUESTION 3. NDE3MCtTSFVCSEFNICBTQUhBK3NodWJoYW0uc2FoYTQ1MkBnbWFpbC5jb20rOTkzMjE2MD
USU9OIDI=
A patient presented to the emergency department with symptoms of Intracranial infection and a
lumbar puncture was performed.All the following are causes of xanthochromia, in CSF except

a) Rise of CSF protein > 150 mg/dL


b) Hyperbilirubinemia
c) Subarachnoid hemorrhage
d) Traumatic lumbar puncture with RBCs of 1000/cu.mm
Correct Answer: D
Your Answer: Unanswered
Explanation

Xanthochromia has been expanded to include any bloody, brownish, or even cloudy CSF
sample.

Visual inspection may also falsely identify xanthochromia when the CSF is contaminated with
other pigments. This situation has been reported in patients taking rifampin for treatment of
tuberculosis, excessive carotinoid intake from fad dieting, high total CSF protein concentration,
or hepatic jaundice. Elevated CSF protein levels may cause yellowish discoloration of the CSF
because of the presence of albumin-bound bilirubin; this usually requires a protein
concentration in excess of 150 mg/dl******

. Discoloration may be seen at lower protein concentrations if the patient also has jaundice.
Such high protein concentrations are commonly seen in spinal block (Froin's syndrome),
radicular demyelination, carcinomatous meningitis, intracranial neoplasms, and cryptococcal or
tuberculous meningitis*****.

POSITIVE MED PG
Web: http://www.positivemedpg.com/ Score:
Email: manohrajan@yahoo.co.in 0.00 / 100
Page 3
NIMHANS - DM Neurology -
Exam Title :
MO...
Email : shubham.saha452@gmail.com
Contact : 9932160514

RBC > 1,00,000 implies large amount of blood that has contaminated the CSF while doing
LP.There will then be contamination with serum bilirubin and lipochromes resulting in
xanthochromia

Bloody Tap SAH

Opening pressure Normal Raises

Serial samples Clearance uniform

Allowed to stand Clots – fibrin webs CSF dilutes the blood & defibrinates

Supernatant with prompt


Clear Xantho chromia due RBC hemolysis
centrifugation

WBC :
1-2 per 1000 Proportion higher
RBC - ratio

Crenation Less common Common

1 mg per 100 Higher protein level due to irritating effect


RBC ratio CSF protein
RBCs of hemolyzed RBCs

MORE NIMHANS FACTS-2000

Sensitivity is a measure of - True positive

Sensitivity

* The term sensitivity was introduced by Yerushalmy.

* The ability of a test to identify correctly all those who have the disease, that is “true
positive”.

Specificity

* The ability of a test to identify correctly those who do not have the disease, that is, “true
negatives”.

* An ideal screening test should be 100% sensitive and 100 % specific

Sandfly acts as a vector to all of the following diseases

Arthropod-borne diseases FOR NIMHANS

POSITIVE MED PG
Web: http://www.positivemedpg.com/ Score:
Email: manohrajan@yahoo.co.in 0.00 / 100
Page 4
NIMHANS - DM Neurology -
Exam Title :
MO...
Email : shubham.saha452@gmail.com
Contact : 9932160514

Arthropod Diseases transmitted

1. Mosquito Malaria, filaria, viral


encephalitis (e.g., Japanese
encephalitis), viral fevers
(e.g., dengue, West Nile,
viral haemorrhagic fevers
(e.g., yellow fever, dengue
haemorrhagic fever)

2. Housely Typhoid and paratyphoid


fever, diarrhoea, dysentery,
cholera, gastro-enteritis,
amoebiasis, helminthic
infestations, poliomyelitis,
conjunctivitis, trachoma,
anthrax, yaws, etc

3. Sandfly***** Kala-azar, oriental sore,


* sandfly fever, oraya fever

4. Tsete fly Sleeping sickness

5. Louse Epidemic typhus, relapsing


fever, trench fever,
pediculosis

6. Rat flea Bubonic plague, endemic


typhus, chiggerosis,
hymenolepis diminuta

7. Blackfly Onchocerciasis

8. Reduviid bug Chagas disease

9. Hard tick Tick typhus, viral


encephalitis, viral fevers,
viral haemorrhagic fever,
(e.g., Kyasanur forest
disease), tularemia, tick
paralysis, human babesiosis

10. Soft tick Q fever, relapsing fever

POSITIVE MED PG
Web: http://www.positivemedpg.com/ Score:
Email: manohrajan@yahoo.co.in 0.00 / 100
Page 5
NIMHANS - DM Neurology -
Exam Title :
MO...
Email : shubham.saha452@gmail.com
Contact : 9932160514

11. Trombiculid Scrub typhus, Rickettsial-


mite pox

12. Itch-mite Scabies

13. Cyclops Guinea-worm disease, fish


tapeworm (D.latus)

14. Enteric pathogens


Cockroaches

QUESTION 4. NDE3MCtTSFVCSEFNICBTQUhBK3NodWJoYW0uc2FoYTQ1MkBnbWFpbC5jb20rOTkzMjE2MD
USU9OIDM=
A 75 year old Person is asked to say ‘clog’.When he says,it sounds ‘clong. This type of speech
disturbance is an example of

a) Dysphasia
b) Dysphonia
c) Dysarthia
d) Paraphasia
Correct Answer: B
Your Answer: Unanswered
Explanation

*Dysphasia refers to congenital or developmental languagedisorder and is distinct


from aphasia which is a disorder of language that is acquired secondary to brain
damage.

*Dysarthria refers to disorder of articulation of single sounds.

*Paraphasic speech refers to the presence of errors in the speech output .

*Dysphonia refers to disorder of sound production as would be expected in palatal palsy


when air escapes through the palate on attempting to talk.*****

MORE NIMHANS FACTS-NIMHANS 2000

III nerve palsy associated with contralateral hemiplegla is together known as WEBER
SYNDROME

The syndromes associated with 3 rd nerve are summarised below

Syndrome Feature

POSITIVE MED PG
Web: http://www.positivemedpg.com/ Score:
Email: manohrajan@yahoo.co.in 0.00 / 100
Page 6
NIMHANS - DM Neurology -
Exam Title :
MO...
Email : shubham.saha452@gmail.com
Contact : 9932160514

* Site – supr. cerebellar


peduncle
1. Nothnagel’s
syndrome * Effect – Ipsilat 3 rd N
Palsy, contra lateral
cerebellar Ataxia

* Site - Red nucleus

* Effect – Ipsilateral 3 rd
2. Benedict’s N palsy
(AI2007)
+

Controlateral chorea,
athetosis

* Site – Supr. cerebellar


peduncle + Red Nucleus
3. Claudes
* Effect – 1 + 2

* Site – Cerebral peduncle

4. Weber’s * Effect – Ipsilat – 3 rd N.


Palsy +

Contralateral hemiparesis

The etiology of an isolated, pupil-sparing oculomotor palsy is diabetes [AIIMS


NOV-2009***] , hypertension.

STROKE WITHIN POSTERIOR CIRCULATION

P1 Syndromes

* A third nerve palsy with contralateral ataxia (Claude's syndrome) or with contralateral
hemiplegia (Weber's syndrome) may result. If the subthalamic nucleus is involved,
contralateral hemiballismus occurs. Occlusion of the artery of Percheron presents with
paresis of upward gaze and drowsiness, and abulia.

Bilateral proximal PCA occlusion leads to extensive infarction in the midbrain and
subthalamus and presents as coma, unreactive pupils, bilateral pyramidal signs, and
decerebrate rigidity. Occlusion of the penetrating branches of thalamic and thalamogeniculate
arteries is called the thalamic Déjérine-Roussy syndrome and consists of contralateral
hemisensory loss followed by burning pain in the affected areas.

Ataxia telangiectasia is associated with Spino-cerebellar atrophy,LYMPHOMAS and


Athetosis

POSITIVE MED PG
Web: http://www.positivemedpg.com/ Score:
Email: manohrajan@yahoo.co.in 0.00 / 100
Page 7
NIMHANS - DM Neurology -
Exam Title :
MO...
Email : shubham.saha452@gmail.com
Contact : 9932160514

Ataxia Telangiectasia

* It is an autosomal recessive [AI - 2012***] disorder, presents with telangiectatic lesions


with deficits in cerebellar function and nystagmus. Associations are pulmonary infections, lym
phomas , leukemias, thymic hypoplasia with cellular and humoral (IgA and IgG2)
immunodeficiencies, premature aging, absent thymus gland and Type 1 diabetes mellitus. His
topathology reveals loss of Purkinje, granule, and basket cells in the cerebellar cortex and
deep cerebellar nuclei. Neuronal loss is also seen in the inferior olives of the medulla, anterior
horn neurons in the spinal cord and of dorsal root ganglion cells associated with posterior
column spinal cord demyelination

* The gene associated is the ATM gene ;it is associated with defective DNA repair.
Heterozygotes are prone for breast cancer.

QUESTION 5. NDE3MCtTSFVCSEFNICBTQUhBK3NodWJoYW0uc2FoYTQ1MkBnbWFpbC5jb20rOTkzMjE2MD
USU9OIDQ=
A 35 year old lady was admitted in the emergency department with supected seizures.There
was a history of famliy dispute.Measurement of which hormone may help to differentiate a true
and psychogenic seizures

a) Thyroid stimulating hormone (TSH)


b) Prolactin (PRL)
c) Growth Hormone
d) ACTH
Correct Answer: B
Your Answer: Unanswered
Explanation

The differentiation of true and psychogenic seizures is often difficult and the difficulty is
compounded by the fact that the two often co-exist. Psychogenic seizures are non-epileptic
attacks often part of conversion disorder precipitated by a stressful situation.They often
lastlonger than true seizures and may wax and wane over minutes to hours .The behavioural
manifestations of complex partial seizures especially of frontal origin are often bizarre and may
be suspected for psychogenic seizures. A normal EEG does not exclude a true
seizure.Serum levels of PRL are often raised followingtrue seizures in the immediate
30-minute post ictal period in most generalized seizures and many complex partial seizures
and can be of help in some patients.

MORE NIMHANS FACTS-NIMHANS 2000

Viral meningitis with reduced CSF sugar is seen in Coxsackie virus

Hypoglycorrhachia is generally defined as a cerebrospinal fluid glucose concentration of


less than 50 mg/dl or a cerebrospinal fluid/blood ratio of less than 0.50.

HYPOGLYCORRHACHIA is uncommon in central nervous system syndromes of viral etiology,


with the exception of those due to mumps, lymphocytic choriomeningitis, varicella-zoster,herpes

simplex, and eastern equine encephalitis.

POSITIVE MED PG
Web: http://www.positivemedpg.com/ Score:
Email: manohrajan@yahoo.co.in 0.00 / 100
Page 8
NIMHANS - DM Neurology -
Exam Title :
MO...
Email : shubham.saha452@gmail.com
Contact : 9932160514

Although hypoglycorrhachia has been found in association with meningoencephalitis due to the
enteroviruses in general,4,7 and specifically Coxsackie B2 2,3 and Coxsackie B5*****

Temporal lobe epilepsy is treated by VALPROATE and carbamazepine

In facial palsy, food accumulated in the mouth due to paralysis of- BUCCINATOR

QUESTION 6. NDE3MCtTSFVCSEFNICBTQUhBK3NodWJoYW0uc2FoYTQ1MkBnbWFpbC5jb20rOTkzMjE2MD
USU9OIDU=
A 2 year old girl child is brought to OPD with features of hand ringing stereotype movements,
impaired language and communication skills. Her HC is 42 cm. Her birth record shows HC of
35 cm. What is the most likely diagnosis?

a) Asperger syndrome
b) Rett syndrome
c) Fragile x syndrome
d) Colarad syndrome
Correct Answer: B
Your Answer: Unanswered
Explanation

• Asperger syndrome or Asperger's syndrome or Asperger disorder is an autism spectrum


disorder that is characterized by significant difficulties in social interaction

•It differs from other autism spectrum disorders by its relative preservation of linguistic and co
gnitive development

•Asperger syndrome (AS) is one of the autism spectrum disorders (ASD) or pervasive
developmental disorders (PDD), which are a spectrum of psychological conditions that are
characterized by abnormalities of social interaction and communication that pervade the
individual's functioning, and by restricted and repetitive interests and behavior

•Children with AS may have an unusually sophisticated vocabulary at a young age and have
been colloquially called "little professors", but have difficulty understanding figurative language
and tend to use language literally

•Individuals with AS often have excellent auditory and visual perception

•AS is also associated with high levels of alexithymia , which is difficulty in identifying and
describing one's emotions

• Rett syndrome is a neurodevelopmental disorder of the grey matter of the brain that affects
females more commonly than males. The clinical features include small hands and feet and a
deceleration of the rate of head growth (including microcephaly in some).

•People with Rett syndrome are prone to gastrointestinal disorders and up to 80% have
seizures.

•They typically have no verbal skills, and about 50% of individuals affected are not ambulatory .
Scoliosis , growth failure, and constipation are very common and can be problematic.

POSITIVE MED PG
Web: http://www.positivemedpg.com/ Score:
Email: manohrajan@yahoo.co.in 0.00 / 100
Page 9
NIMHANS - DM Neurology -
Exam Title :
MO...
Email : shubham.saha452@gmail.com
Contact : 9932160514

•Genetically Rett syndrome (symbolized RTT) is caused by mutations in the gene MECP2 locate
d on the X chromosome and can arise (1) sporadically or (2) from germline mutations

•Brain levels of norepinephrine are lower in people with Rett syndrome. The genetic loss of ME
CP2 changes the properties of cells in the locus coeruleus , the exclusive source of
noradrenergic innervation to the cerebral cortex and hippocampus

•Development is typically normal until 6–18 months, when language and motor milestones
regress, purposeful hand use is lost, and acquired deceleration in the rate of head growth (resu
lting in microcephaly in some ) is seen.

•Hand stereotypes are typical, and breathing irregularities such as hyperventilation,


breathholding, or sighing are seen in many. Early on, autistic-like behavior may be seen.

•The infant with Rett syndrome often avoids detection until 6–18 months

•The syndrome is associated with the expansion of a single trinucleotide gene sequence (CGG)
on the X-chromosome , and results in a failure to express the protein coded by the FMR1 gene,
which is required for normal neural development

•Fragile X is the most common known single gene cause of autism and the most common
inherited cause of intellectual disability

MORE NIMHANS FACTS-NIMHANS 2000

The most common lesion in Wernicke’s encephalopathy is found in the MAMILLARY BODIES

1% of alcoholics develop cerebellar degeneration or atrophy. 1 in 500 develop Wernicke's (oph


thalmoparesis, ataxia, and encephalopathy) and Korsakoff's (retrograde and anterograde
amnesia) syndromes

Wernicke's encephalopathy presents with confusion, ataxia, and diplopia resulting from
inflammation and necrosis of periventricular midline structures, including dorsomedial
thalamus, mammillary bodies, midline cerebellum, periaqueductal gray matter, and trochlear
and abducens nuclei.

Which one of the following is used to halt the process of degeneration of dopaminergic neurons
in striatonigral pathway- VITAMIN E

Prosopognosia is Inability to recognize face

POSITIVE MED PG
Web: http://www.positivemedpg.com/ Score:
Email: manohrajan@yahoo.co.in 0.00 / 100
Page 10
NIMHANS - DM Neurology -
Exam Title :
MO...
Email : shubham.saha452@gmail.com
Contact : 9932160514

* The patient Denies the


Anosognosia
existence of hemiparesis
(AIIMS
and even deny ownership
NOV-2011***)
of paralyzed limb.

* It is the inability to
integrate visual
information in the centre of
the gaze with more
peripheral information. It is
described as ‘Misses the
Simultagnosia Forest for the Trees’. It is a
component of Balint’s
syndrome.

* It is due to bilateral
involvement of network for
spatial attention.

* Bilateral or right sided


Dressing
dorsal parietal lobe lesions
Apraxia
is the cause

* Bilateral lesion in
fusiform and lingual gyri of
occipito temporal cortex is
Prosopagnosia
the cause and it presents
with face and object
recognition defects.

* Acute onset of severely


impaired fluency (often
mutism).

* Writing, reading, and


Aphemia
comprehension are intact.

* It is due to partial lesions


of Broca's area or
subcortical lesions.

QUESTION 7. NDE3MCtTSFVCSEFNICBTQUhBK3NodWJoYW0uc2FoYTQ1MkBnbWFpbC5jb20rOTkzMjE2MD
USU9OIDY=
Hyperkalemic periodic paralysis is caused by which of the following channelopathy

a) calcium channelopathy
b) Sodium channelopathy

POSITIVE MED PG
Web: http://www.positivemedpg.com/ Score:
Email: manohrajan@yahoo.co.in 0.00 / 100
Page 11
NIMHANS - DM Neurology -
Exam Title :
MO...
Email : shubham.saha452@gmail.com
Contact : 9932160514

c) Potassium channelopathy
d) Magnesium channelopathy
Correct Answer: B
Your Answer: Unanswered
Explanation

Channelopathies

Sodium channel disorders

Generalized epilepsy with febrile seizure plus -α and β1 subunit

Hyperkalemic periodic paralysis* **** – SCN1a & SCN4a

Paramyotonia congenita- α subunit of SCN4a

Potassium sensitive myotonias.

Potassium channel disorders :

Benign familial neonatal convulsions

Episodic ataxia type 1

Acquired neuromyotonia

Paroxysmal dykinesias

Calcium channel disorders:

Absence epilepsy

Familial hemiplegic migraine

Episodic ataxia type 2

Spinocereberal ataxia type 6 (progressive)

Hypokalemic periodic paralysis

Eaton lambert myesthenic syndrome.

MORE NIMHANS FACTS-NIMHANS 2000

True regarding Cerebral venous sinuses - Transverse sinus is unequal

The transverse sinus is one of the dural venous sinuses and drains the superior sagittal sinus
, the occipital sinus and the straight sinus , and empties into the sigmoid sinus which in turn
reaches the jugular bulb. The transverse sinuses are frequently of unequal size******, wit
h the one formed by the superior sagittal sinus being the larger; they increase in size as they
proceed, from back to center.

Highly variable anatomy. For example one study demonstrated

• 39% hypoplasia of the left sinus


• 31% symmetric

POSITIVE MED PG
Web: http://www.positivemedpg.com/ Score:
Email: manohrajan@yahoo.co.in 0.00 / 100
Page 12
NIMHANS - DM Neurology -
Exam Title :
MO...
Email : shubham.saha452@gmail.com
Contact : 9932160514
• 20% aplasia of the left sinus
• 6% hypoplasia of the right sinus
• 4% aplasia of the right sinus

The following drug is avoided in patient with Non ketotic hyperglycemia with seizure - PHENY
TOIN

. Hang up angle reflex is seen in -HYPOTHYROIDISM

All of the following are true regarding Poliomyelitis

a)Maximum recovery after paralysis can occur within months

b. Flaccid paralysis develops during febrile phase of illness

c. Asymmetrical paralysis of proximal leg can occur

QUESTION 8. NDE3MCtTSFVCSEFNICBTQUhBK3NodWJoYW0uc2FoYTQ1MkBnbWFpbC5jb20rOTkzMjE2MD
USU9OIDc=
Regarding SSRIs which of the following is the correct statement

a) SSRIare useful in mania


b) SSRI blocks reuptake of norepinephrine
c) SSRI causes effect within 2 weeks
d) SSRIare useful in management of Eatingdisorders
Correct Answer: D
Your Answer: Unanswered
Explanation

SSRIs are antidepressants and they are contraindicated in mania*****

.SSRI specifically blocks the reuptake of serotonin (specific serotonin reuptake inhibitors).

SSRIs include

(1) Fluoxetine(2) Fluvoxamine

(3) Paroxetine(4) Citalopram

(5) Escitalopram(6) Sertraline

SSRI have equal efficacy compared to tricyclics and MAOIbut they are superior in causing less
side effects than TCAand MAOI. Cardiac side effects, Anticholinergic and postural hypotension
is less with SSRI.

SSRI has GI side effects and sexual dysfunction.

SSRI are first time antidepressants used not only for depression also for anxiety disorder, Panic
disorder, Phobia, PTSD, Eating disorders ***** and OCD.

All antidepressant shows clinical effect by4 – 6 wks.*****

MORE NIMHANS FACTS-NIMHANS 2000

POSITIVE MED PG
Web: http://www.positivemedpg.com/ Score:
Email: manohrajan@yahoo.co.in 0.00 / 100
Page 13
NIMHANS - DM Neurology -
Exam Title :
MO...
Email : shubham.saha452@gmail.com
Contact : 9932160514

HYPOKALEMIA LEADS to paralytic ileus, leads to metabolic alkalosis

Tumour presenting 3 cm. from anal region is treated by CHEMOIRRADIATION

In healing by first intention all of the following occurs

A continuous layer of epidermis forms within 48 hrs

b. Reticulin can be demonstrated in the dermal gap by the 2 nd to 3 rd day

C.By the 4 th week the cellular and vascular elements decrease in number and concentration

Within 24 hours • Neutrophils appear at margins of incision, moving toward fibrin clot •
Epidermis at its cut edges thickens as a result of mitotic activity of basal cells

• Within 24 to 48 hours , spurs of epithelial cells from the both edges migrate and grow
along the cut margins of the dermis, depositing BM components as they move. They fuse in the
midline beneath the surface scab, thus producing a continuous but thin epithelial layer.

1. By day 3, • Neutrophils replaced by macrophages • Granulation tissue progressively


invades incision space • Collagen fibers are now present in the margins of the incision,
but at first these are vertically oriented. • Epithelial cell proliferation continues,
thickening epidermal covering layer
2. By day 5 , • Incisional space is filled with granulation tissue • Neovascularisation is
maximal • Collagen fibrils become more abundant and begin to bridge incision • The
epidermis recovers its normal thickness, and differentiation of surface cells yields a
mature epidermal architecture with surface keratinizatio
3. During the second week • Continued accumulation of collagen and proliferation of
fibroblasts • Leukocytic infiltrate, edema, and increased vascularity have largely
disappeared.
4. By the end of the first month, • Scar comprises a cellular connective tissue devoid of
inflammatory infiltrate, covered now by intact epidermis. • Dermal appendages that have
been destroyed in the line of the incision are permanently lost. • Tensile strength of the
wound increases thereafter, but it may take months for the wounded area to obtain its
maximal strengt

QUESTION 9. NDE3MCtTSFVCSEFNICBTQUhBK3NodWJoYW0uc2FoYTQ1MkBnbWFpbC5jb20rOTkzMjE2MD
USU9OIDg=
A chronic addict was admitted with symptoms suggestive of Opiate Withdrawal.All of the
following drugs are used in the Treatment of this patient except

a) Methadone
b) Buprenorphine
c) Lofexidine
d) Naloxone
Correct Answer: D
Your Answer: Unanswered
Explanation

Opiate Withdrawal Treatment

POSITIVE MED PG
Web: http://www.positivemedpg.com/ Score:
Email: manohrajan@yahoo.co.in 0.00 / 100
Page 14
NIMHANS - DM Neurology -
Exam Title :
MO...
Email : shubham.saha452@gmail.com
Contact : 9932160514

* The principle is to substitute a longer-acting, orally active, pharmacologically equivalent drug


for the abused drug.

* Methadone, buprenorphine (the partial mu agonist) are used. Clonidine, a centrally acting
sympatholytic agent, can be used.. Lofexidine, a clonidine analogue with less hypotensive
effect, is better.

* Methadone for Detoxification is from 2 to 3 weeks to as long as 180 days.

* Buprenorphine is a partial agonist, and produces fewer withdrawal symptoms. Buprenorphine


is superior to the alpha-2-adrenergic agonist clonidine.

* Clonidine relieves lacrimation, rhinorrhea, muscle pain, joint pain, restlessness, and
gastrointestinal symptoms.

* Clonidine or lofexidine are typically administered orally. The side effects are sedation,
lethargy, and dry mouth.

Rapid and Ultrarapid Opiate Detoxification

* The opioid antagonist naltrexone typically is combined with an alpha-2-adrenergic to shorten


the duration of withdrawal. Buprenorphine in combination with naltrexone and clonidine
reduced ROD from 3 to 1 day of detoxification. Ultrarapid opiate detoxification is an
extension of ROD using anesthetics., but it carries risk.

MORE NIMHANS FACTS- NIMHANS 2001

Sine qua non for cerebral cortex- PYRAMIDAL CELLS

IPSP is due to C1- influx

As an example of inhibitory postsynaptic action, consider a neuronal synapse that uses GABA as
its transmitter . At such synapses, the GABA receptors typically open channels that are selectiv
ely permeable to Cl - .***** When these channels open, negatively charged chloride ions can
flow across the membrane

Initiation of impulse starts in- AXON HILLOCK

Melatonin is Serotonergic

.Sprinter’s Energy supply for first 4-5 seconds in 100metrs running race is provided
form CARBOHYDRATE RESERVE

QUESTION 10. NDE3MCtTSFVCSEFNICBTQUhBK3NodWJoYW0uc2FoYTQ1MkBnbWFpbC5jb20rOTkzMjE2M


USU9OIDk=
The antidepressant with reuptake inhibition property of both dopamine and norepinephrine
uptake is

a) Amineptine
b) Nomifensine
c) Amoxapine
d) Mianserine

POSITIVE MED PG
Web: http://www.positivemedpg.com/ Score:
Email: manohrajan@yahoo.co.in 0.00 / 100
Page 15
NIMHANS - DM Neurology -
Exam Title :
MO...
Email : shubham.saha452@gmail.com
Contact : 9932160514
Correct Answer: B
Your Answer: Unanswered
Explanation

Nomifensine is a norepinephrine-dopamine reuptake inhibitor developed by a team at Hoechst


AG in the 1960s. i.e. a drug that increases the amount of synaptic norepinephrine and dopamine
available to receptors by blocking the dopamine and norepinephrine reuptake transporters.

Due to the risk of a risk of haemolytic anaemia , the U.S. Food and Drug Administration (FDA)
withdrew approval for nomifensine on March 20, 1992.

MORE NIMHANS FACTS-NIMHANS 2001

*Mucosa in Crohn’s disease shows the appearance of- Cobble stone

*Myophosphorylase deficiency, also known as McArdle's disease, is the most common of the
glycolytic defects to cause exercise intolerance. Exertion can be followed by myoglobinuria and
acute renal failure. Forearm exercise test - An impaired rise in venous lactate is seen in
glycolytic defect. In lactate dehydrogenase deficiency, venous levels of lactate do not increase,
but pyruvate rises to normal.

*Estimation of age examination of teeth is by Gustafson’s method

* Optic atrophy is characteristic feature of poisoning by Methyl alcohol.

The ingestion of methanol (wood alcohol) causes metabolic acidosis, and its metabolites forma
ldehyde and formic acid cause severe optic nerve and central nervous system damage.

* Treatment is similar to that for ethylene glycol intoxication with pyridoxine supplements, fom
epizole or ethanol, and hemodialysis. Fomepizole is an alcohol dehydrogenase inhibitor (4-
methyl pyrazole).

QUESTION 11. NDE3MCtTSFVCSEFNICBTQUhBK3NodWJoYW0uc2FoYTQ1MkBnbWFpbC5jb20rOTkzMjE2M


USU9OIDEw
A 55 year old man presents with a 10 day history of confusion. His friend mentions that he
drinks 15 units of alcohol a day. Which of the following strongly suggests a diagnosis of
Korsakoff’s psychosis?

a) Absence seizures
b) Delusional beliefs
c) Poor long term memory
d) Auditory hallucinations
e) Confabulation
Correct Answer: E
Your Answer: Unanswered
Explanation

There are six major symptoms of Korsakoff's syndrome:

• ◦ antegrade amnesia- inability to form new memories


◦ retrograde amnesia, severe loss of memories formed before the onset of the
condition

POSITIVE MED PG
Web: http://www.positivemedpg.com/ Score:
Email: manohrajan@yahoo.co.in 0.00 / 100
Page 16
NIMHANS - DM Neurology -
Exam Title :
MO...
Email : shubham.saha452@gmail.com
Contact : 9932160514
◦ confabulation that is, invented memories which are then taken as true due to gaps
in memory sometimes associated with blackouts
◦ minimal content in conversation
◦ lack of insight
◦ apathy - the patients lose interest in things quickly and generally appear indifferent
to change.

Thiamine (vitamin B 1 ) deficiency

* Wernicke's encephalopathy presents with confusion, ataxia, and diplopia resulting from
inflammation and necrosis of periventricular midline structures, including dorsomedial
thalamus, mammillary bodies, midline cerebellum, periaqueductal gray matter, and trochlear
and abducens nuclei.

* Prolonged untreated thiamine deficiency can result in an irreversible dementia/amnestic


syndrome (Korsakoff's syndrome). Korsakoff's syndrome - Memory for new events is impaired,
but have normal immediate memory, attention span, and level of consciousness. Mammillary
body atrophy (irreversible)

*1 in 500 develop Wernicke's (ophthalmoparesis, ataxia, and encephalopathy) and Korsakoff's


(retrograde and anterograde amnesia) syndromes.

MORE NIMHANS FACTS-NIMHANS 2001

Parentectomy : A slang term meaning removal of a parent (or both parents) from the child. It
is of relevance to the hospitalization of children.It can be used in

a. Reactive attachment disorder

b. Child abuse

c. Anorexia nervosa

LEIGH disease is due to the accumulation of GLYCOGEN

Mutations of SDHA do not predispose to paraganglioma tumors but instead cause Leigh'
s disease , a form of encephalopathy .

It is most commonly inherited in an autosomal recessive manner OR MITOCHONDRIAL


INHERITANCE.In a few cases of Leigh syndrome due to mutations in nuclear DNA, inheritance
is X-linked recessive

symptoms may include weakness and lack of muscle tone ; spasticity; movement disorders; cere
bellar ataxia ; and peripheral neuropathy.

Mitochondrial DNA Depletion Syndromes

* Mitochondrial DNA depletion syndrome (MDS) includes Leigh's syndrome), PEO, an isolated
myopathy, myo - neuro - gastrointestinal - encephalopathy (MNGIE).

QUESTION 12. NDE3MCtTSFVCSEFNICBTQUhBK3NodWJoYW0uc2FoYTQ1MkBnbWFpbC5jb20rOTkzMjE2M


USU9OIDEx

POSITIVE MED PG
Web: http://www.positivemedpg.com/ Score:
Email: manohrajan@yahoo.co.in 0.00 / 100
Page 17
NIMHANS - DM Neurology -
Exam Title :
MO...
Email : shubham.saha452@gmail.com
Contact : 9932160514

A 55 year old lady is admitted to a hospital because of cardiac disease.She is on lithium. Even if
her lithium dose remained the same, all of the following would be expected to increase her
plasma lithium level except

a) associated renal disease


b) thiazides
c) low sodium diet
d) caffeine
Correct Answer: D
Your Answer: Unanswered
Explanation

Lithium

Mood stabilising agent

Lithium concentrations are known to be increased with concurrent use of diuretics —


especially loop diuretics (such as furosemide) and thiazides — and non-steroidal anti-
]
inflammatory drugs (NSAIDs) such as ibuprofen and aspirin . Lithium concentrations can also
be increased with concurrent use of ACE inhibitors such as captopril , enalapril , and lisinopril .

There are also drugs that can increase the clearance of lithium from the body, which can result
in decreased lithium levels in the blood. These drugs include theophylline , caffeine *********
**, and acetazolamide . Additionally, increasing dietary s odium intake may also reduce
lithium levels by prompting the kidneys to excrete more lithium

renal failure will increase the level of lithium

Indications

Trt. of acute mania


Prophylaxis of bipolar mood disorder

Others

Schizo affective disorder


Prophylaxis of Unipolar Cyclothymia

Acute depression

Impulsive aggression

Chronic alchoholism

Kleine - levin syndrome

Pharmacokinetics

Rapidly absorbed

Not protein bound

POSITIVE MED PG
Web: http://www.positivemedpg.com/ Score:
Email: manohrajan@yahoo.co.in 0.00 / 100
Page 18
NIMHANS - DM Neurology -
Exam Title :
MO...
Email : shubham.saha452@gmail.com
Contact : 9932160514

No metabolism

Entirely by kidneys

Mood of action -Decreasescatecholamine activity Before Starting Therapy

Kidneys-urine routine,

Thyroid-- T4TSH

Heart-ECG

CNS

Dose-900-2100 mg/ d

Side effects

Neurological

Commonest-Tremor-50%

Seizures

Cog wheelrigidity

Delirium

Renal 10-50%

NephrogenicDI

Nephrotic syndrome

CVS

‘T’wavedepression

Endocrine

Hypothyroidism

Weight gain

Pregnancy

Teratogenic

Ebstein’s

Blood Levels

Therapeutic level (acute mania) -0.8 – 1.2 meg/ l

POSITIVE MED PG
Web: http://www.positivemedpg.com/ Score:
Email: manohrajan@yahoo.co.in 0.00 / 100
Page 19
NIMHANS - DM Neurology -
Exam Title :
MO...
Email : shubham.saha452@gmail.com
Contact : 9932160514

Prophylactic level (BPD) -0.6 – 1.2 meq/L

Toxic level >2.0meq/L

MORE NIMHANS FACTS-NIMHANS 2001/2002

The surest sign of intrauterine death includes GAS IN AORTA

Oculomotor nerve passes between Superior cerebellar artery and Posterior cerebellar artery

The vagus nerve supplies the large intestine closest up to SPLENIC FLEXURE

PERITONEAL ORGANS

The intraperitoneal organs are the stomach , spleen , liver , bulb of the duodenum , jejun
um , ileum, transverse colon , and sigmoid colon .

The retroperitoneal organs are the remainder of the duodenum , the cecum and ascending
colon , the descending colon , the pancreas , and the kidneys .

QUESTION 13. NDE3MCtTSFVCSEFNICBTQUhBK3NodWJoYW0uc2FoYTQ1MkBnbWFpbC5jb20rOTkzMjE2M


USU9OIDEy
Operant condition where paradigm pain stimulus are given to a child for decreasing a certain
undesired behaviour can be classified as:

a) Positive reinforcement
b) Negative reinforcement
c) Punishment
d) Negotiation
Correct Answer: C
Your Answer: Unanswered
Explanation

Behaviour Therapy

POSITIVE MED PG
Web: http://www.positivemedpg.com/ Score:
Email: manohrajan@yahoo.co.in 0.00 / 100
Page 20
NIMHANS - DM Neurology -
Exam Title :
MO...
Email : shubham.saha452@gmail.com
Contact : 9932160514

I. Operant Conditioning procedure for Increasing a behaviour Positive reinforcement –


Reward

Negative reinforcement – Punishment is avoided

Modelling

II. Operant conditioning procedures for

Decreasing a behavior

a)Time out

-Reinforcement is withdrawn for

some time

b) Punishment *

c)Satiation

-Undesired response is positively

reinforced

Hysteria – conversion reaction

*Symptoms of motor or sensory function

– suggesting a medical disorder

*Sudden on set

*Symptoms in the presence of psychosocial

stress

*Do not intentionally produce symptoms

* ‘Secondary gain’

-Autonomic system*

-typically not involved

MORE NIMHANS FACTS-NIMHANS 2002

Pulmonary fibrosis is caused by

NItrofurantoin

Methotrexate

Cyclophosphamide

The treatment of choice in the management of acute manic illness is Lithium

POSITIVE MED PG
Web: http://www.positivemedpg.com/ Score:
Email: manohrajan@yahoo.co.in 0.00 / 100
Page 21
NIMHANS - DM Neurology -
Exam Title :
MO...
Email : shubham.saha452@gmail.com
Contact : 9932160514

In a patient post myocardial infarction developed atrial fibrillation. Subsequently the ECHO
showed a mural thrombus. The management is by warfarin

QUESTION 14. NDE3MCtTSFVCSEFNICBTQUhBK3NodWJoYW0uc2FoYTQ1MkBnbWFpbC5jb20rOTkzMjE2M


USU9OIDEz
Six diseases of animals are caused by prions.In contrast to other prion diseases, Which of the
following is highly communicable

a) Scrapie
b) Mink encephalopathy
c) feline spongiform encephalopathy
d) chronic wasting disease (CWD)
Correct Answer: D
Your Answer: Unanswered
Explanation

MORE NIMHANS FACTS from HARRISON 20th


*Prions also appear to play a role in more common
illnesses such as Alzheimer’s disease (AD) and Parkinson’s disease (PD).

*PrP prions reproduce by binding to the normal, cellular isoform of the prion protein (PrPC) and
stimulating conversion
of PrPC into the disease-causing isoform PrPSc . PrPC is rich in α-helix and has little β-
structure, whereas PrPSc
has less α-helix and a high amount of β-structure . This α-to-β structural transition in PrP is
the
fundamental event underlying this group of prion diseases

*PrP gene located on human chromosome 20.

*Familial CJD (fCJD), Gerstmann-Sträussler-Scheinker (GSS) disease, and fatal familial


insomnia (FFI) are all dominantly inherited prion diseases that are caused by
mutations in the PrP gene

*Iatrogenic CJD (iCJD) seems to be the result of the accidental inoculation of patients
with
prions.
Variant CJD (vCJD) in teenagers and young adults in Europe is the result of exposure
to tainted beef
from cattle with bovine spongiform encephalopathy (BSE).

*Six diseases of animals are caused by prions . Scrapie of sheep and goats is the
prototypic PrP
prion disease. Mink encephalopathy, BSE, feline spongiform encephalopathy, and
exotic ungulate
encephalopathy are all thought to occur after the consumption of prion-infected
foodstuffs.Chronic wasting disease (CWD), a prion disease endemic in deer.

POSITIVE MED PG
Web: http://www.positivemedpg.com/ Score:
Email: manohrajan@yahoo.co.in 0.00 / 100
Page 22
NIMHANS - DM Neurology -
Exam Title :
MO...
Email : shubham.saha452@gmail.com
Contact : 9932160514

In contrast to other prion diseases, CWD is highly communicable.

* Both the rods and the PrP amyloid filaments found in brain tissue exhibit similar
ultrastructural
morphology and green-gold birefringence after staining with Congo red dye.
*In FFI, the protease-resistant fragment of PrPSc after deglycosylation has a molecular
mass of 19 kDa, whereas in fCJD and most sporadic prion diseases, it is 21 kDa

*Highly stable amyloids gave rise to stable prions with long incubation times; low-
stability amyloids led to
prions with short incubation times

*The amino acid sequence of PrP encodes the


species of the prion, and the prion derives its PrPSc sequence from the last mammal in
which it was
passaged. This “species barrier” t

o transmission is
correlated with the degree of similarity between the amino acid sequences of PrPC in
the inoculated host and
of PrPSc in the inoculum

*A mutation at codon 178 that results in substitution of asparagine for aspartic acid
produces FF I if a methionine is encoded at the polymorphic residue 129 on the same
allele.

* Typical CJD is seen if the D 1 78N mutation occurs with a valine encoded at position
129 of the same allele

*Accidental transmission of CJD to humans appears to have occurred with corneal


transplantation,
contaminated electroencephalogram (EEG) electrode implantation, and surgical
procedures

*Dura Mater Grafts,human growth hormone (hGH) preparations derived from


human pituitaries can transmit

VARIANT CJD-BSE prions had been transmitted to humans through the consumption
of tainted beef.

NEUROPATHOLOGY-PRIONS

*On light microscopy, the pathologic hallmarks of CJD are spongiform degeneration and
astrocytic gliosis.Generally, the spongiform changes occur in the cerebral cortex,putamen,
caudate nucleus, thalamus, and molecular layer of the cerebellum

*The amyloid plaques of GSS disease are morphologically distinct from those seen in kuru or
scrapie. GSS
plaques consist of a central dense core of amyloid surrounded by smaller globules of
amyloid.The plaques can be distributed throughout the brain but are most frequently found in
the cerebellum .

POSITIVE MED PG
Web: http://www.positivemedpg.com/ Score:
Email: manohrajan@yahoo.co.in 0.00 / 100
Page 23
NIMHANS - DM Neurology -
Exam Title :
MO...
Email : shubham.saha452@gmail.com
Contact : 9932160514

*In vCJD, a characteristic feature is the presence of “florid plaques. ” These are composed of a
central core of
PrP amyloid, surrounded by vacuoles in a pattern suggesting petals on a flower

*Most patients (~90%) with CJD exhibit myoclonus

*Dementia with myoclonus can also be due to AD , dementia with Lewy bodies , corticobasal
degeneration , cryptococcal
encephalitis , or the myoclonic epilepsy disorder Unverricht-Lundborg disease

CLINICAL PRESENTATION

*Familial CJD has an earlier mean age of onset than sCJD.

*In GSS disease , ataxia is usually a prominent and presenting feature, with dementia
occurring late in the disease course.

*GSS disease presents earlier than CJD (mean age 43 years) and is typically more slowly
progressive than CJD; death usually occurs within 5 years of onset.

*FFI is characterized by insomnia and dysautonomia; dementia occurs only in the terminal
phase of the illness.
Variant CJD has an unusual clinical course , with a prominent psychiatric prodrome that may
include visual hallucinations and early ataxia, whereas frank dementia is usually a late sign of
vCJD.

LABORATORY TESTS

The only specific diagnostic tests for CJD and other human PrP prion diseases measure PrPSc-
conformation-dependent immunoassay (CDI)

*sCJD On FLAIR sequences and diffusion-weighted imaging, ~90% of patients show increased
intensity
in the basal ganglia and cortical ribboning -can also be seen in viral encephalitis, paraneoplastic
syndromes, or seizures.

*Stress protein 14-3-3 is elevated in the CSF of some patients with CJD

*EEG -The presence of these stereotyped periodic bursts of <200 ms in duration, occurring
every 1–2 s, makes the diagnosis of CJD very
likely.

*Biosafety level 2 practices,

DECONTAMINATION OF CJD PRIONS

*Autoclaving at 134°C for 5 h or treatment with 2 N NaOH for several hours is recommended
for sterilization of
prions

*Amyloid plaques in AD do not correlate with the level of dementia ;****** however, the
level of soluble (oligomeric) Aβ peptide does correlate with memory loss

*The finding that phenothiazines and


acridines inhibit PrPSc formation in cultured cells led to clinical studies of quinacrine in CJD
patient

POSITIVE MED PG
Web: http://www.positivemedpg.com/ Score:
Email: manohrajan@yahoo.co.in 0.00 / 100
Page 24
NIMHANS - DM Neurology -
Exam Title :
MO...
Email : shubham.saha452@gmail.com
Contact : 9932160514

QUESTION 15. NDE3MCtTSFVCSEFNICBTQUhBK3NodWJoYW0uc2FoYTQ1MkBnbWFpbC5jb20rOTkzMjE2M


USU9OIDE0
Following are true about Tic disorder except

a) More common in girls


b) Motor and vocal tics
c) Age of onset before 7 yrs
d) Motor tic precede vocal tics
Correct Answer: A
Your Answer: Unanswered
Explanation

Tics disorders are common in boys which is characterised by multiple motor or vocal tics
appearing before age of7 years.

Haloperidol or pimozide are preffered treatment.

TIC disorder - Multiple motor tics and one or more vocal tics (Tourette’s disorder [ Tic –
involuntary, sudden, rapid, recurrent, nonrhythmic, stereotyped, motor movements or
vocalisation) .

Haloperidol and pimozide most commonly used

MORE NIMHANS FACTS-NIMHANS 2002

True regarding aortic aneurysm is Thoracic aneurysms have better prognosis

Full thickness graft is indicated in Fascial regions for cosmetic effect

The most common complication of Colle’s fracture is FINGER STIFFNESS

In a trauma patient presenting to the casualty the fluid to be given initially is RL

QUESTION 16. NDE3MCtTSFVCSEFNICBTQUhBK3NodWJoYW0uc2FoYTQ1MkBnbWFpbC5jb20rOTkzMjE2M


USU9OIDE1
A patient of Schizophrenia is getting Chlorpromazine, but his auditory hallucinations are not
controlled.The next drug to be given is:

a) Haloperidol
b) Clozapine
c) Sulpride
d) Tianeptin
Correct Answer: B
Your Answer: Unanswered
Explanation

MORE NIMHANS FACTS-NIMHANS 2004

POSITIVE MED PG
Web: http://www.positivemedpg.com/ Score:
Email: manohrajan@yahoo.co.in 0.00 / 100
Page 25
NIMHANS - DM Neurology -
Exam Title :
MO...
Email : shubham.saha452@gmail.com
Contact : 9932160514

A 32 year old male presented with painless lymphnode in the cervical region. Lymph node
biopsy shown normal thyroid gland features. But clinically, on palpation thyroid is normal the
diagnosi is LINGUAL THYROID

All are true regarding Myelofibrosis

a. Tear drop poikilocytes

b. Giant abnormal platelets

c. Leucoerythroblastic blood picture

Water hammer pulse is seen in AR

A little girl who was underweight and hypotonic in infancy is obsessed with food, eats
compulsively, and at age 4, sge is already grossly overweight. She is argumentative,
oppositional, and rigid. She has a narrow face, almond – shaped eyes, and a small mouth -
PRADER WILLI SYNDROME

QUESTION 17. NDE3MCtTSFVCSEFNICBTQUhBK3NodWJoYW0uc2FoYTQ1MkBnbWFpbC5jb20rOTkzMjE2M


USU9OIDE2
Which of the following is the wrong statement regarding the Hounsfield units

a) Air – 1000 HU
b) Fat- 90 HU
c) Muscle – 50 HU
d) White matter-300
Correct Answer: D
Your Answer: Unanswered
Explanation

CT

Material Hounsefield units

Air 1000*

Lung 300

Fat 90*

Water 0

White matter 30

POSITIVE MED PG
Web: http://www.positivemedpg.com/ Score:
Email: manohrajan@yahoo.co.in 0.00 / 100
Page 26
NIMHANS - DM Neurology -
Exam Title :
MO...
Email : shubham.saha452@gmail.com
Contact : 9932160514

Grey matter 40

Muscle 50*

Bone 1000 +*****

*Relative attenuation coefficient

– expressed in Hounsefield unit

MORE NIMHANS FACTS-NIMHANS 2004

Cerebro spinal fluid Is actively secreted by chroidal plexus

“Lesbianism” is also known as -TRIBADISM

EEG pattern in REM sleep is Low amplitude, rapid waves

Plasma ACTH levels are high between these following hours -4.00am-10.00am

QUESTION 18. NDE3MCtTSFVCSEFNICBTQUhBK3NodWJoYW0uc2FoYTQ1MkBnbWFpbC5jb20rOTkzMjE2M


USU9OIDE3
Beaded septum sign is seen in Which of the following condition

a) Mitral stenosis
b) Alveolar proteinosis
c) Lymphangitis carcinomatosa
d) Histiocytosis
Correct Answer: C
Your Answer: Unanswered
Explanation

POSITIVE MED PG
Web: http://www.positivemedpg.com/ Score:
Email: manohrajan@yahoo.co.in 0.00 / 100
Page 27
NIMHANS - DM Neurology -
Exam Title :
MO...
Email : shubham.saha452@gmail.com
Contact : 9932160514

beaded septum sign

CT scans. —This sign consists of irregular and nodular thickening of interlobular septa
reminiscent of a row of beads . It is frequently seen in lymphangitic spread of cancer and less
often in sarcoidosis

Also seen in sarcoidosis

Unilateral lymphangitis -ca lung

Alveolar proteinosis -crazy pavement

Kerley B line –mitral stenosis

MORE NIMHANS FACTS-NIMHANS 2005

SSRI drug which has less effect on CYP isoenzyme -Sertraline

Choline overdose causes

Fishy body odour

Sweating

Voiding of the bladder

Short acing anticholinesterase is EDROPHONIUM

Antipsychotics are safer in the patient with HYPERTENSION

Adverse effects of INH

Peripheral neuritis

POSITIVE MED PG
Web: http://www.positivemedpg.com/ Score:
Email: manohrajan@yahoo.co.in 0.00 / 100
Page 28
NIMHANS - DM Neurology -
Exam Title :
MO...
Email : shubham.saha452@gmail.com
Contact : 9932160514

Hepatitis

Acne

Acneiform eruptions can occur with glucocorticoids, topical or systemic , oral contraceptive
pills, lithium, isoniazid, androgenic steroids, halogens, phenytoin, and phenobarbital

QUESTION 19. NDE3MCtTSFVCSEFNICBTQUhBK3NodWJoYW0uc2FoYTQ1MkBnbWFpbC5jb20rOTkzMjE2M


USU9OIDE4
Lateral thoracic meningocoele is seen in

a) Neurofibromatosis
b) Arnold chiary malformation
c) Dandy walker syndrome
d) diastematomyelia
Correct Answer: A
Your Answer: Unanswered
Explanation

Lat. Thor. meningocele -neurofibromatosis Lumbar meningocele - chiary mal-2

Ant sacral meningocele -caudal regression ,neurofibromatosis

Occipital meningocele -chiary type 3

MORE NIMHANS FACTS-NIMHANS 2005

Elimination of avoidable blindness

POSITIVE MED PG
Web: http://www.positivemedpg.com/ Score:
Email: manohrajan@yahoo.co.in 0.00 / 100
Page 29
NIMHANS - DM Neurology -
Exam Title :
MO...
Email : shubham.saha452@gmail.com
Contact : 9932160514

According to WHO estimates, about 80% of global blindness is avoidable :

WHO and a Task Force of international NGOs have jointly prepared and launched a common
agenda for global action: " VISION 2020 -- The Right to Sight".

VISION 2020 will be implemented through 4 five-year plans, the first one starting in
2000. The three subsequent phases of implementation will commence in 2005, 2010 and 2015
respectively.

Soil is involved as a mode of transmission by faccoral route in the following

Ancylostomasis

Ascariasis

Hydatid disease

Order of steps involved in CHI – SQUARE test

a. Test the null hypothesis

b.Applying the X2 test

c. Finding the degree of freedom

d.Probability tables

State the null and research/alternative hypotheses.

2. Specify the decision rule and the level of statistical significance for the test, i.e., .05, .01, or
.001. (A significance level of .01 would mean that the probability of the chi-square value must
be .01 or less to reject the null hypothesis, a more stringent criterion than .05.)

3. Compute the expected values.

4. Compute the chi-square statistic.

5. Determine the degrees of freedom for the table.

6. Compare the computed chi-square statistic with the critical value of chi-square; reject the
null hypothesis if the chi-square is equal to or larger than the critical value; accept the null
hypothesis if the chi-square is less than the critical value.

7. State a substantive conclusion

QUESTION 20. NDE3MCtTSFVCSEFNICBTQUhBK3NodWJoYW0uc2FoYTQ1MkBnbWFpbC5jb20rOTkzMjE2M


USU9OIDE5
Bare orbit syndrome is seen in which of the following condition

a) Retinoblastoma
b) Neurofibromatosis
c) Enucleation of eye
d) Orbital lymphoma

POSITIVE MED PG
Web: http://www.positivemedpg.com/ Score:
Email: manohrajan@yahoo.co.in 0.00 / 100
Page 30
NIMHANS - DM Neurology -
Exam Title :
MO...
Email : shubham.saha452@gmail.com
Contact : 9932160514
Correct Answer: B
Your Answer: Unanswered
Explanation

Bare orbit syndrome is due to absence of greater wing of sphenoid.It is classically observed in
Neurofibromatosis and produces pulsatile Ex-ophthalmosis.Usually associated with plexiform
Neurofibroma of trigeminal nerve. Retinoblastoma showscalcification

Bare orbit sign , is described as a characteristic appearance of orbit, where the innominate
line is absent. The innominate line is a projection of the greater wing of the sphenoid, and its
absence or destruction is responsible for this appearance.
It is the classical frontal radiograph sign of neurofibromatosis type I (NF1). In addition to bare
orbit sign, it is accompanied by egg shaped enlargement of anterior orbital rim, bony defect in
posterior orbit and anteroposterior enlargement of middle cranial fossa .
Bare orbit sign may also be seen in destruction of sphenoid wing, most commonly in neuroblast
oma metastases.

MORE NIMHANS FACTS-NIMHANS 2005

PYROSIS

Seen in reflux esophagitis

May radiate to the neck and the angles of the jaw

Produced by infusion of neural hyperosmolar solutions into the esophagus

The diagnosis is one of elimination, based upon the Rome III criteria : 1) burning retrosternal
discomfort; 2) elimination of heart attack and GERD as the cause; and 3) no esophageal motility
disorders .

POSITIVE MED PG
Web: http://www.positivemedpg.com/ Score:
Email: manohrajan@yahoo.co.in 0.00 / 100
Page 31
NIMHANS - DM Neurology -
Exam Title :
MO...
Email : shubham.saha452@gmail.com
Contact : 9932160514

Wilson disease can present as

a. Dystonia

b. Fulminant hepatitis

c. Repeated spontaneous abortion

QUESTION 21. NDE3MCtTSFVCSEFNICBTQUhBK3NodWJoYW0uc2FoYTQ1MkBnbWFpbC5jb20rOTkzMjE2M


USU9OIDIw
Cytochrome oxidase enzyme contains which of the following metals in thier chemical structure

a) Iron and Copper


b) Iron and Zinc
c) Copper and Zinc
d) Iron and Manganes
Correct Answer: A
Your Answer: Unanswered
Explanation

Metalloenzymes Iron : Cytochromeoxidase*****, Catalase, Peroxidase, Xanthine oxidase

Copper : Tyrosinase, Cytochrome oxidase*****, SOD, Lysyl oxidase

Zinc : Carbonic anhydrase, Carboxy peptidase,Alcohol dehydrogenase, ALP, Glutamate


dehydrogenase, RNA polymerase

Magnesium : Hexokinase, Enolase, PFK

Manganese : Hexokinase, Enolase

Molybdenum : Xanthine oxidase

Calcium: Lipase, Lecithinase

MORE NIMHANS FACTS-NIMHANS 2005

Most potent anti TB drug -RFM

Risk factors for active tuberculosis among persons who have been infected with tubercle bacilli
-SILICOSIS

Most common cause of postural hypotension is PERIPHERAL NEUROPATHY

QUESTION 22. NDE3MCtTSFVCSEFNICBTQUhBK3NodWJoYW0uc2FoYTQ1MkBnbWFpbC5jb20rOTkzMjE2M


NUSU9OIDIx
All of the following statements are correct about metabolism in brain except :

POSITIVE MED PG
Web: http://www.positivemedpg.com/ Score:
Email: manohrajan@yahoo.co.in 0.00 / 100
Page 32
NIMHANS - DM Neurology -
Exam Title :
MO...
Email : shubham.saha452@gmail.com
Contact : 9932160514

a) Fatty acids are utilized in starvation


b) 60 % of total energy is utilized during resting stage
c) Ketone bodies are used in starvation
d) Has no stored energy
Correct Answer: A
Your Answer: Unanswered
Explanation

Tissue FED Fasting Starvation

Brain Glucose Glucose Ketone bodies

Heart Fatty acids Fatty acids Ketone bodies

Liver Glucose Fatty acids Amino aicds

Muscle Glucose Fatty acids Fatty acids

Adipose tissue Fatty acid - -

MORE NIMHANS FACTS-NIMHANS 2005

Drug induced SLE seen in

a. Hydralazine

b. Procainamide

c. Isoniazid

The characteristic histopathologic feature of Parkinson's disease is the Lewy body, an eosinop
hilic cytoplasmic inclusion that contains both neurofilaments and alfa-synuclein .

Abdominal aortic aneurysm is common in Males & infrarenal in location

Most common cause of abdominal aortic aneurysm is ATHEROSCLEROSIS

Hyperkalemic periodic paralysis associated with Graves’ disease is seen commonly in A


SIAN MALES

The following are the genetic syndromes associated with DM

a. Prader Willi syndrome

b. Down’s syndrome

c. Klinefelter’s syndrome

POSITIVE MED PG
Web: http://www.positivemedpg.com/ Score:
Email: manohrajan@yahoo.co.in 0.00 / 100
Page 33
NIMHANS - DM Neurology -
Exam Title :
MO...
Email : shubham.saha452@gmail.com
Contact : 9932160514

QUESTION 23. NDE3MCtTSFVCSEFNICBTQUhBK3NodWJoYW0uc2FoYTQ1MkBnbWFpbC5jb20rOTkzMjE2M


NUSU9OIDIy
Hepotocytes loaded with minimally branched glycogen is seen in which of the following
glycogen storage disorder

a) Von Gierkes disease


b) Tauris disease
c) Cori’s disease
d) Anderson’s disease
Correct Answer: D
Your Answer: Unanswered
Explanation

Anderson’s disease:

Deficiency : defect in Branching enzyme.

Effect : during glycogen synthesis, branching points will not introduced properly leading to
accumulation of abnormal glycogen with minimal branches.

Cori’s disease:

Deficiency : defect in Debranching enzyme.

Effect : during glycogenolysis, branching points will not be broken properly leading to
accumulation of abnormal glycogen with multiple short branches.

Glycogen storage disorders:

Inborn errors of metabolism of glycogen due to which abnormal glycogen gets deposited in
liver, spleen & muscles etc.

Diseases Defective enzyme

Von Gierke’s disease Gluc-6-phosphotase

Pompe’s disease Lysosomal Acid maltase

Limit dextrinosis (Cori’s) Debranching enzyme

Amylopectinoses (Anderson’s) Branching enzyme

Mc Ardle’s Myophosphorylase

POSITIVE MED PG
Web: http://www.positivemedpg.com/ Score:
Email: manohrajan@yahoo.co.in 0.00 / 100
Page 34
NIMHANS - DM Neurology -
Exam Title :
MO...
Email : shubham.saha452@gmail.com
Contact : 9932160514

Her’s Liver phosphorylase

Tauris Muscle & RBC Phosphofructokinase

Type VIII Phosphorylase kinase

Glycogen storage diseases presenting with liver involvement and hypoglycemia.

I, III, VI , IV

Muscle involvement:

V, VII, III.

Cardiac involvement:

Pompe’s disease (II)

MORE NIMHANS FACTS-NIMHANS 2005

Drugs used in alcohol addiction are

a. Disulfiram

b. Metronidazole

C.. Fluoxetine

SIADH

Investigations for confirming the diagnosis are 1) low serum osmolality, 2) high urine
osmolality 3) Urine Na + concentration usually >40 mmol/L .
4) Hypouricemia due to the uricosuric state induced by volume expansion 5) absence of
hypothyroidism

QUESTION 24. NDE3MCtTSFVCSEFNICBTQUhBK3NodWJoYW0uc2FoYTQ1MkBnbWFpbC5jb20rOTkzMjE2M


NUSU9OIDIz
Set of data which correctly defines restrictive lung disease is :

a) ↑ FRC, ↑ compliance of lung tissue


b) ↑ FEV 1 /FVC, ↓ compliance of lung tissue
c) ↓ FEV 1 /FVC, ↓ compliance of lung tissue
d) ↑ TLC, RVis ↓
Correct Answer: B
Your Answer: Unanswered
Explanation

POSITIVE MED PG
Web: http://www.positivemedpg.com/ Score:
Email: manohrajan@yahoo.co.in 0.00 / 100
Page 35
NIMHANS - DM Neurology -
Exam Title :
MO...
Email : shubham.saha452@gmail.com
Contact : 9932160514

Restrictive lung disease

* VC ↓ * FEV1 – Normal * Total lung capacity ↓

* MVV - ↓

* Compliance - ↓

MORE FACTS

Surfactant

Substance %

POSITIVE MED PG
Web: http://www.positivemedpg.com/ Score:
Email: manohrajan@yahoo.co.in 0.00 / 100
Page 36
NIMHANS - DM Neurology -
Exam Title :
MO...
Email : shubham.saha452@gmail.com
Contact : 9932160514

Dipalmitoyl phosphatidyl choline 62

Phosphatidyl glycerol 5

Neutral lipids 13

Other phospholipid 10

Protein 8

Carbohydrate 2

* Produced by Type II alveolar epithelial cells * Decreases surface Tension Vital capacity

Increase Decrease

A. Physiological
* Females
A. Physiological * Old age
* Males
* Standing
* Sitting or lying

* Pregnancy

B. Pathological
* Ascites
B Pathological * Pneumothorax, effusion

* Pulmonary fibrosis

Tracheostomy

DECRREASES dead space

MORE NIMHANS FACTS-NIMHANS 2005

Drug of choice in amoebic liver abscess is METRONIDAZOLE

Primary Hypothyroidism is seen in all except [Nim 2005]

a. Iodine deficiency

POSITIVE MED PG
Web: http://www.positivemedpg.com/ Score:
Email: manohrajan@yahoo.co.in 0.00 / 100
Page 37
NIMHANS - DM Neurology -
Exam Title :
MO...
Email : shubham.saha452@gmail.com
Contact : 9932160514

b. Iodine excess

c. Lithium

d. Bexarotene

Primary Hypothyroidism is seen in

a. Iodine deficiency

b. Iodine excess

c. Lithium

Therapy with the retinoid X receptor agonist bexarotene is associated with hypothyroidism
caused by decreased pituitary TSH secretion-SECONDARY HYPOTHYROIDISM

ANS -D

QUESTION 25. NDE3MCtTSFVCSEFNICBTQUhBK3NodWJoYW0uc2FoYTQ1MkBnbWFpbC5jb20rOTkzMjE2M


NUSU9OIDI0
A 25 YEAR old college student met with an RTA and was admitted in an ICU. He was opening
his eyes on calling and was making incomprehensible sounds. He withdrew his leg on touching.
Calculate the Glascow coma scale

a) 5
b) 6
c) 7
d) 9
Correct Answer: D
Your Answer: Unanswered
Explanation

Glasgow Coma Scale(AI-2006*-AIIMS NOV-2013***)

Eye-opening (E)

* Spontaneous4
* To speech3
* To pain2 * Nil1

Best motor response (M)

POSITIVE MED PG
Web: http://www.positivemedpg.com/ Score:
Email: manohrajan@yahoo.co.in 0.00 / 100
Page 38
NIMHANS - DM Neurology -
Exam Title :
MO...
Email : shubham.saha452@gmail.com
Contact : 9932160514

* Obeys6
* Localises5
* Withdraws4 * Abnormal flexion3

* Extensor response2

* Nil1

Verbal response (V)

* Orientated5
* Confused conversation4
* Inappropriate words3 * Incomprehensible sounds2

* Nil1

Coma score = E + M + V

* Minimum3
* Maximum15

MORE NIMHANS FACTS-NIMHANS 2005

HYSTERICAL GAIT DISORDER

Hysterical gait disorders present with abnormal posture with wastage of muscular energy ( asta
sia-abasia) , extreme slow motion, and dramatic fluctuations over time.

Hysterical gait disorder is characterized by

b. Similar presentation like atrophy of the vermis

c. No leg circumduction

d. No hyperreflexia

QUESTION 26. NDE3MCtTSFVCSEFNICBTQUhBK3NodWJoYW0uc2FoYTQ1MkBnbWFpbC5jb20rOTkzMjE2M


NUSU9OIDI1
The facilitatory area for micturition is situated in the

a) pontine region
b) midbrain region
c) medulla
d) spinal cord

POSITIVE MED PG
Web: http://www.positivemedpg.com/ Score:
Email: manohrajan@yahoo.co.in 0.00 / 100
Page 39
NIMHANS - DM Neurology -
Exam Title :
MO...
Email : shubham.saha452@gmail.com
Contact : 9932160514
Correct Answer: A
Your Answer: Unanswered
Explanation

POSITIVE MED PG
Web: http://www.positivemedpg.com/ Score:
Email: manohrajan@yahoo.co.in 0.00 / 100
Page 40
NIMHANS - DM Neurology -
Exam Title :
MO...
Email : shubham.saha452@gmail.com
Contact : 9932160514

Reflex Control

* Fibers in the pelvic nerves are the afferent limb of the voiding reflex, and the parasympathetic
fibers to the bladder that constitute the efferent limb also travel in these nerves. The reflex is
integrated in the sacral portion of the spinal cord- volume of urine in the bladder that normally
initiates a reflex contraction is about 300 to 400 mL.

* The sympathetic nerves to the bladder play no part in micturition.

* The threshold for the voiding reflex, like the stretch reflexes, is adjusted by the activity of
facilitatory and inhibitory centers in the brainstem.

* There is a facilitatory area in the pontine region******* and an inhibitory area in the
midbrain .

* After transection of the brain stem just above the pons, the threshold is lowered and less
bladder filling is required to trigger it

* After transection at the top of the midbrain, the threshold for the reflex is essentially normal.

* There is another facilitatory area in the posterior hypothalamus. Humans with lesions in the
superior frontal gyrus have a reduced desire to urinate and difficulty in stopping micturition
once it has commenced.

Effects of Deafferentation

* When the sacral dorsal roots are cut in experimental animals or interrupted by diseases of the
dorsal roots, such as tabes dorsalis in humans, all reflex contractions of the bladder are

POSITIVE MED PG
Web: http://www.positivemedpg.com/ Score:
Email: manohrajan@yahoo.co.in 0.00 / 100
Page 41
NIMHANS - DM Neurology -
Exam Title :
MO...
Email : shubham.saha452@gmail.com
Contact : 9932160514
abolished. The bladder becomes distended, thin-walled, and hypotonic, but some contractions
occur because of the intrinsic response of the smooth muscle to stretch.

Effects of Denervation

* When the afferent and efferent nerves are both destroyed, as they may be by tumors of the
cauda equina or filum terminale, the bladder is flaccid and distended for a while.

* The muscle of the "decentralized bladder " becomes active, with many contraction waves
that expel dribbles of urine out of the urethra.

* The bladder becomes shrunken and the bladder wall hypertrophied .

* The reason for the difference between the small, hypertrophic bladder seen in this condition
and the distended, hypotonic bladder seen when only the afferent nerves are interrupted is not
known.

* Three major types of bladder dysfunction are due to neural lesions: (1) the type due to
interruption of the afferent nerves from the bladder, (2) the type due to interruption of both
afferent and efferent nerves, and (3) the type due to interruption of facilitatory and inhibitory
pathways descending from the brain. In all three types the bladder contracts, but the
contractions are generally not sufficient to empty the viscus completely, and residual urine is
left in the bladder.

Effects of Spinal Cord Transection

* During spinal shock, the bladder is flaccid and unresponsive. It becomes overfilled, and urine
dribbles through the sphincters (overflow incontinence).

* After spinal shock has passed, the voiding reflex returns, although there is, of course, no
voluntary control and no inhibition or facilitation from higher centers when the spinal cord is
transected.

* In some instances, the voiding reflex becomes hyperactive, bladder capacity is reduced, and
the wall becomes hypertrophied- spastic neurogenic bladder.

POSITIVE BITS- Renal blood flow is regulated by norepinephrine (constriction, reduction of


flow), dopamine (vasodilation, increases flow), angiotensin II (constricts), prostaglandins
(dilation in the renal cortex and constriction in the renal medulla), and acetylcholine
(vasodilation).

MORE NIMHANS FACTS-NIMHANS 2005

Resistant schizophrenia is treated with CLOZAPINE

LITHIUM SIDE EFFECTS

Agent and Dosing Side Effects

Lithium Common Side Effects

POSITIVE MED PG
Web: http://www.positivemedpg.com/ Score:
Email: manohrajan@yahoo.co.in 0.00 / 100
Page 42
NIMHANS - DM Neurology -
Exam Title :
MO...
Email : shubham.saha452@gmail.com
Contact : 9932160514

* Anorexia/diarrhea,
fine tremor, thirst,
polyuria, fatigue,
weight gain, acne,
folliculitis,
neutrophilia,
hypothyroidism

* Blood level is
Starting dose: 300 increased by thiazides,
mg bid or tid tetracyclines, and
NSAIDs
Therapeutic blood
level: 0. 8–1. 2 meq/ * Blood level is
L decreased by
bronchodilators,
verapamil, and
carbonic anhydrase
inhibitors

* Rare side effects: Neur


otoxicity, renal toxicity,
hypercalcemia, ECG
changes

QUESTION 27. NDE3MCtTSFVCSEFNICBTQUhBK3NodWJoYW0uc2FoYTQ1MkBnbWFpbC5jb20rOTkzMjE2M


NUSU9OIDI2
Cerebral blood flow is increase in all except

a) chronic anaemia
b) inhalation of 5% carbon dioxide
c) seizures
d) inhalation of hyperbaric oxygen
Correct Answer: D
Your Answer: Unanswered
Explanation

• CBF = CPP / CVR


• The hagen-Poiseuille equation
• Cerebral Blood Flow = Δ P π R4 8 η l
• where: Δ P = cerebral perfusion pressure
• R = radius of the blood vessels
• η = viscosity of the fluid (blood)
• l = length of the tube (blood vessels)
• π = constant, 3.14
• high oxygen consumption of the brain of 3.3ml.100g-1.min -1 (50ml.min-1 in

POSITIVE MED PG
Web: http://www.positivemedpg.com/ Score:
Email: manohrajan@yahoo.co.in 0.00 / 100
Page 43
NIMHANS - DM Neurology -
Exam Title :
MO...
Email : shubham.saha452@gmail.com
Contact : 9932160514
• total) which is 20% of the total body consumption

Cerebral blood flow

• In an adult, CBF is typically 750 millilitres per minute or 15% of the cardiac output .
• This equates to 50 to 54 millilitres of blood per 100 grams of brain tissue per minute
• ( ischemia ) results if blood flow to the brain is below 18 to 20 ml per 100 g per minute,
and tissue death occurs if flow dips below 8 to 10 ml per 100 g per minute
• CVR is controlled by four major mechanisms :
• Metabolic control (or 'metabolic autoregulation')
• Pressure autoregulation
• Chemical control (by arterial pCO 2 and pO 2 )
• Neural control
• CBF increases once PaO2 drops below about 6.7kPa so that cerebral oxygen delivery
remains constant.
• Hypoxia acts directly on cerebral tissue to promote the release of adenosine, and in some
cases prostanoids that contribute significantly to cerebral vasodilatation.
• Hypoxia also acts directly on cerebrovascular smooth muscle to produce
hyperpolarisation and reduce calcium uptake, both mechanisms enhancing vasodilatation.
• MORE NIMHANS FACTS-NIMHANS 2005

• Rett’s syndome is characterized by

a. Common in girls

b. Severe mental retardation

c. Hand wringing present

Descending order of types of viruses causing paralytic polio Type 1, type 3, type 2
• Presenting symptom of Normotensive Hydrocephalus is VERTIGO
• Pencil – in – cup deformity is seen in PSORIATIC ARTHRITIS

QUESTION 28. NDE3MCtTSFVCSEFNICBTQUhBK3NodWJoYW0uc2FoYTQ1MkBnbWFpbC5jb20rOTkzMjE2M


NUSU9OIDI3
Which of the following nuclei belongs to the General Visceral Efferent column?

a) Facial nerve nucleus


b) Trigeminal nerve nucleus
c) Dorsal nucleus of vagus
d) Nucleus ambiguous
Correct Answer: C
Your Answer: Unanswered
Explanation

POSITIVE MED PG
Web: http://www.positivemedpg.com/ Score:
Email: manohrajan@yahoo.co.in 0.00 / 100
Page 44
NIMHANS - DM Neurology -
Exam Title :
MO...
Email : shubham.saha452@gmail.com
Contact : 9932160514

• A, C & D belong to the Special Visceral Efferent Colum (Branchial Efferent Column)
• The nuclei belonging to the General Visceral Efferent column give rise to parasympathetic
fibres, which form the cranial outflow of parasympathetic nervous system. They are four
in number and are situated in the brainstem.

They include

• ◦ Edinger Westphal nucleus (in the midbrain), the fibres of which pass through the
oculomotor nerve to innervate the sphincter papillae and ciliaris muscles.
◦ Superior salivatory and lacrimatory nuclei (in the pons), the fibres of which pass
through the facial nerve to provide secretomotor innervation to submandibular and
sublingual salivary glands and all glands of nasal and oral cavities and lacrimal
gland.
◦ Inferior salivatory nucleus (in the medulla), the fibres of which pass through the
glossopharyngeal nerve to provide secretomotor innervation to parotid gland.
◦ Dorsal vagal nucleus (in the medulla), the fibres of which pass through the vagus
to provide parasympathetic innervation to thoracic viscera and abdominal viscera
down to the midgut. i.e. up to the junction between the right two thirds and left one
third of transverse colon.

The facial nerve has three nuclei.

1.Motor nucleus belongs to the Branchial Efferent (Special Visceral Efferent) column. Its fibres
innervate the muscles derived from the first pharyngeal (branchial) arch mesoderm. I.e.
muscles of facial expression and others.

2.Superior salivatory nucleus belongs to the General Visceral Efferent column. Its fibres
innervate the submandibular and sublingual salivary glands via. Submandibular ganglion and
glands of nasal and oral cavities and lacrimal gland via. Pterygopalatine ganglion.

POSITIVE MED PG
Web: http://www.positivemedpg.com/ Score:
Email: manohrajan@yahoo.co.in 0.00 / 100
Page 45
NIMHANS - DM Neurology -
Exam Title :
MO...
Email : shubham.saha452@gmail.com
Contact : 9932160514

3.Nucleus of tractus solitarius belongs to the Special Visceral Afferent column. This nucleus
receives the sensation of taste from the anterior two thirds of tongue via. Facial nerve and, the
Posterior one third of tongue via. Glossopharyngeal and vagus nerves.

The trigeminal nerve has four nuclei. One motor nucleus belonging to the Branchial (Special
Visceral) Efferent column and three sensory nuclei belonging to the General Somatic Afferent
column.

• ◦ Fibres from motor nucleus pass through the mandibular nerve to innervate the
muscles derived from the first branchial arch mesoderm. i.e. muscles of mastication
and others.
◦ Main sensory nucleus receiving touch and pressure sensation from the anterior part
of head.
◦ Spinal nucleus receiving the sensation of pain and temperature from the anterior
part of head.
◦ Mesencephalic nucleus receiving proprioceptive sensation from the muscles of
mastication and other muscles innervated through mandibular nerve.

The nucleus ambiguous is a motor nucleus belonging to the Somatic Efferent column. The IX, X
and XI cranial nerves arise from this nucleus.

MORE NIMHANS FACTS-NIMHANS 2005

In genital TB, the menstrual blood Does not contain tuberculids

In PCOD, Increase LH, increase estrogen

Oral contraceptive pills are contraindicated in

Lactating woman

Patients on sodium valproate

Thyroid disease

Oral contraceptives offer a protective effect against ovarian epithelial tumors and endometrial
cancers and decrease bone fractures and colorectal cancer

Oral contraceptives are contraindicated for cyanotic women because of the enhanced risk of
vascular thrombosis

Oral contraceptives can cause hepatic adenoma, hepatocellular carcinoma and hepatic vein
occlusion (Budd-Chiari syndrome).

Oral Contraceptive Pills

* They act by suppressing ovulation, changing cervical mucus, and altering the endometrium.
The current formulations are made from synthetic estrogens and progestins. The estrogen
component of the pill consists of ethinyl estradiol or mestranol.

* Levonorgestrel is the most androgenic of the progestins and should be avoided in patients
with hyperandrogenic symptoms.

* Contraindications of OC pills are given below

POSITIVE MED PG
Web: http://www.positivemedpg.com/ Score:
Email: manohrajan@yahoo.co.in 0.00 / 100
Page 46
NIMHANS - DM Neurology -
Exam Title :
MO...
Email : shubham.saha452@gmail.com
Contact : 9932160514

Absolute

Previous thromboembolic event or stroke

History of an estrogen-dependent tumor

Active liver disease

Pregnancy

Undiagnosed abnormal uterine bleeding

Hypertriglyceridemia

Women aged >35 years who smoke heavily

Disease Risks

Increased

Coronary heart disease

Hypertension

Venous thrombosis—relative risk 4

Stroke

Cerebral vein thrombosis

Cervical cancer—relative risk 2–4 Breast


cancer(mainly BRCA1)

Decreased

Ovarian cancer—50% reduction in risk

POSITIVE MED PG
Web: http://www.positivemedpg.com/ Score:
Email: manohrajan@yahoo.co.in 0.00 / 100
Page 47
NIMHANS - DM Neurology -
Exam Title :
MO...
Email : shubham.saha452@gmail.com
Contact : 9932160514

Endometrial cancer—40% reduction in risk

QUESTION 29. NDE3MCtTSFVCSEFNICBTQUhBK3NodWJoYW0uc2FoYTQ1MkBnbWFpbC5jb20rOTkzMjE2M


NUSU9OIDI4
Posterior gastric artery is a branch of which of the following artery

a) Left gastric artery


b) Right gastric artery
c) Splenic artery
d) Hepatic artery
Correct Answer: C
Your Answer: Unanswered
Explanation

POSITIVE MED PG
Web: http://www.positivemedpg.com/ Score:
Email: manohrajan@yahoo.co.in 0.00 / 100
Page 48
NIMHANS - DM Neurology -
Exam Title :
MO...
Email : shubham.saha452@gmail.com
Contact : 9932160514

IMPORTANT POINTS

1.TORTUOUS ARTERIES :

1.Facial artery, uterine artery, splenic artery, posterior inferior cerebellar artery, lingual artery.

2.BLOOD LESS STRUCTURES :

Cartilage, cornea, lens, epithelium.

3.STRUCTURES WITHOUT LYMPHATICS :

2.Epidermis, Hair, Nails, Cornea, Cartilage central nervous system, bone marrow.

4.PORTAL CIRCULATION :

System of vessels between two capillary beds.

1. Hypothalamic – pituary fortal system.

2. Hepatic system

3. Adrenal, kidney.

POSITIVE MED PG
Web: http://www.positivemedpg.com/ Score:
Email: manohrajan@yahoo.co.in 0.00 / 100
Page 49
NIMHANS - DM Neurology -
Exam Title :
MO...
Email : shubham.saha452@gmail.com
Contact : 9932160514

5.VALVELESS VEINS :

Superior vena cava : Inferior vena cava: veins in Head & Neck (Ophthalmic veins) Dural
Sinuses: veetenral veins: Cardiac veins: Pulmonary veins: Hepatic veins: Portal veins: Pelvic
veins: Emissary Veins: Cerebral veins

6.VEINS CARRYING OXYGENATED BLOOD :

1. Pulmonary veins

2. Umbilical veins

7.ARTERIES CARRYING VENOUS BLOOD :

1. Pulmonary artery

2. Umbilical artery

8.SINUSOIDS – lined by endothelium, present in liver, spleen, adrenal, pituitary, lymphnode.

9.THE END ARTERIES : (ARTERY WHICH DO NOT ANASTOMOSE)

Spleen Heart Lungs

Kidneys Metaphysis of long bone Liver

Brain Central branch of cerebral .A

Central artery of Retina

Parts of gastrointestinal tract (vasa recta of mesenteric artery)

The functional end arteries – anastomose, with much smaller arteries. Is not sufficient to
maintain blood supply of the part.

(Coronary arteries)

10.THE ARTERIO-VENOUS ANASTOMOSIS

These are sites where blood is transferred from the arteries to the veins without passing
through capillary plexus.

• The A-V anastomoses lie in organs, whose functions are intermittent. These also help in
temperature regulation.

Skin of apical part of fingers Nose

Lips Ears

MORE NIMHANS FACTS-NIMHANS 2005

Early dumping syndrome

Early dumping takes place 15–30 minutes after meals and consists of crampy abdominal
discomfort, nausea, diarrhea, tachycardia, palpitations, diaphoresis, and rarely, syncope. Ca
use is due to rapid emptying of hyperosmolar gastric contents into the small intestine, leading

POSITIVE MED PG
Web: http://www.positivemedpg.com/ Score:
Email: manohrajan@yahoo.co.in 0.00 / 100
Page 50
NIMHANS - DM Neurology -
Exam Title :
MO...
Email : shubham.saha452@gmail.com
Contact : 9932160514
to plasma volume contraction and acute intestinal distention. Vasoactive intestinal
polypeptide, neurotensin, motilin may play a role.

*Late phase dumping takes place 90 min to 3 h after meals. Vasomotor symptoms are light-
headedness, diaphoresis, palpitations, tachycardia, and syncope predominate during this
phase. It is due to hypoglycemia from excessive insulin release.

* Precipitating factors are meals rich in simple carbohydrates (especially sucrose) and high
osmolarity. Ingestion of large amounts of fluids may also contribute.

QUESTION 30. NDE3MCtTSFVCSEFNICBTQUhBK3NodWJoYW0uc2FoYTQ1MkBnbWFpbC5jb20rOTkzMjE2M


NUSU9OIDI5
Which of the following muscle is considered as the Abductor of vocal cord

a) Posterior cricoarytenoid
b) Cricothroid
c) Interarytenoid
d) Lateral cricoarytenoid
Correct Answer: A
Your Answer: Unanswered
Explanation

POSITIVE MED PG
Web: http://www.positivemedpg.com/ Score:
Email: manohrajan@yahoo.co.in 0.00 / 100
Page 51
NIMHANS - DM Neurology -
Exam Title :
MO...
Email : shubham.saha452@gmail.com
Contact : 9932160514

Glottis – Vocal cord

* Ant 3/5 – intermembranous part

* Post 2/5 – intercartilagenous part

Movement Muscles

* Abduction * Postr. Crico arytenoid*****

* Lateral crico arytenoid * Transverse arytenoid


* Adductors
* Oblique arytenoideus

* Tensor * Cricothyroid * Vocalis

* Relaxors * Thyroarytenoideus

POSITIVE MED PG
Web: http://www.positivemedpg.com/ Score:
Email: manohrajan@yahoo.co.in 0.00 / 100
Page 52
NIMHANS - DM Neurology -
Exam Title :
MO...
Email : shubham.saha452@gmail.com
Contact : 9932160514

* Opening inlet of larynx * Thyro epiglotticus

Tit bits * All muscles are supplied by Recurrent laryngeal N except

--> cricothyroid (Ext. larynged N)

* Arteries – <st1>Superior, Inferior thyroid</st1>

* Lymphatics

a) Above vocal cord * Upper deep cervical

* Below vocal cord * Pretracheal, plaryngeal

Nerve supplySensory

-Above the vocal cord – Int. laryngeal N

- Below the vocal cord

– Recurrent laryngeal N

Muscles of Larynx

Action Intrinsic Muscle

Abductor of Vocal Cord Posterior Crico Arytenoid

Thyro arytenoids (external part) Transverse arytenoids


ADDuctor of Vocal Cord
Lateral Crico arytenoids

Tensor of vocal cord Crico Thyroid Thyro Arytenoid (internal part i.e. vocalis )

Openers of Laryngeal
Thyroepiglotti (part of thyro arytenoids)
inlet

Inter arytenoids (oblique part) Inter arytenoids (Posterior oblique part


Closers of laryngeal known as
inlet
Aryepiglottic)

Nerve supply

POSITIVE MED PG
Web: http://www.positivemedpg.com/ Score:
Email: manohrajan@yahoo.co.in 0.00 / 100
Page 53
NIMHANS - DM Neurology -
Exam Title :
MO...
Email : shubham.saha452@gmail.com
Contact : 9932160514

Sensory Upto the level of vocal


Motor All intrinsic ms. are supplied by recurrent laryngeal n. folds - Internal laryngeal n.
except for cricothyroid which is supplied by external laryngeal
nerve Below the level of vocal folds -
Recurrent laryngeal n.

MORE NIMHANS FACTS-NIMHANS 2005

Retinal output is mainly from Ganglion cell layer

Suprasellar aneurysms causes BITEMPORAL HEMIANOPIA

GRADENIGO SYNDROME

Syndromes associated with 6th nerve are listed below

Syndrome Effect

Site- Dorsal Pons

Effect

- Ipsilateral lat. Gazepalsy


1. Foville (6 th )

- Ipsilat 7 th N Palsy

- Contralat Lateral
Hemiparesis

Site- Ventral PONS

Effect- Similar to

1. Except eye findings


2. Milard Gubler - LR weakness only instead
of gaze palsy

(Because Abducen fascicles


are involved, not the
nucleus)

POSITIVE MED PG
Web: http://www.positivemedpg.com/ Score:
Email: manohrajan@yahoo.co.in 0.00 / 100
Page 54
NIMHANS - DM Neurology -
Exam Title :
MO...
Email : shubham.saha452@gmail.com
Contact : 9932160514

Site- Petrous Apex


(Mastoiditis)

Effect
3. Gradenigo
- Deafness

- Pain

- Ipsilat 6th N Palsy

QUESTION 31. NDE3MCtTSFVCSEFNICBTQUhBK3NodWJoYW0uc2FoYTQ1MkBnbWFpbC5jb20rOTkzMjE2M


USU9OIDMw
Tentorium cerebelli is innervated by recurrent tentorial nerve, a branch of

a) Maxillary brach of trigeminal nerve


b) Mandibular branch of trigeminal nerve
c) Ophthalmic branch of trigeminal nerve
d) Trochlear nerve
Correct Answer: C
Your Answer: Unanswered
Explanation

Ophthalmic branch of trigeminal nerve- Cranial nerve V A

POSITIVE MED PG
Web: http://www.positivemedpg.com/ Score:
Email: manohrajan@yahoo.co.in 0.00 / 100
Page 55
NIMHANS - DM Neurology -
Exam Title :
MO...
Email : shubham.saha452@gmail.com
Contact : 9932160514

POSITIVE MED PG
Web: http://www.positivemedpg.com/ Score:
Email: manohrajan@yahoo.co.in 0.00 / 100
Page 56
NIMHANS - DM Neurology -
Exam Title :
MO...
Email : shubham.saha452@gmail.com
Contact : 9932160514

MORE NIMHANS FACTS-NIMHANS 2006

Diabetes mellitus is associated with

a. Down’s syndrome

c. Klinefelter’s syndrome

d. Turner’s syndrome

Most common cause of pleural effusion in HIV patients

A wide variety of infectious and malignant conditions cause pleural effusions in HIV infected
patients, the most common cause in this group was Kaposi's sarcoma

The earliest pathological change in x-ray finding of Ulcerative colitis includes MUCOSAL
GRANULARITY

The earliest feature of multiple sclerosis is OPTIC NEURITIS

QUESTION 32. NDE3MCtTSFVCSEFNICBTQUhBK3NodWJoYW0uc2FoYTQ1MkBnbWFpbC5jb20rOTkzMjE2M


USU9OIDMx
Corona radiata of ovum is from which of the following structure

POSITIVE MED PG
Web: http://www.positivemedpg.com/ Score:
Email: manohrajan@yahoo.co.in 0.00 / 100
Page 57
NIMHANS - DM Neurology -
Exam Title :
MO...
Email : shubham.saha452@gmail.com
Contact : 9932160514

a) Cummulus ovarius
b) Zona pellucida
c) Formative yolk
d) Follicular cells
Correct Answer: A
Your Answer: Unanswered
Explanation

OVUM

Consists of cell membrane, Nucleus, Cytoplasm and outter coverings

Outter coverings-Zona pellucida and Corona radiata*

Cytoplasm

Ooplasm or yolk

Formative yolk

Nutritive yolkor deutroplasm *

Human oocyte- Microlecithal

(Bird – Macro lecithal)

Follicular cells (Granulosa cells)

layer of flattened epithelial cells

surrounding primary oocyte

outter layer-stratum granulosum

POSITIVE MED PG
Web: http://www.positivemedpg.com/ Score:
Email: manohrajan@yahoo.co.in 0.00 / 100
Page 58
NIMHANS - DM Neurology -
Exam Title :
MO...
Email : shubham.saha452@gmail.com
Contact : 9932160514

inner layer-cummulus ovaricus*

Cummulus ovaricus

the outter cells disappear

adjacent to the zona pellucida are arranged radially- Zona radiata*

Granulosa cells (Follicular cells) attached to Zona pellucida elongate forming – Corona
Radiata*

Tit bits

*Zona pellucida disappears by day 5 of devolpment

MORE NIMHANS FACTS-NIMHANS 2006

The clinical feature of normal pressure hydrocephalus is GAIT DISTURBANCE

Drug of choice for social phobia is SSRIs

Treatment : antidepressant medication is the drug of choice.. Selective serotonin reuptake


inhibitors (SSRIs) like Fluoxetine, paroxetine, sertraline are used. Selective serotonin-
norepinephrine reuptake inhibitor (SNRI) venlafaxine also can be used. These drugs should
be started at one-third to one-half of their usual antidepressant dose (5–10 mg fluoxetine, 25–
50 mg sertraline, 10 mg paroxetine, venlafaxine 37. 5 mg). Monoamine oxidase inhibitors
(MAOIs) are also effective mainly in patients with features of atypical depression
(hypersomnia and weight gain). It Typically take 2–6 weeks to become effective.

The drug that is used for Nicotine addiction includes BUPROPION

QUESTION 33. NDE3MCtTSFVCSEFNICBTQUhBK3NodWJoYW0uc2FoYTQ1MkBnbWFpbC5jb20rOTkzMjE2M


USU9OIDMy
A body was identied in a forest region with putrefaction.The first organ to putrefy is which of
the following

a) Trachea and larynx


b) Brain
c) Heart
d) Liver
Correct Answer: A
Your Answer: Unanswered
Explanation

Order of Putrefaction

POSITIVE MED PG
Web: http://www.positivemedpg.com/ Score:
Email: manohrajan@yahoo.co.in 0.00 / 100
Page 59
NIMHANS - DM Neurology -
Exam Title :
MO...
Email : shubham.saha452@gmail.com
Contact : 9932160514

• Trachea and Larynx

• Stomach, intestine, spleen


• Liver, lungs
• Brain
• Heart
• Kidney, uterus
• Skin, muscle
• Bone

Putrefaction

- Slower in water than air

- Slower in salt water

MORE NIMHANS FACTS-NIMHANS 2006

Side effects of INH

a. Rash

c. Hepatitis

d. Peripheral neuritis

“Seal finger” and “whale finger” are associated with infection of Erysipelothrix.Seal finger
respond to doxycycline

Bites of seals , walruses, and polar bears may cause a chronic suppurative infection known as
seal finger due to Mycoplasma

Foramen spinosum transmits MIDDLE MENINGEAL ARTERIES

Incubation period of Diphtheria is 2-4 DAYS

QUESTION 34. NDE3MCtTSFVCSEFNICBTQUhBK3NodWJoYW0uc2FoYTQ1MkBnbWFpbC5jb20rOTkzMjE2M


USU9OIDMz
Person with Eonismattains sexual gratification by which of the following mechanism

a) Getting hurt by partner


b) Seeing a woman nude
c) Fondling female body parts
d) Getting gratification wearing dress of opposite sex
Correct Answer: D
Your Answer: Unanswered
Explanation

DISCUSSION

SEXUAL PERVERSIONS

POSITIVE MED PG
Web: http://www.positivemedpg.com/ Score:
Email: manohrajan@yahoo.co.in 0.00 / 100
Page 60
NIMHANS - DM Neurology -
Exam Title :
MO...
Email : shubham.saha452@gmail.com
Contact : 9932160514

SODOMY

SODOMY denotes homosexual penile-anal intercourse.

Heterosexual penile-anal intercourse is called BUGGERY.

The offender is the active agent, the other partner is the passive agent.

If the passive agent is a child, the practice is known as PAEDERASTY.

Habitual passive agents are called CATAMITES, (fairies, gays or queens)

In India, HIJRAS (castrated males) & ZENANAS (male transvestites)

Buccal coitus (sin of Gomorrah)

Buccal-Penile intercourse is called FELLATIO.

The partner who performs the act is called the FELLATOR, and on whom it is performed is the
FELLATEE.

Buccal-vaginal stimulation is referred to as CUNNILINGUS.

Sexual Paraphilias (perversions)

These form a group of psychosexual disorders, which involves involuntary, repetitive, unusual
acts, on which Sexual Arousal and Orgasm are dependent.

Achievement of sexual gratification by means other than natural sexual intercourse.

Fetishism (Fetichism) Sexual focus is on relatively indestructible objects intimately associated


with the human body.

Transvestism : Cross dressing or Eonism.(AIIMS MAY-2014***)

Dressing in the opposite sex, for the purpose of arousal and as an adjunct to masturbation or
sexual intercourse.

Sadism : sexual arousal and orgasm linked to active infliction of injuries or torture of the
sexual partner.

Masochism : sexual excitement linked with passive experience of physical or emotional


humiliation or torture.

Exhibitionism & Voyeurism

They are paired opposite of the Scoptophiic paraphilia.

Exhibitionism involves repeated acts of exposing one’s genitals to a stranger or unsuspected


person

Voyeurism : perversion with desire to observe the genitals or other private parts of the body of
others – Peeping Tom.

FROTTEURISM : Practiced by a male pervert in a crowded place to drive sexual gratification


by rubbing his private parts against a female’s body.

POSITIVE MED PG
Web: http://www.positivemedpg.com/ Score:
Email: manohrajan@yahoo.co.in 0.00 / 100
Page 61
NIMHANS - DM Neurology -
Exam Title :
MO...
Email : shubham.saha452@gmail.com
Contact : 9932160514

TROILISM : The pervert gets sexual gratification by inducing his wife to sexual intercourse
with another person and by observing the same.

Necrophilia

Sexual arousal and orgasm can be attained only by intercourse with a corpse,

NECROPHAGIA is an extreme form of Sadism, where in sexual gratification is attained by


tearing out the genitals or other part of the body of a corpse and eating them.

Sec. 297 IPC

Who ever with the intention of offering any indignity to any human corpse, shall be punished
with imprisonment of either description for a term which may extend to one year, or with fine,
or both.

PAEDOPHILIA : Preferential sexual activity with children.

SATYRIASIS : Excessive sexual desire among males. These subjects are liable to commit
sexual offences.

NYMPHOMANIA : Excessive sexual desire among woman.

Indecent assault

Indecent assault generally means sex-linked misbehaviour towards a person of opposite sex or
the same sex.

Any offence committed on a female with the intention or knowledge to outrage her modesty.

Sec. 509 IPC

Whoever, intending to insult the modesty of a woman, utters any word, makes any sound or
gesture, or exhibits any object shall be punished with imprisonment which may extend to one
year .

MORE NIMHANS FACTS-NIMHANS 2006

Kocher Debre Semelaigne Syndrome is a rare syndrome with clinical manifestation of


hypothyroidism associated with pseudomuscular hypertrophy.

The absence of painful spasms and pseudomyotonia differentiates this syndrome from its adult
form, which is Hoffmann syndrome

“Salmon patch” is seen in INTERSTITIAL KERATITIS

Acutely or at the early sign includes painful, photophobic , red and watery eye. This is due to
active corneal inflammation resulting in vascular invasion and stromal necrosis which can be
diffuse or localized. This cause the pinkish discoloration of what was a clear transparent normal
corneal tissue (called Salmon patch of Hutchinson ).

Conjunctival lymphoma is the second most common ocular adnexal tumor and generally
manifests as a salmon -pink patch on the fornix of the eye

POSITIVE MED PG
Web: http://www.positivemedpg.com/ Score:
Email: manohrajan@yahoo.co.in 0.00 / 100
Page 62
NIMHANS - DM Neurology -
Exam Title :
MO...
Email : shubham.saha452@gmail.com
Contact : 9932160514
QUESTION 35. NDE3MCtTSFVCSEFNICBTQUhBK3NodWJoYW0uc2FoYTQ1MkBnbWFpbC5jb20rOTkzMjE2M
USU9OIDM0
Components of Cannabis includes all except

a) charas
b) bhang
c) Ganja
d) afeem
Correct Answer: D
Your Answer: Unanswered
Explanation

Cannabis preparations.

THE Potency (as


Cannabis
Portions of Plant content (% compared to
preparation
) ‘Bhang)

1.Hashish/ - Resinous exudate from the flowering


8 – 4% 10
Charas tops of cultivated plants

- Small leaves and brackets of


2. Ganja 1 – 2% 2
inflorescence of highly cultivated plants

- Dried leaves, flowering shoots and


3. Bhang 1% 1
cut tops of uncultivated plants

4. Hash oil - Lipid soluble plant extract 15 – 40% 25

MORE NIMHANS FACTS-NIMHANS 2007

The daily production of CSF is 550ML

Circadian rhythm is regulated by Supra chiasmatic nucleus

Vibration sense is carried by Pacinian corpuscles

A widely accepted dogma is that about 15–20% of cardiac output is received by the brain in
healthy adults under resting conditions

QUESTION 36. NDE3MCtTSFVCSEFNICBTQUhBK3NodWJoYW0uc2FoYTQ1MkBnbWFpbC5jb20rOTkzMjE2M


USU9OIDM1
Lateral traction test is done in unnatural sexual offences in:

a) Habitual active agent

POSITIVE MED PG
Web: http://www.positivemedpg.com/ Score:
Email: manohrajan@yahoo.co.in 0.00 / 100
Page 63
NIMHANS - DM Neurology -
Exam Title :
MO...
Email : shubham.saha452@gmail.com
Contact : 9932160514

b) Pedophilia active agent


c) Bestiality active agent
d) Habitual passive agent
Correct Answer: D
Your Answer: Unanswered
Explanation

Lateral traction test is done in unnatural sexual offences in habitual passive agent.
Lateral buttock traction test is a useful guide as to the patient’s habituation to anal intercourse.

MORE NIMHANS FACTS-NIMHANS 2007

HIV

Treatment guidelines recommend that anyone with a viral load greater than 100,000 copies/mL
of blood should begin treatment.

In mammals, DNA synthesis occurs in -S PHASE

Carnitine deficiency impairs Fatty oxidation

Deficiency of which mineral causes Diabetes mellitus -ZINC

Pramipexole is Dopamine agonist

QUESTION 37. NDE3MCtTSFVCSEFNICBTQUhBK3NodWJoYW0uc2FoYTQ1MkBnbWFpbC5jb20rOTkzMjE2M


USU9OIDM2
All are prodrugs EXCEPT

a) INH
b) Nevirapine
c) Metronidazole
d) Mycophenolate mofetil
Correct Answer: B
Your Answer: Unanswered
Explanation

• A prodrug is a pharmacological substance (drug) that is administered in an inactive (or


significantly less active) form. Once administered, the prodrug is metabolised in vivo into
an active metabolite. The rationale behind the use of a prodrug is generally for
absorption, distribution, metabolism, and excretion (ADME) optimization. Prodrugs are
usually designed to improve oral bioavailability,
• Prodrugs can be classified into two types based on their sites of conversion into the final
active drug form: Type I, those that are converted intracellularly (e.g., anti-viral
nucleoside analogs, lipid-lowering statins, antibody-directed/gene-directed enzyme
prodrugs [ADEP/GDEP] for chemotherapy), and Type II, those that are converted
extracellularly, especially in digestive fluids or the systemic circulation (e.g., etoposide
phosphate, valganciclovir, fosamprenavir).

Ø Enalapril is converted by esterase to the active enalaprilat


Ø Valacyclovir is converted by esterase to the active acyclovir
Ø Fosamprenavir is hydrolysed to the active amprenavir

POSITIVE MED PG
Web: http://www.positivemedpg.com/ Score:
Email: manohrajan@yahoo.co.in 0.00 / 100
Page 64
NIMHANS - DM Neurology -
Exam Title :
MO...
Email : shubham.saha452@gmail.com
Contact : 9932160514
Ø Levodopa is converted by DOPA decarboxylase to the active dopamine
Ø Chloramphenicol succinate ester is used as intravenous prodrug of chloramphenicol, because
pure chloramphenicol does not dissolve in water
Ø Psilocybin is dephosphorylated to the active psilocin
Ø Heroin is deacetylated by esterase to the active morphine
Ø Codeine is demethylated by the liver enzyme CYP2D6 to the active morphine, as well as
several other compounds that may be active in analgesia
Ø Molsidomine is metabolized into SIN-1 which decomposes into the active compound nitric
oxide
Ø Paliperidone is an atypical antipsychotic for schizophrenia. It's the active metabolite of
risperidone

Ø Mycophenolic acid (INN) (pronounced or mycophenolate******** is an


immunosuppressant drug used to prevent rejection in organ transplantation. It was initially
marketed as the prodrug mycophenolate mofetil (abbreviated MMF) to improve oral
bioavailability. More recently, the salt mycophenolate sodium has also been introduced
Ø Mycophenolate is derived from the fungus Penicillium stoloniferum. Mycophenolate mofetil is
metabolised in the liver to the active moiety mycophenolic acid. It inhibits inosine
monophosphate dehydrogenase, the enzyme that controls the rate of synthesis of guanine
monophosphate in the de novo pathway of purine synthesis used in the proliferation of B and T
lymphocytes.
Ø In general, mycophenolate is used for the prevention of organ transplant rejection.
Ø Its increasing application in treating lupus nephritis has demonstrated more frequent
complete response and less frequent complications
Ø Common adverse drug reactions (≥1% of patients) associated with mycophenolate therapy
include diarrhea, nausea, vomiting, infections, leukopenia, and/or anemia. Mycophenolate
sodium is also commonly associated with fatigue, headache, and/or cough. Intravenous (IV)
administration of mycophenolate mofetil is also commonly associated with thrombophlebitis and
thrombosis. Infrequent adverse effects (0.1–1% of patients) include esophagitis, gastritis,
gastrointestinal tract hemorrhage, and/or invasive cytomegalovirus (CMV) infection
Ø

Nevirapine******** , [not a prodrug******] i s a non-nucleoside reverse transcriptase


inhibitor (NNRTI) used to treat HIV-1 infection and AIDS.
Ø Both nucleoside and non-nucleoside RTIs inhibit the same target, the reverse transcriptase
enzyme, an essential viral enzyme which transcribes viral RNA into DNA. Unlike nucleoside
RTIs, which bind at the enzyme's active site, NNRTIs bind within a pocket termed the NNRTI
pocket.
Ø Resistance to nevirapine develops rapidly if viral replication is not completely suppressed.The
most common mutations observed after nevirapine treatment are Y181C and K103N,
Ø The most common adverse effect of nevirapine is the development of mild or moderate rash
(13%). Severe or life-threatening skin reactions have been observed in 1.5% of patients,
including Stevens-Johnson syndrome, toxic epidermal necrolysis and hypersensitivity.
Ø Nevirapine may cause severe or life-threatening liver toxicity, usually emerging in the first six

POSITIVE MED PG
Web: http://www.positivemedpg.com/ Score:
Email: manohrajan@yahoo.co.in 0.00 / 100
Page 65
NIMHANS - DM Neurology -
Exam Title :
MO...
Email : shubham.saha452@gmail.com
Contact : 9932160514
weeks of treatment.
Ø Significant lowering of nevirapine levels occurs with the anti-tuberculosis drug, rifampicin,
and the drugs should not be administered together.
Ø Nevirapine is an inducer of cytochrome P450 isoenzymes CYP3A4 and CYP2B6. It reduces the
levels of several co-administered drugs including the antiretrovirals efavirenz, indinavir,
lopinavir, nelfinavir and saquinavir, as well as clarithromycin, ketoconazole, forms of hormonal
contraception, and methadone
Ø A single dose of nevirapine given to both mother and child reduced the rate of HIV
transmission by almost 50% compared with a very short course of zidovudine (AZT) prophylaxis
Ø Half life45 hours
Metronidazole
Ø

Metronidazole is a prodrug********* . I
Ø
Metronidazole is listed by the International Agency for Research on Cancer (IARC) as
a potential human carcinogen

INHØ Isoniazid is a prodrug******* and must be activated by bacterial catalase. It is


activated by catalase-peroxidase enzyme KatG to form isonicotinic acyl anion or radical. Ø This
mechanism inhibits the synthesis of mycolic acid in the mycobacterial cell wall. Ø Isoniazid is
bactericidal to rapidly-dividing mycobacteria, but is bacteriostatic if the mycobacterium is slow-
growing. Ø Half life0.5-1.6h (fast acetylators), 2-5h (slow acetylators)Ø Adverse reactions
include rash, abnormal liver function tests, hepatitis, sideroblastic anemia, peripheral
neuropathy, mild central nervous system (CNS) effects, drug interactions resulting in increased
phenytoin (Dilantin) or disulfiram (Antabuse) levels and intractable seizures (status epilepticus).
Ø Peripheral neuropathy Ø The N-acetylhydrazine metabolite is believed to be responsible for
the hepatotoxic effects seen in patients treated with isoniazid. The rate of acetylation is
genetically determined. Approximately 50% of blacks and Caucasians are slow inactivators; the
majority of Inuit and Orientals are rapid inactivators. The half-life in fast acetylators is 1 to 2
hours while in slow acetylators it is 2 to 5 hours. Elimination is largely independent of renal
function, however the half-life may be prolonged in liver disease. The rate of acetylation has not
been shown to significantly alter the effectiveness of isoniazid. However, slow acetylation may
lead to higher blood concentrations with chronic administration of the drug, with an increased
risk of toxicity.

QUESTION 38. NDE3MCtTSFVCSEFNICBTQUhBK3NodWJoYW0uc2FoYTQ1MkBnbWFpbC5jb20rOTkzMjE2M


USU9OIDM3
All of the following statements about Phenytoin are true, Except

a) Follows saturation kinetics


b) Is teratogenic
c) Is highly protein bound

POSITIVE MED PG
Web: http://www.positivemedpg.com/ Score:
Email: manohrajan@yahoo.co.in 0.00 / 100
Page 66
NIMHANS - DM Neurology -
Exam Title :
MO...
Email : shubham.saha452@gmail.com
Contact : 9932160514

d) Stimulates Insulin secretion


Correct Answer: D
Your Answer: Unanswered
Explanation

Phenytoin can cause insulin resistsance* *****


The side effects of common antiepileptics are given below

Drug adverse effect

* Diplopia
* Ataxia
* Phenytoin -Level increased by isoniazid, sulfonamides,
fluoxetine. Half life is 24 h* * Hirsuitism

* Gumhypertrophy

* Ataxia
* Diplopia
* Carbama zepine- Level increased by erythromycin,
* Aplastic Anemia
propoxyphene, isoniazid, cimetidine, fluoxetine. Half life is 10–
17 h * Hepatotoxicity

* Hyponatremia

* Tremor, Ataxia
* Wt. Gain

* Valproic acid -. Half life is 15 hors * Hepatotoxicity

* Alopecia

* Hyperammonemia*

* Lamotrigine. -Half life is25 h * 14 h (with enzyme-inducers)


* Steven. Johnson syndrome
* 59 h (with valproic acid)

* Gabapentin -. Half life is 5-9 hours * Wt. Gain

* Wt. loss* [AIIMS


* Topiramate –also used in Lennox-Gastaut syndrome. Half life
MAY-2006***], glaucoma
is 20–30 h
* Renal stones, hypohydrosis

POSITIVE MED PG
Web: http://www.positivemedpg.com/ Score:
Email: manohrajan@yahoo.co.in 0.00 / 100
Page 67
NIMHANS - DM Neurology -
Exam Title :
MO...
Email : shubham.saha452@gmail.com
Contact : 9932160514

* Wt. loss*
* Felbamate - Increases phenytoin, valproic acid, active * Aplastic Anemia
carbamazepine. Half life is 16-20 hours
* Hepatic failure

* Zonisamide-. Half life is 50-68 hours * Renal stones, hypohydrosis

* Lacosamide- Dizziness * Ataxia

* Diplopia

* Vertigo

(PR interval prolongation)

* Half life is 13 hours

Rufinamide used in Lennox-Gastaut syndrome(AIIMS NOV-2013***) * Side effects

* Sedation

* Fatigue

* Dizziness

* Ataxia

* Headache

* Diplopia, ( QT interval prolongation)

* Rufinamide-interactions

* Level increased by valproic acid

* May increase phenytoin

* Half life is 6-10 hours

MORE NIMHANS FACTS-NIMHANS 2007

The lipid lowering drug to be avoided in diabetes as it worsens the glycemic profile NIACIN

All of the following are side effects of Hydralazine

a. Lupus syndrome

b. Rheumatoid arthritis

c. Postural hypotension

POSITIVE MED PG
Web: http://www.positivemedpg.com/ Score:
Email: manohrajan@yahoo.co.in 0.00 / 100
Page 68
NIMHANS - DM Neurology -
Exam Title :
MO...
Email : shubham.saha452@gmail.com
Contact : 9932160514

Hydralazine is a potent direct vasodilator that has antioxidant and nitric oxide–enhancing
actions.

* Hydralazine may induce a lupus-like syndrome, and side effects of minoxidil include
hypertrichosis and pericardial effusion .

QUESTION 39. NDE3MCtTSFVCSEFNICBTQUhBK3NodWJoYW0uc2FoYTQ1MkBnbWFpbC5jb20rOTkzMjE2M


USU9OIDM4
Which is incorrectely matched for adverse effects?

a) Carbamazepine- Hepatotoxicity
b) Ethosuximide- Bone marrow suppression
c) Lamotrigene – Steven Johnson syndrome
d) Gabapentin – Drug interaction
Correct Answer: D
Your Answer: Unanswered
Explanation

Drugs that are prone to cause hepatotoxicity are Carbamazepine, Valproate & Felbamate.
Blood dyscrasias are induced by Carbamazepine, Ethosuximide & Felbamate. Lamotrigine
causes fatal dermatitis. Gabapentin does not cause drug interaction .

MORE NIMHANS FACTS-NIMHANS 2007

The following cell is involved in LE cell phenomenon- Neutrophil

Firm, warty vegetations along the line of closure of valve is seen in RHD

“Programmed cell death” is known as APOPTOSIS

The toxin responsible for LAthyrism is BOAA

Lathyrism

* neurolathyrism because it affects the nervous system, and in animals as osteolathyrism

The pulse

* Lathyrus sativus is commonly known as “Khesari dhal”

* The seeds of lathyrus have a characteristic triangular shape and grey colour

* diets containing over 30% of this dhal if taken over a period of 2-6 months will result in
neurolathyrism

The toxin

* The toxin present in lathyrus seeds has been identified as Beta oxalyl amino alanine (BOAA)

The disease

POSITIVE MED PG
Web: http://www.positivemedpg.com/ Score:
Email: manohrajan@yahoo.co.in 0.00 / 100
Page 69
NIMHANS - DM Neurology -
Exam Title :
MO...
Email : shubham.saha452@gmail.com
Contact : 9932160514

* mainly young men

a) Latent stage:

b) No-stick stage:

c) One-stick stage

d) Two-stick stage

e) Crawler stage

Interventions

a) Vitamin C prophylaxis

b) Banning the crop

c) Removal of toxin

1. Steeping method

* A large quantity of water is boiled and the pulse is soaked in hot water for 2 hours

2. Parboiling

c. Genetic approach: Certain strains of lathyrus contain very low levels of toxin (0.1%)

d. Socio-economic changes

QUESTION 40. NDE3MCtTSFVCSEFNICBTQUhBK3NodWJoYW0uc2FoYTQ1MkBnbWFpbC5jb20rOTkzMjE2M


USU9OIDM5
A patient with smear proved malaria was planed for treatment with chloroquine.Chloroquine is
contraindicated in which of the following conditions

a) Pregnancy
b) Epilepsy
c) Psoariasis
d) Psychiatric disorder
Correct Answer: C
Your Answer: Unanswered
Explanation

Antimalarial Contraindication

* Psoariasis
* Chloroquine
* Porphyria

* Primaquine * Connective Tissue disorder

POSITIVE MED PG
Web: http://www.positivemedpg.com/ Score:
Email: manohrajan@yahoo.co.in 0.00 / 100
Page 70
NIMHANS - DM Neurology -
Exam Title :
MO...
Email : shubham.saha452@gmail.com
Contact : 9932160514

* Epilepsy
* Mefloquine
* Psychiatric Disorder * Pregnancy

* Pregnancy
* Quinine
* Myasthenia

Tit bits

Mefloquine

* Highly protein bound * In chloroquine resistant falciparum malaria


* Only oral

* Good CSF penetration

* t ½-13-33 days (shortened in acute malaria)

* Contraindicated in

- Epilepsy

- Psychiatric disorder

* Shouldn’t be used with Quinine, Quinidine, b blockers, Calcium channel Blockers

* Mefloquine + chloroquine

→ Seizure s

MORE NIMHANS FACTS-NIMHANS 2007

An elderly man with ataxic hemiparesis, diagnosis is CERBELLAR BLEED

MEDIAL MEDULLARY SYNDROME

Medial medullary syndrome (occlusion of vertebral artery or of branch of vertebral or lower


basilar artery) features are listed below

On side opposite
On side of lesion
lesion

Paralysis of arm and


leg, sparing face;
Paralysis with
impaired tactile and
atrophy of one-half
proprioceptive sense
half the tongue:
over one-half the body:
Ipsilateral twelfth
Contralateral
nerve
pyramidal tract and
medial lemniscus

POSITIVE MED PG
Web: http://www.positivemedpg.com/ Score:
Email: manohrajan@yahoo.co.in 0.00 / 100
Page 71
NIMHANS - DM Neurology -
Exam Title :
MO...
Email : shubham.saha452@gmail.com
Contact : 9932160514

QUESTION 41. NDE3MCtTSFVCSEFNICBTQUhBK3NodWJoYW0uc2FoYTQ1MkBnbWFpbC5jb20rOTkzMjE2M


USU9OIDQw
All are indicators of air pollution except

a) CO2
b) SO2
c) Soiling index
d) Smoke index
Correct Answer: A
Your Answer: Unanswered
Explanation

Air pollutants

* The combination of smoke and fog is called “smog”.

1. Carbon monoxide:

* Environmental concentrations tend to be expressed in terms of 8 hour average


concentrations.

2. Sulphur dioxide:

* When SO 2 combines with water, it forms sulphuric acid; this is the main component of acid
rain.

* A SO 2 concentration of 500 µg/m 3 should not be exceeded over average periods of 10


minutes duration.

* The revision of 24 hour guideline for SO 2 from 125 to 20 µg/m 3 is based on the health
effects.

3. Lead:

* An estimated 80-90 % of lead in ambient air derives from the combustion of leaded petrol.

* Children up to 6 years of age are a population at increased risk.

4. Carbon dioxide:

* Not commonly regarded as an air pollution.*******

5. Hydrocarbons:

6. Cadmium:

* Steel industry, waste incineration, volcanic acition and zinc production seem to account for
the largest emissions.

* Cigarettes may contain from 0.5 to 3 µg cadmium per gram of tobacco.

POSITIVE MED PG
Web: http://www.positivemedpg.com/ Score:
Email: manohrajan@yahoo.co.in 0.00 / 100
Page 72
NIMHANS - DM Neurology -
Exam Title :
MO...
Email : shubham.saha452@gmail.com
Contact : 9932160514

7. Hydrogen sulphide:

* Elemental sulphur or sulphur containing compounds come in contact with organic material at
high temperatures.

* The first noticeable effect of hydrogen sulphide at low concentration is its unpleasant odour
Conjunctival irritation is the next subjective symptom.

8. Ozone:

* Ozone at ground level – not to be confused with the ozone layer in the upper atmosphere

* The highest levels of ozone pollution occurs during periods of sunny weather,

* It can caused breathing problems, trigger asthma, reduce lung function and cause lung
diseases.

* The previously recommended limit, which was fixed at 120 µg/m 3 of 8-hour mean, has been
reduced to 100 µg/m 3.

9. Oxides of nitrogen:

* Nitric oxide, which comprises around 95% of nitrogen oxides from a combustion source.

* The current WHO guideline value of 40 µg/m 3 (anuual mean).

10. Polycyclic aromatic hydrocarbons (PAH):

* The examples are Benzo.

a. Pyrene (BaP), Benzanthracene, Benzo

fluoranthene, fluoranthene, Nephthalene etc.

* Produced by incomplete burning of carbon containing material like wood, garbage, coal and
oil.

* PAHs from when materials burn at low temperatures, such as in wood fires and cigarettes.

* BaP is one of the most potent carcinogens among the known PAHs.

* A unit risk for lung cancer for PAH mixture is etimated to be 8.7 x 10 -5 ng/m 3 BaP.

11. Particulate matter:

* Mass and composition tend to divide into two principal groups: coarse particles larger than
2.5 µm in aerodynamic diameter, and fine particles smaller than 2.5 µm (PM 2.5 ) in
aerodynamic diameter.

* The particulate matter of diameter smaller than 2.5 µm are more dangerous.

POSITIVE MED PG
Web: http://www.positivemedpg.com/ Score:
Email: manohrajan@yahoo.co.in 0.00 / 100
Page 73
NIMHANS - DM Neurology -
Exam Title :
MO...
Email : shubham.saha452@gmail.com
Contact : 9932160514

* The 2005 AQG set for the first time a guideline value for particular matter (PM) as 10 µg/
m 3 annual mean and 25 µg/m 3 24-hour mean for PM 2.5 and 20 µg/m 3 annual mean

and 50 µg/m 3 24 hours mean for PM 10.

Indoor air pollution

Monitoring of air pollution

* The best indicators of air pollution are sulphur dioxide, smoke and suspended particies.

a. Sulphur dioxide:

b. Smoke or soiling index:(1992-JIP***).

* A known volume of air is filtered through a white filter paper.

* Smoke concetration is estimated and expressed as micrograms/cubic metre of air.

c. Grit and dust measurement.

d. Coefficient of haze.

e. Air pollution index.

MORE NIMHANS FACTS-NIMHANS 2007

The most common malignancy in childhood is ALL

A female child of a mother is diagnosed to have Duchenne’s muscular dystrophy. What


risk does the mother have for her next male child to suffer from similar illness -50%

The most common cause of perinatal mortality is PREMATURITY

The most common pancreatic tumor is ADENOCARCINOMA

Skin lesion characteristic of Lyme’s disease includes ERYTHEMA CHRONICUM MIGRANS

QUESTION 42. NDE3MCtTSFVCSEFNICBTQUhBK3NodWJoYW0uc2FoYTQ1MkBnbWFpbC5jb20rOTkzMjE2M


USU9OIDQx
Prevention of first attack of rheumatic activity by early detection & adequate treatment of all
cases of streptococcal pharyngitis is

a) Primordial prevention
b) Primary prevention
c) Secondary prevention
d) Tertiary prevention
Correct Answer: C
Your Answer: Unanswered
Explanation

POSITIVE MED PG
Web: http://www.positivemedpg.com/ Score:
Email: manohrajan@yahoo.co.in 0.00 / 100
Page 74
NIMHANS - DM Neurology -
Exam Title :
MO...
Email : shubham.saha452@gmail.com
Contact : 9932160514

(P-31, 37- Park)

Prevention

Levels

-Primary

-Secondary

-Teritiary

Prevention

Levels Features

Primary * Health Promotion * Specific Protection

Secondary * Early Diagnosis and Trt

Teritiary * Disability limitation * Rehabilitation

Primordial Prevention

New concept Prevention in its purest sense

Prevention of devolpment of risk factors

eg – Life style modification (obesity)

Mass education

Primary

Action taken prior to onset of disease Intervention in prepathogenesis phase Types

-Population (mass) strategy

-High risk strategy

eg – improving standard of living to prevent cholera

Secondary

POSITIVE MED PG
Web: http://www.positivemedpg.com/ Score:
Email: manohrajan@yahoo.co.in 0.00 / 100
Page 75
NIMHANS - DM Neurology -
Exam Title :
MO...
Email : shubham.saha452@gmail.com
Contact : 9932160514

Action which halts the progress of a disease at its incipient stage and prevents complication
** eg – screening tests and adequate Trt.

Health Promotion *Health education

*Life style modification

*Food fortification

Specific Protection

*Immunisation

*Chemoprophylaxis

*Avoidance of allergy

Any loss or abnormality of psychological physiological or anatomical structure or


Impairment
function (eg :Loss of foot)

Disability Inability to carry out activities because of impairment

As a result of disability, the person experiences certain disadvantages (eg –


Handicap
Unemployment)

MORE NIMHANS FACTS-NIMHANS 2007

The most common site of Berry Aneurysm is Anterior circulation

The most important characteristic feature of breast malignancy in young age is PALPABLE
LUMP

QUESTION 43. NDE3MCtTSFVCSEFNICBTQUhBK3NodWJoYW0uc2FoYTQ1MkBnbWFpbC5jb20rOTkzMjE2M


USU9OIDQy
Which among the food toxicants is incorrectly matched

a) Aflatoxins – mycotoxins (storage fungus) produced by Aspergillus flavus, Aspergillus


parasiticus and are hepatotoxins
b) Ergot – this is due to field fungus – claviceps fusiformis and Easily removed by floating
them in salt water
c) Endemic ascites – over all mortality – 80%, causes ascites and jaundice, contamination of
weed seeds of Crotalaria (Jhunjhunia plants) – contain pyrrolizidine alkaloids –
hepatotoxins.
d) Epidemic dropsy – contamination of mustard oil with argemone oil -Sangunarine is the
toxic principle in argemone oil
Correct Answer: C

POSITIVE MED PG
Web: http://www.positivemedpg.com/ Score:
Email: manohrajan@yahoo.co.in 0.00 / 100
Page 76
NIMHANS - DM Neurology -
Exam Title :
MO...
Email : shubham.saha452@gmail.com
Contact : 9932160514
Your Answer: Unanswered
Explanation

Endemic ascites – over all mortality – 40%, causes ascites and jaundice******

MORE NIMHANS FACTS

FOOD TOXICANTS

Aflatoxins – mycotoxins (storage fungus) produced by Aspergillus flavus, Aspergillus


parasiticus

These fungi infest food grains like – ground nut, maize, wheat, rice etc.,

Aflatoxins – B1 and G1 – hepatotoxins

More prevalent in Rajasthan and Gujarat

Ergot – this is due to field fungus – claviceps fusiformis

Severe vasoconstriction of capillaries resulting in peripheral gangrene, rarely fatal

Easily removed by floating them in salt water

Endemic ascites – over all mortality – 40%, causes ascites and jaundice******, contaminati
on of weed seeds of Crotalaria (Jhunjhunia plants) – contain pyrrolizidine alkaloids –
hepatotoxins. Prevalent in Madhya pradesh

Epidemic dropsy – contamination of mustard oil with argemone oil

Sangunarine is the toxic principle in argemone oil

Interfers with oxidation of pyruvic acid which accumulates in the blood

Sudden, non inflammatory swelling of legs, with diarrhea, dysnoea, cardiac failure

Tests for the detection of argemone oil – nitric acid test, paper chromatography test (most
sensitive test), detects argemone oil up to 0.0001%

MORE NIMHANS FACTS-NIMHANS 2007

Rheumatoid arthritis commonly affects Cervical spine

Tumor that develops from metaphysis includes Osteosarcoma

EWINGS SARCOMA

Ewing’s Sarcoma(AIIMS-NOV-2010***)

* These tumors form 10-15% of bone sarcomas. Seen mostly in adolescence age group, they
arise from the diaphysis, ( AI-07***) [AIIMS MAY-2014***] and have affinity for flat
bones.They show an Onion peel periosteal reaction with a generous soft tissue mass. On Histo
logy monotonous, small round blue cells are seen [AIIMS MAY-2012***]. D/D include
lymphoma, Embryonal habdomyosarcoma, small cell carcinoma.

* Cytogenic abnormality are Reciprocal Translocation of long arms of 11, chromosome 22.
The surface marker is P 30/32 the product of the myc-2[AIIMS MAY-2012***], gene. A
variant of PNET -peripheral neuroepithelioma is Askin's tumor (chest wall), and

POSITIVE MED PG
Web: http://www.positivemedpg.com/ Score:
Email: manohrajan@yahoo.co.in 0.00 / 100
Page 77
NIMHANS - DM Neurology -
Exam Title :
MO...
Email : shubham.saha452@gmail.com
Contact : 9932160514
esthesioneuroblastoma [AI-2012***]. Glycogen-filled cytoplasm detected by staining with
periodic acid–Schiff is characteristic.

* It is very aggressive and considered a systemic disease. Main stay of therapy is


chemotherapy. The Drugs used are Doxorubicin, cyclophosphamide, vincristine,
Dactinomycin, etoposide.

* Lesions below elbow and below mid calf have a 5 yr survival rate 80%. Ewing’s is a
curable tumor even with metastasis in children < 11 yrs.Metastasis usually occurs to the lung,
bone, bone marrow.

The benign and malignant bone lesions have been listed below

Benign Malignant

Cartilage
Common
* Enchondroma
* Multiple myeloma
* Osteochondroma
* Osteosarcoma
* Chondroblastoma
* Chondrosarcoma
* Chondromyxoid
Fibroma * Ewing’s sarcoma
Bone * Malignant fibrous
histiocytoma
* Osteoid Osteoma
Rare
* Osteoblastoma
* Chordoma
Fibrous Tissue
(Notochordal)
* Fibroma
Unknown origin
* Desmoplastic
* Malignant Giant cell
Fibroma
tumor
Vascular
* Adamantinoma
Hemangioma
Vascular
Unknown
Hemangioendothelioma
Giant cell Tumor

QUESTION 44. NDE3MCtTSFVCSEFNICBTQUhBK3NodWJoYW0uc2FoYTQ1MkBnbWFpbC5jb20rOTkzMjE2M


USU9OIDQz
The factors that disturb the Hardy Weinburg law of equilibrium is all except

a) Immigration

POSITIVE MED PG
Web: http://www.positivemedpg.com/ Score:
Email: manohrajan@yahoo.co.in 0.00 / 100
Page 78
NIMHANS - DM Neurology -
Exam Title :
MO...
Email : shubham.saha452@gmail.com
Contact : 9932160514

b) Gene drift
c) Random mating
d) Natural selection
Correct Answer: C
Your Answer: Unanswered
Explanation

Population genetics

• The study of the precise genetic composition of population.


• Hardy is England and Weinberg in Germany.
• The Hardy-Weinberg law (AIIMS NOV-2011***) states that “the relative frequencies of
each gene allele tends of remain constant from generation to generation”.

Factors which influence the gene frequencies

• The Hardy-Weinberg law assumes that human population is static.


• There are several factors which influence the human gene pool.

(a) Mutation:

• The heterozygotes of sickle cell trait were found to be resistant to falciparum malaria.
• Mutation rates. It is said that each gene has its own characteristic mutation rate which is
estimated anywhere from 104 to 106 per generation.

(b) Natural selection:

(c) Population movements:

(d) Breeding structure

• It practice, however, matings tends to occur selectively.


• This type of mating is called “assortative mating”.

(e) Public health measures.

MORE NIMHANS FACTS-NIMHANS 2007

‘Facial palsy’ in Herpes Zoster infection is seen in Ramsay Hunt syndrome

A pregnant lady of 10 wks, with her sister having a Down’s syndrome baby, wanted to
terminate her pregnancy if her foetus is also having Down’s syndrome. What
investigation is needed- CHORIONCI VILLI SAMPLING

A 24 year old lady with 24 hrs comes with complaints of pain in the right iliac fossa, vomitting,
central dyspareunia, rebound tenderness and with a temperature of 37°C. Pregnancy test
negative. The probable diagnosis is . Acute PID

Scale for sexual maturity is TANNERS STAGING

QUESTION 45. NDE3MCtTSFVCSEFNICBTQUhBK3NodWJoYW0uc2FoYTQ1MkBnbWFpbC5jb20rOTkzMjE2M


USU9OIDQ0
A young boy presents with slow athetoid movement with giveaway weakness. The diagnosis is

POSITIVE MED PG
Web: http://www.positivemedpg.com/ Score:
Email: manohrajan@yahoo.co.in 0.00 / 100
Page 79
NIMHANS - DM Neurology -
Exam Title :
MO...
Email : shubham.saha452@gmail.com
Contact : 9932160514

a) Dystonia musculorum
b) Niemann –Pick type A
c) Cerebral Palsy
d) Lesch – Nyhan syndrome
Correct Answer: A
Your Answer: Unanswered
Explanation

A condition characterized by focal DYSTONIA that progresses to involuntary spasmodic


contractions of the muscles of the legs, trunk, arms, and face. The hands are often spared,
however, sustained axial and limb contractions may lead to a state where the body is grossly
contorted. Onset is usually in the first or second decade. Familial patterns of inheritance,
primarily autosomal dominant with incomplete penetrance, have been identified.

QUESTION 46. NDE3MCtTSFVCSEFNICBTQUhBK3NodWJoYW0uc2FoYTQ1MkBnbWFpbC5jb20rOTkzMjE2M


USU9OIDQ1
All of these are critical components of an FRU except:

a) Surgical intervention availability


b) Newborn care
c) Blood storage facility
d) Laporoscopic services
Correct Answer: D
Your Answer: Unanswered
Explanation

• First referral unit


• Emergency obseteric care
• 1 FRU caters to?

3 Essential elements

• Availability of surgical intereventions


• New born care
• Blood storage facility -24 hrs
• Minimum bed strength- 20-30
• MORE NIMHANS FACTS-NIMHANS 2008
• following acts as a vector of West Nile fever-CULEX MOSQUITO
• A patient aged 50 years, with Down’s Syndrome , presents with difficulty in speech, what
is the diagnosis- Senile dementia Alzhemeimer’s type

• Lesh – Nyhan syndrome

a. Hyperuricemia & Mental retardation

b. Self – Mutilative behaviour

c. Choreoathetosis & Spasticity

d. COMPLETE deficiency of HPRT


POSITIVE MED PG
Web: http://www.positivemedpg.com/ Score:
Email: manohrajan@yahoo.co.in 0.00 / 100
Page 80
NIMHANS - DM Neurology -
Exam Title :
MO...
Email : shubham.saha452@gmail.com
Contact : 9932160514

QUESTION 47. NDE3MCtTSFVCSEFNICBTQUhBK3NodWJoYW0uc2FoYTQ1MkBnbWFpbC5jb20rOTkzMjE2M


USU9OIDQ2
Keratoconus is best treatedwith

a) Contact lens
b) Spherical lens
c) Convex lens
d) Cylindrical lens
Correct Answer: A
Your Answer: Unanswered
Explanation

KERATOCONUS:

1.Munson’s sign : It is a V-shaped conformation ofthe lower lid

produced by the ecstatic cornea in downgaze.

2. Rizutti’s sign . -Lateral illumination of the cornea produces a steeply focused beam of light
near the limbus.

3. Vogt’s striae - Posterior stress lines. These are vertical lines in the deep stroma and
descemet s membrane that parallel the axis of the cornea.

4. Fleischer’s ring -Iron deposits within epithelial layers. The iron

comes from tears and gets accumulated at the junction of normal and

steep cornea or the base of the cone.

5.Epithelial or subepithelial scarring

6.Scissoring of the light reflex on retinoscopy

7.Oil droplet sign of Charleux

Keratometry findings:

Distortion of the mire lines .

A documented increase in corneal curvature over time is a

sensitiveindicator of keratoconus .

A corneal topogram of an eye affected by keratoconus .Blue shows the flattest areas, and red
the steepest.

POSITIVE MED PG
Web: http://www.positivemedpg.com/ Score:
Email: manohrajan@yahoo.co.in 0.00 / 100
Page 81
NIMHANS - DM Neurology -
Exam Title :
MO...
Email : shubham.saha452@gmail.com
Contact : 9932160514

MORE NIMHANS FACTS-NIMHANS 2008

Patient had circumduction and loss of ankle jerk might have injury of SCIATIC NERVE

Petechiae, ecchymosis, joint effusion are all symptoms due to the deficiency of VITAMI
NC

The various nutrients, the associated presentation in deficiency states and the dietary
requirements are summarised below

Dietary Level
Nutrient Clinical Finding Associated with
Deficiency

Thiamine Beriberi: neuropathy, muscle weakness and wasting, <0.3 mg/1000 kcal
cardiomegaly, edema, ophthalmoplegia, confabulation

Riboflavin Magenta tongue, angular stomatitis, seborrhea, cheilosis <0.6 mg

Niacin Pellagra: pigmented rash of sun-exposed areas, bright <9.0 niacin equivalents
red tongue, diarrhea, apathy, memory loss,
disorientation

Vitamin B 6 Seborrhea, glossitis convulsions, neuropathy, depression, <0.2 mg


confusion, microcytic anemia

Folate Megaloblastic anemia, atrophic glossitis, depression, <100 microg/d


homocysteine

Vitamin B 12 Megaloblastic anemia, loss of vibratory and position <1.0 microg/d


sense, abnormal gait, dementia, impotence, loss of
bladder and bowel control and increasd homocysteine,
methylmalonic acid levels

POSITIVE MED PG
Web: http://www.positivemedpg.com/ Score:
Email: manohrajan@yahoo.co.in 0.00 / 100
Page 82
NIMHANS - DM Neurology -
Exam Title :
MO...
Email : shubham.saha452@gmail.com
Contact : 9932160514

Vitamin C Scurvy: petechiae, ecchymosis, coiled hairs, inflamed <10 mg/d


and bleeding gums, joint effusion, poor wound healing,
fatigue

Vitamin A Xerophthalmia, nightblindness, Bitot's spots, follicular <300 microg/d


hyperkeratosis, impaired embryonic development,
immune dysfunction

Vitamin D Rickets: skeletal deformation, rachitic rosary, bowed <2.0 microg/d


legs; osteomalacia

Vitamin E Peripheral neuropathy, spinocerebellar ataxia, skeletal Not described unless


muscle atrophy, retinopathy [AI-2005***] underlying
contributing factor is
present

QUESTION 48. NDE3MCtTSFVCSEFNICBTQUhBK3NodWJoYW0uc2FoYTQ1MkBnbWFpbC5jb20rOTkzMjE2M


USU9OIDQ3
The laser procedure, most often used for treating iris neovascularisation is

a) Goniophotocoagulation
b) Laser trabeculoplasty
c) Panretinal photocoagulation
d) Laser iridoplasty
Correct Answer: C
Your Answer: Unanswered
Explanation

Neovascularization, a pathological change characterized by an uncontrolled growth of vascular


tissue, could occur in various parts of eye, including the cornea, iris, retina, and choroid. The
consequences of neovascularization within these delicate ocular tissues are fibrosis, exudation,
and/or hemorrhage that are responsible for vision loss in many common eye diseases.
The current treatment for many forms of ocular neovascularization involves photocoagulation or
cryotherapy. Pan-retinal or focalphotocoagulation is current standard therapy for diabetic
retinopathy.

MORE NIMHANS FACTS-NIMHANS 2008

Which vitamin excess causes peripheral neuropathy- B 6

A 5 year old child presents with mental retardation seizures, with ligher skin complexion. The
diagnosis is PHENYLKETONURIA

Breast feeding jaundice

a. Pregnanediol may be a factor

b. Free fatty acids can be a cause

POSITIVE MED PG
Web: http://www.positivemedpg.com/ Score:
Email: manohrajan@yahoo.co.in 0.00 / 100
Page 83
NIMHANS - DM Neurology -
Exam Title :
MO...
Email : shubham.saha452@gmail.com
Contact : 9932160514

c. Temporary interruption of bilirubin conjugation

d.develops after the first 4-7 days of life

QUESTION 49. NDE3MCtTSFVCSEFNICBTQUhBK3NodWJoYW0uc2FoYTQ1MkBnbWFpbC5jb20rOTkzMjE2M


USU9OIDQ4
In the grading of trachoma, trachomatous inflammation – follicular is defined as the presence of

a) 5 or more follicles in the lower tarsal conjunctiva


b) 3 or more follicles in the lower tarsal conjunctiva
c) 5 or more follicles in the upper tarsal conjunctiva
d) 3 or more follicles in the upper tarsal conjunctiva
Correct Answer: C
Your Answer: Unanswered
Explanation

Trachoma control programme started in 1968

POSITIVE MED PG
Web: http://www.positivemedpg.com/ Score:
Email: manohrajan@yahoo.co.in 0.00 / 100
Page 84
NIMHANS - DM Neurology -
Exam Title :
MO...
Email : shubham.saha452@gmail.com
Contact : 9932160514

MORE NIMHANS FACTS

TRACHOMA

* a chronic infectious disease of the conjunctiva and cornea, caused by Chlamydia trachomatis

Diagnosis

At least 2 of the following diagnostic criteria

a. follicles on the upper tarsal conjunctiva

POSITIVE MED PG
Web: http://www.positivemedpg.com/ Score:
Email: manohrajan@yahoo.co.in 0.00 / 100
Page 85
NIMHANS - DM Neurology -
Exam Title :
MO...
Email : shubham.saha452@gmail.com
Contact : 9932160514

b. limbal follicles or their sequelae, Herbert’s pits

c. typical conjunctival scarring (trichiasis, entropion)

d. vascular pannus, most marked at the superior limbus

* responsible for 0.2 % of visual impairment and blindness in India

Agent factors

AGENT

* C. trachomatis of immune types A,B or C.

* The sexually-transmitted C.trachomatis (serotypes D,E,F,G,H,I,J or K) may be infect, causing


an eye disease

* Morax-Axenfeld diplobacillus is the most innocuos; the Koch-Weeks bacillus is the most
widespread, and the gonococcus the most dangerous

COMMUNICABILITY:

* Trachoma is a disease of low infectivity****

Host factors

AGE

* Two to five years

SEX

* Prevalence equal in younger age groups

* Older age groups, females

Environmental factors

* April-May

Mode of tranmission

* Eye-to-eye transmission

* Direct or indirect contact

Incubation period

* 5 to 12 days

CONTROL OF TRACHOMA

1. Assessment of the problem

* “Blinding trachoma” – as indicated by the presence of

(a) Corneal blindness

POSITIVE MED PG
Web: http://www.positivemedpg.com/ Score:
Email: manohrajan@yahoo.co.in 0.00 / 100
Page 86
NIMHANS - DM Neurology -
Exam Title :
MO...
Email : shubham.saha452@gmail.com
Contact : 9932160514

(b) Trachomatous trichiasis and entropion, and

(c) Moderate and severe trachomatous inflammation

2. Chemotherapy

* The antibiotic of choice is 1% ophthalmic ointment or oily suspension of tetracyclines

(a) Mass treatment

* A prevalence of more than 5%****** severe and moderate trachoma in children under 10
years

* The treatment consists of the application twice daily of tetracycline 1% ointment to all
children, for 5 consecutive days each month or once daily for 10 days each month for 6
consecutive months, or for 60 consecutive days

Selective treatment

* The trachoma control programme in India was launched in 1963

* “Health for All by 2000” had set a target of reducing the prevalence of blindness to 0.3%

World Health Organization's Global Campaign

* S-A-F-E strategy :

* Surgery for deformed eyelids

* Periodic mass treatment with azithromycin

* Face washing

* E nvironmental improvements

MORE NIMHANS FACTS-NIMHANS 2008

A 30 year old male, laborer, chronic smoker, alcoholic presents with acute ches pain,
severe back pain, radiation to interscapular area. The diagnosis is Ruptured aortic
aneurysm

Anisocoria that increases in dim light indicates a sympathetic paresis of the iris dilator
muscle. Anisocoria that increases in bright light suggests a parasympathetic palsy.

Mucormycosis causes Invasive sinusitis

QUESTION 50. NDE3MCtTSFVCSEFNICBTQUhBK3NodWJoYW0uc2FoYTQ1MkBnbWFpbC5jb20rOTkzMjE2M


USU9OIDQ5
Actions of Inferior rectus are all of the following except

a) Adduction
b) Abduction
c) Depression

POSITIVE MED PG
Web: http://www.positivemedpg.com/ Score:
Email: manohrajan@yahoo.co.in 0.00 / 100
Page 87
NIMHANS - DM Neurology -
Exam Title :
MO...
Email : shubham.saha452@gmail.com
Contact : 9932160514

d) Extortion
Correct Answer: B
Your Answer: Unanswered
Explanation

POSITIVE MED PG
Web: http://www.positivemedpg.com/ Score:
Email: manohrajan@yahoo.co.in 0.00 / 100
Page 88
NIMHANS - DM Neurology -
Exam Title :
MO...
Email : shubham.saha452@gmail.com
Contact : 9932160514

POSITIVE MED PG
Web: http://www.positivemedpg.com/ Score:
Email: manohrajan@yahoo.co.in 0.00 / 100
Page 89
NIMHANS - DM Neurology -
Exam Title :
MO...
Email : shubham.saha452@gmail.com
Contact : 9932160514

MORE NIMHANS FACTS

A lesion of the oculomotor nucleus in the rostral midbrain , causes bilateral ptosis because the
levator muscle is innervated by a single central subnucleus. There is also weakness of the
contralateral superior rectus, because it is supplied by the oculomotor nucleus on the other side
[AIIMS MAY 2014***].

The syndromes associated with 3 rd nerve are summarised below

Syndrome Feature

* Site – supr. cerebellar


peduncle
1. Nothnagel’s
syndrome * Effect – Ipsilat 3 rd N
Palsy, contra lateral
cerebellar Ataxia

POSITIVE MED PG
Web: http://www.positivemedpg.com/ Score:
Email: manohrajan@yahoo.co.in 0.00 / 100
Page 90
NIMHANS - DM Neurology -
Exam Title :
MO...
Email : shubham.saha452@gmail.com
Contact : 9932160514

* Site - Red nucleus

* Effect – Ipsilateral 3 rd
2. Benedict’s N palsy
(AI2007)
+

Controlateral chorea,
athetosis

* Site – Supr. cerebellar


peduncle + Red Nucleus
3. Claudes
* Effect – 1 + 2

* Site – Cerebral peduncle

4. Weber’s * Effect – Ipsilat – 3 rd N.


Palsy +

Contralateral hemiparesis

Syndromes associated with 6th nerve are listed below

Syndrome Effect

Site- Dorsal Pons

Effect

- Ipsilateral lat. Gazepalsy


1. Foville (6 th )

- Ipsilat 7 th N Palsy

- Contralat Lateral
Hemiparesis

POSITIVE MED PG
Web: http://www.positivemedpg.com/ Score:
Email: manohrajan@yahoo.co.in 0.00 / 100
Page 91
NIMHANS - DM Neurology -
Exam Title :
MO...
Email : shubham.saha452@gmail.com
Contact : 9932160514

Site- Ventral PONS

Effect- Similar to

1. Except eye findings


2. Milard Gubler - LR weakness only instead
of gaze palsy

(Because Abducen fascicles


are involved, not the
nucleus)

Site- Petrous Apex


(Mastoiditis)

Effect
3. Gradenigo
- Deafness

- Pain

- Ipsilat 6th N Palsy

MORE NIMHANS FACTS-NIMHANS 2009

Reverse genetics is a method that is used to help understand the function of a gene by
analyzing the phenotypic effects of specific engineered gene sequences .

Reverse genetics usually proceeds in the opposite direction of so-called forward genetic screens
of classical genetics . In other words, while forward genetics seeks to find the genetic basis of a
phenotype or trait, reverse genetics seeks to find what phenotypes arise as a result of particular
genetic sequences.

Serotonin is a derivative of TRYPTOPHAN

The Rate limiting Enzyme for heme Synthesis is . Delta ALA synthase

QUESTION 51. NDE3MCtTSFVCSEFNICBTQUhBK3NodWJoYW0uc2FoYTQ1MkBnbWFpbC5jb20rOTkzMjE2M


USU9OIDUw
A patient was on treatment with anticoagulants as he underwent valve replacement.He
presented in the emergency room with history of profuse Epistaxsis. His prescription
revealed dabigartan intake.Which of the following can be used for reversing the
bleeding

a) Idarucizumab
b) Andexanet alfa
c) ciraparantag
d) protamine sulfate
Correct Answer: A

POSITIVE MED PG
Web: http://www.positivemedpg.com/ Score:
Email: manohrajan@yahoo.co.in 0.00 / 100
Page 92
NIMHANS - DM Neurology -
Exam Title :
MO...
Email : shubham.saha452@gmail.com
Contact : 9932160514
Your Answer: Unanswered
Explanation

*Anticoagulant reversal should be considered in patients with life-threatening bleeding, such as


intracranial bleeding, if bleeding continues despite supportive measures or if patients require
urgent surgery.

*Idarucizumab is a specific reversal agent for dabigatran.*****

* A monoclonal antibody fragment, idarucizumab binds dabigatran with high affinity to form a
1:1 complex that is then cleared by the kidneys.

*Idarucizumab is given as an intravenous bolus of 5 g. no dosage adjustment required for renal


failure

*Andexanet alfa and ciraparantag are under development for reversal of rivaroxaban, apixaban,
and edoxaban,but not approved

PREGNANCY

As small molecules, the direct oral anticoagulants can all pass through the placenta.
Consequently, these agents are contraindicated in pregnancy, and when used by women of
childbearing potential, appropriate contraception is important. The direct oral anticoagulants
should be avoided in nursing mothers and their safety in children has yet to be established.

MORE NIMHANS FACTS - [Nim DM June 2010]

NEUROLOGICAL TERMS

* HYPERASTHESIA or hypoesthesia refers to a reduction of cutaneous sensation to a specific


type of testing such as pressure, light touch, and warm or cold stimuli; anesthesia, to a
complete absence of skin sensation to the same stimuli plus pinprick;and hypalgesia or analgesi
a, to reduced or absent pain perception (nociception). * Hyperesthesia means pain or increased
sensitivity in response to touch.

* ALLODYNIA describes the situation in which a nonpainful stimulus, once perceived, is


experienced as painful, even excruciating. An example is elicitation of a painful sensation by
application of a vibrating tuning fork.

* HYPERALGESIA denotes severe pain in response to a mildly noxious stimulus, and hyperpat
hia , a broad term, encompasses all the phenomena described by hyperesthesia, allodynia,
and hyperalgesia.

*Following injury and resultant sensitization, normally innocuous stimuli can produce pain
(termed allodynia******[AIIMS-2006] ).

* HYPERPATHIA, a greatly exaggerated pain response to innocuous or mild nociceptive


stimuli, especially when applied repeatedly, is also characteristic of neuropathic pain

QUESTION 52. NDE3MCtTSFVCSEFNICBTQUhBK3NodWJoYW0uc2FoYTQ1MkBnbWFpbC5jb20rOTkzMjE2M


USU9OIDUx
Perilymph communicates with subarachnoid space through which of the following
structure

POSITIVE MED PG
Web: http://www.positivemedpg.com/ Score:
Email: manohrajan@yahoo.co.in 0.00 / 100
Page 93
NIMHANS - DM Neurology -
Exam Title :
MO...
Email : shubham.saha452@gmail.com
Contact : 9932160514

a) Ductus reuniens
b) Helicotrema
c) Utriculosaccular duct
d) Cochlear aqueduct
Correct Answer: D
Your Answer: Unanswered
Explanation

Explanation

The cochlear aqueduct provides an important communication between the subarachnoid and pe
rilymphatic space . The only outlet from increased intracochlear pressure is via a tear of the
oval or round window membranes.

MORE NIMHANS FACTS

Development and growth of paranasal sinuses

POSITIVE MED PG
Web: http://www.positivemedpg.com/ Score:
Email: manohrajan@yahoo.co.in 0.00 / 100
Page 94
NIMHANS - DM Neurology -
Exam Title :
MO...
Email : shubham.saha452@gmail.com
Contact : 9932160514

First
Status of birth Growth radiologic
evidence

* Rapid growth from birth to 3 years and from 4-5 months


Present of birth
Maxillary 7-12 years . Adult size – 15 years after birth

Present of birth
Anterior group:
* 5X2X2 mm
1 year
Ethmoid Reach adult size by 12 years
Posterior group:
5X4X2 mm

Invades frontal bone at the age of 4 years Size


* Frontal Not present 6 years
increases until teens

Reaches sella turcica by the age of 7 years,


* dorsum sellae by late teens and basisphenoid by
Not present 4 years
Sphenoid adult age Reaches full size between 15 years to
adult age

MKT 21,25

Degree of hearing loss

1. Mild 26-40 dB

2. Moderate 41-55 dB

3. Moderately severe 56-70 dB

4. Severe 71-91 dB

5. Profound More than 91 dB

6. Total

Showing results of various tests to differentiate a cochlear from a retrocochlear lesion

POSITIVE MED PG
Web: http://www.positivemedpg.com/ Score:
Email: manohrajan@yahoo.co.in 0.00 / 100
Page 95
NIMHANS - DM Neurology -
Exam Title :
MO...
Email : shubham.saha452@gmail.com
Contact : 9932160514

Retrocochlear
Normal Cochlear lesion
lesion

* Pure tone Sensorineural hearing Sensorineural


Normal
audiogram loss hearing loss

* Speech
90-100% Below 90% Very poor
discrimination score

* Roll over
Absent Present Absent
phenomenon

* Recruitment Absent Present Absent

* SISI score 0 – 15% Over 70% 0 – 20%

* Threshold tone
0 – 15 dB Less than 25dB Above 25dB
decay test

* Stapedial reflex Present Present Absent

* Stapedial reflex
Normal Normal Abnormal
decay

Normal interval Normal interval between Wave V delayed or


* BE.R.A
between wave I & V wave I & V absent

Differences between inner and outer hair cells

Inner hair cells Outer hair cells

Total No 3500 12,000

Rows One row Three or four rows

Shape Flask-shaped Cylindrical

POSITIVE MED PG
Web: http://www.positivemedpg.com/ Score:
Email: manohrajan@yahoo.co.in 0.00 / 100
Page 96
NIMHANS - DM Neurology -
Exam Title :
MO...
Email : shubham.saha452@gmail.com
Contact : 9932160514

Primarily afferent fibres and very Mainly efferent fibres and very few
Nerve supply
few efferent afferent

Development Develop earlier Develop late

Function Transmit auditory stimuli Modulate function of inner hair cells

Vulner Easily damaged by ototoxic drugs and


More resistant
ability high intensity noise

MORE NIMHANS FACTS- [Nim June Neurosurg 2010]

All of the following statements are true except [Nim June Neurosurg 2010]

a. 60 % body weight in male is water

b. Lean person has more water than obese person

c. Most of the body water is interstitial

d. 50% of body weight is protein and fat

Ans (C)

About two-thirds of total body water is intracellular and one-third is extracellular.


Approximately one-fourth of the ECF is in the plasma and the remainder comprises the
interstitial fluid******

MORE NIMHANS FACTS

*Renin is an enzyme with a molecular mass of

about 40,000 Da that acts on its substrate, angiotensinogen, an α 2 -globulin synthesized by the
liver, to release angiotensin I, a decapeptide, which in turn is converted to angiotensin II (AII),
an octapeptide.

*AII has generalized vasoconstrictor properties, particularly on the renal e ff erent arterioles.

DRUGS ASSOCIATED WITH EDEMA

Nonsteroidal anti-inflammatory drugs

Antihypertensive agents

Direct arterial/arteriolar vasodilators

Hydralazine

Clonidine

Methyldopa

POSITIVE MED PG
Web: http://www.positivemedpg.com/ Score:
Email: manohrajan@yahoo.co.in 0.00 / 100
Page 97
NIMHANS - DM Neurology -
Exam Title :
MO...
Email : shubham.saha452@gmail.com
Contact : 9932160514

Guanethidine

Minoxidil

Calcium channel antagonists

α-Adrenergic antagonists

Thiazolidinediones

Steroid hormones

Glucocorticoids

Anabolic steroids

Estrogens

Progestins

Cyclosporine

Growth hormone

Immunotherapies

Interleukin 2

QUESTION 53. NDE3MCtTSFVCSEFNICBTQUhBK3NodWJoYW0uc2FoYTQ1MkBnbWFpbC5jb20rOTkzMjE2M


USU9OIDUy
Which of the following bears the maximum pressure during mastication?

a) Pterygomaxillary Buttress
b) Zygomaxillary Buttress
c) Nasomaxillary Buttress
d) Buccomaxillary Buttress
Correct Answer: A
Your Answer: Unanswered
Explanation

Explanation

The vertical buttresses consist of the paired nasomaxillary (NM), zygomaticomaxillary


(ZM) and pterygomaxillary (PM) midfacial buttresses as well as the ramus of the
mandible. These buttresses define the vertical height of the face and provide the bony
support required for mastication.

MORE NIMHANS FACTS

Fracture Maxilla- Le Fort Fracture

POSITIVE MED PG
Web: http://www.positivemedpg.com/ Score:
Email: manohrajan@yahoo.co.in 0.00 / 100
Page 98
NIMHANS - DM Neurology -
Exam Title :
MO...
Email : shubham.saha452@gmail.com
Contact : 9932160514

I guerin Lower part of maxilla

II pyramidal Common * Severe Trauma

III * Craniofacial dysfunction

CSF rhinorhea

– Commonest cribriform plate fracture*****

Maxilla fracture

* Malocclusion

* Mobility of maxilla

* Elongation of mid face

* CSF rhinorrhoea – cribriform plate involved *****

Le Fort fractures - types of facial fractures involving the maxillary bone and surrounding
structures in a usually bilateral and either horizontal, pyramidal or transverse way. LeFort
fractures are classic in facial trauma

To qualify for LeFort Fractures the pterygoid plates must be involved. These are seen posterior
to the maxillary sinuses on axial CT and inferior to the orbital rim on coronal slices

Le Fort I fractures (horizontal)

POSITIVE MED PG
Web: http://www.positivemedpg.com/ Score:
Email: manohrajan@yahoo.co.in 0.00 / 100
Page 99
NIMHANS - DM Neurology -
Exam Title :
MO...
Email : shubham.saha452@gmail.com
Contact : 9932160514

may result from a force of injury directed low on the maxillary alveolar rim in a downward
direction. It is also known as a Guerin fracture or 'floating palate', and usually involves the
inferior nasal aperture.

The fracture extends from the nasal septum to the lateral pyriform rims, travels horizontally
above the teeth apices, crosses below the zygomaticomaxillary junction, and traverses the
pterygomaxillary junction

Guerin's sign is present characterised by ecchymosis in the region of greater palatine


vessels.

Le Fort II fractures (pyramidal)

may result from a blow to the lower or mid maxilla and usually involve the inferior orbital rim.
Such a fracture has a pyramidal shape and extends from the nasal bridge at or below the
nasofrontal suture through the frontal processes of the maxilla, inferolaterally through the
lacrimal bones and inferior orbital floor and rim through or near the inferior orbital foramen

Le Fort III fractures (transverse)

are otherwise known as craniofacial dissociation and involve the zygomatic arch. These may
follow impact to the nasal bridge or upper maxilla. These fractures start at the nasofrontal and
frontomaxillary sutures and extend posteriorly along the medial wall of the orbit through the
nasolacrimal groove and ethmoid bones

Dishpan Face"

MORE FACTS

Ape face appearance in acromegaly.

LeForte III produces donkey face/ Dish-face deformity.

Lion face in leprosy.


Elephant face in plexiform neurofibroma.
Frog face in ethmoidal tumour.

QUESTION 54. NDE3MCtTSFVCSEFNICBTQUhBK3NodWJoYW0uc2FoYTQ1MkBnbWFpbC5jb20rOTkzMjE2M


USU9OIDUz
A 30 year old female with history of singing, vocal abuse and gastrooesphageal efflux
developed nodules at junction of anterior 1/3 rd & middle 3 rd of her vocal cords. What
is treatment of choice?

a) Speech therapy and Proton Pump Inhibitor (PPI)


b) Microlaryngoscopic surgery & CO 2 laser
c) Microlaryngectomy
d) Direct laryngoscopy and biopsy
Correct Answer: A
Your Answer: Unanswered
Explanation

POSITIVE MED PG
Web: http://www.positivemedpg.com/ Score:
Email: manohrajan@yahoo.co.in 0.00 / 100
Page 100
NIMHANS - DM Neurology -
Exam Title :
MO...
Email : shubham.saha452@gmail.com
Contact : 9932160514

Explanation

Nodules are typically treated conservatively with voice therapy and behavioral modification
under the guidance of a speech language pathologist. Surgery is reserved for refractory lesions
or in situations where vocal needs are not being met with voice therapy alone.

Treatment of underlying medical problems that affect the voice, such as reflux, allergies, and
sinusitis, may help lessen the severity or occurrence of vocal lesions and enhance vocal hygiene
in general

MORE NIMHANS FACTS

Which one of the following is true about GH? [Nim June 2010]

a. GH secretion increases after exercise, stress and hypoglycemia

b. GH secretion is maximum during evening

c. GH is produced equally in all age groups

d. GH decreases circulating IGF and Somatostatin

GH is the most abundant anterior pituitary hormone

The pituitary GH gene (hGH-N) give rise to 191 amino acids GH.

POSITIVE MED PG
Web: http://www.positivemedpg.com/ Score:
Email: manohrajan@yahoo.co.in 0.00 / 100
Page 101
NIMHANS - DM Neurology -
Exam Title :
MO...
Email : shubham.saha452@gmail.com
Contact : 9932160514

Somatostatin [somatotropin-release inhibiting factor (SRIF)] is synthesized in the


medial preoptic area of the hypothalamus and inhibits GH secretion

GH secretion is pulsatile, with highest peak levels occurring at night*****. Hormone


levels in middle age is 15% of pubertal levels******. GH secretion is reduced in obese
individuals.

The majority of action of GH is via IGF-I. The liver is the major source of circulating
IGF-I.

STIMULATORS

Ghrelin
(Gastric derived peptide)

Estrogen

High protein meals

Sleep

Prolonged fasting ******

Dopamine

L-arginine

Apomorphine

Alfa agonist

Ans (A)

MORE NIMHANS FACTS

Growth Hormone Insensitivity is caused by defects of GH receptor structure and are


associated with partial or complete GH insensitivity and growth failure called as
Laron syndrome. This has normal or high GH levels, with decreased circulating GHBP,
and low IGF-I levels.

QUESTION 55. NDE3MCtTSFVCSEFNICBTQUhBK3NodWJoYW0uc2FoYTQ1MkBnbWFpbC5jb20rOTkzMjE2M


USU9OIDU0
The muscle relaxant which causes maximum pain in injection site is: which of the following

a) Succinyl choline
b) Vecuronium bromide
c) Cisatracurium
d) Rocuronium bromide
Correct Answer: D
Your Answer: Unanswered
Explanation

POSITIVE MED PG
Web: http://www.positivemedpg.com/ Score:
Email: manohrajan@yahoo.co.in 0.00 / 100
Page 102
NIMHANS - DM Neurology -
Exam Title :
MO...
Email : shubham.saha452@gmail.com
Contact : 9932160514

rocuronium bromide can cause intense pain as it is injected*****

NMJ blockers

NON-DEPOLARISING
Bind to Alpha sub units of NMJreceptors and prevent acetylcholine (Ach) from stimulating
receptors.
Effect: compete with Ach for nicotinic receptor binding sites. The blockade is competitive, h
ence muscle paralysis occurs gradually.
Examples include tubocurarine, gallamine, atracurium, vecuronium, mivacurium, rocuronium
and cisatracurium.

These drugs are highly ionised at body pH and contain two quaternary ammonium groups. They
are poorly lipid soluble and poorly protein bound.

Nondepolarizing Muscle Relaxants

• Long acting
◦ Pancuronium
• Intermediate acting
◦ Atracurium
◦ Vecuronium
◦ Rocuronium
◦ Cisatracurium
• Short acting
◦ Mivacurium

POSITIVE MED PG
Web: http://www.positivemedpg.com/ Score:
Email: manohrajan@yahoo.co.in 0.00 / 100
Page 103
NIMHANS - DM Neurology -
Exam Title :
MO...
Email : shubham.saha452@gmail.com
Contact : 9932160514

Nondepolarizing Muscle Relaxants

• Atracurium
• Metabolized by
◦ Ester hydrolysis
◦ Hofmann elimination
• Onset 3-5 minutes, duration 25-35 minutes
• Intubating dose 0.5 mg/kg
• MORE NIMHANS FACTS
• Side effects :

histamine release causing hypotension, tachycardia, bronchospasm

Laudanosine toxicity

Rocuronium

• Rocuronium is a monoquaternary amine with a rapid onset and intermediate duration of


action.
• It is 6-8times less potent than vecuronium.
• An intubating dose of 0.6 mg kg produces satisfactory intubating conditions within 60--90
s, and it can be used as an alternative to succinylcholine in larger doses for rapid
sequence intubation.

First Selective Relaxant Binding Agent


SUGAMMADEX

synthetic, gamma-cyclodextrin and is the first and only selective relaxant binding agent (SRBA).

Sugammadex is a unique reversal agent because it encapsulates and then inactivates rocuroni
um and vecuroniu m rather than counteracting their effects. With this innovative mode of
action, it provides rapid reversal of neuromuscular blockade caused by these neuromuscular
blocking agents.

Monitoring Neuromuscular Function

How to monitor?

• Clinical signs
• Use of nerve stimulator

• Clinical signs
• Signs of adequate recovery
◦ Sustained head lift for 5 seconds
◦ Lift the leg (child)
◦ Ability to generate negative inspiratory pressure at least 25 cmH 2 O, able to
swallow and maintain a patent airway
◦ Other crude tests : tongue protrusion, arm lift, hand grip strength

POSITIVE MED PG
Web: http://www.positivemedpg.com/ Score:
Email: manohrajan@yahoo.co.in 0.00 / 100
Page 104
NIMHANS - DM Neurology -
Exam Title :
MO...
Email : shubham.saha452@gmail.com
Contact : 9932160514

Evoked responses during depolarizing and nondepolarizing block

POSITIVE MED PG
Web: http://www.positivemedpg.com/ Score:
Email: manohrajan@yahoo.co.in 0.00 / 100
Page 105
NIMHANS - DM Neurology -
Exam Title :
MO...
Email : shubham.saha452@gmail.com
Contact : 9932160514

Monitoring Neuromuscular Function

Use of nerve stimulator

• Single twitch : single pulse 0.2 msec


• Tetanic stimulation
• Train-of-four : series of 4 twitch, 0.2 msec long, 2 Hz frequency, administer every 10-15
seconds
• Double burst stimulation
• Post tetanic count

MORE NIMHANS FACTS

Drug interactions: -

1. GA – potentiate competitive blockers – ether, isoflurane potentiate most &Nitrous oxide to


least extent.

2. Flourinated anesthetics predispose to Phase II blockade by Sch.

3. Maligant hyperthermia is more common with sch & halothane or isoflourane used
concomitantly.

POSITIVE MED PG
Web: http://www.positivemedpg.com/ Score:
Email: manohrajan@yahoo.co.in 0.00 / 100
Page 106
NIMHANS - DM Neurology -
Exam Title :
MO...
Email : shubham.saha452@gmail.com
Contact : 9932160514

4. Anticholinesterase – reverse competitive blockers. Neostigmine is used.

5. Antibiotics – Aminoglycosides reduce Ach release – potentiate block.

6. CCBs – verapamil – potentiate SMKRs.

7. Diuretics – hypokalemia can enhance block.

Complications of Succinylcholine Administration

Cardiovascular Tachycardia (ganglionics stimulation)

Bradycardia, sinus arrest, junctional rhythm

Hyperkalemia

Fasciculations; myalgia*

Myoglobinuria, elevated plasma creatine phosphokinase

Sustained muscle contraction (myotonic dystrophy, congenital myotonia)

Malignant hyperpyrexia

Masseter muscle rigidity

Prolonged relaxation

Phase II block

Inadequate pseudocholinesterase activity

Increased intraocular pressure*

Increased intracranial pressure*

Increased intragastric pressure*

Histamine release

QUESTION 56. NDE3MCtTSFVCSEFNICBTQUhBK3NodWJoYW0uc2FoYTQ1MkBnbWFpbC5jb20rOTkzMjE2M


USU9OIDU1
POYNTING EFFECT (overpressure effect) is associated with which of the following

a) Ether
b) Cyclopropane
c) Nitrous oxide
d) Trichlorethylene
Correct Answer: C
Your Answer: Unanswered
Explanation

POSITIVE MED PG
Web: http://www.positivemedpg.com/ Score:
Email: manohrajan@yahoo.co.in 0.00 / 100
Page 107
NIMHANS - DM Neurology -
Exam Title :
MO...
Email : shubham.saha452@gmail.com
Contact : 9932160514

NITROUS OXIDE

• manufactured by heating ammonium nitrate to 250°C


• NH 4 NO 3 » N 2 O + 2H 2 O
• During this process, a number of contaminants may be produced, unless the temperature
is carefully controlled. These include NH 3 , N 2 , NO, NO 2 and HNO 3 , and they are
actively removed by passage through scrubber, caustic soda and water.
• Storage
• French blue cylinders are used to store N 2 O in a liquid phase with its vapour on top at
a gauge pressure of 4400 kPa at room temperature. As the liquid is less compressible
than a gas, the cylinder should be only partially filled.
• The filling ratio is the weight of the fluid in the cylinder divided by the weight of water
required to fill the cylinder. In the UK, the filling ratio for N 2 O is 0.75, but in hotter
climates the filling ratio needs to be 0.67, to avoid cylinder explosion.

• Physicochemical properties
• Molecular weight 44
Boiling point -88°C
Critical temperature 36.5°C
Critical pressure 72 bar
MAC 105%
Partition coefficients Blood/gas 0.47, Oil/gas 1.4

• Fink effect –diffusion hypoxia during recovery.

• POYNTING EFFECT (overpressure effect) ***** – the critical temperature and


pressure of a gas may be affected when it is mixed with another gas. For example, in a
cylinder of Entonox , the new critical temperature of nitrous oxide (known as the
pseudocritical temperature) changes to –6 o C. Therefore, precautions regarding the
cooling of cylinders should be taken into account.

FINK EFFECT – Diffusion hypoxia

MORE NIMHANS FACTS-[Nim June 2010]

Measurement of serum osmolality

In “conventional” units, the calculated serum osmolality (2 × [serum sodium + serum


potassium] +

plasma glucose [mg/dL]/18 + BUN/2.8)

QUESTION 57. NDE3MCtTSFVCSEFNICBTQUhBK3NodWJoYW0uc2FoYTQ1MkBnbWFpbC5jb20rOTkzMjE2M


USU9OIDU2
Father of local anaesthesia is which of the following scientist

a) karl coller

POSITIVE MED PG
Web: http://www.positivemedpg.com/ Score:
Email: manohrajan@yahoo.co.in 0.00 / 100
Page 108
NIMHANS - DM Neurology -
Exam Title :
MO...
Email : shubham.saha452@gmail.com
Contact : 9932160514

b) Morton
c) john snow
d) H F W Braun
Correct Answer: A
Your Answer: Unanswered
Explanation

Karl Koller (1857-1944), Austrian ophthalmologist. Koller is best known for


introducing cocaine as a local anaesthetic for eye surgery.

MORE NIMHANS FACTS

The impulse propagation is associated with movement of sodium ions inwards and
potasium ions outwards , through their respective channels. The membrane becomes
depolarized. During recovery the ions reverse the direction of their movement across the cell
membrane. Local anaesthetic agents may prevent depolarization and so may prevent
conduction of impulses. It is thought that local anaesthetic drugs exert their effect by
bonding to the internal mouth of the sodium channel . The local analgesic solution preve
nts depolarization of the nerve membrane . As the concentration increases the height of
the action potential is reduced. The firing threshold is elevated, the spread of impulse
conduction is slowed and the refractory period lengthened. Finally nerve conduction is
completely blocked.

MORE NIMHANS FACTS

POSITIVE MED PG
Web: http://www.positivemedpg.com/ Score:
Email: manohrajan@yahoo.co.in 0.00 / 100
Page 109
NIMHANS - DM Neurology -
Exam Title :
MO...
Email : shubham.saha452@gmail.com
Contact : 9932160514

LOCAL ANESTHESIA: -

1. Injectable anaesthetic –

Low potency – procaine, chloroprocaine

Intermediate potency – lidocaine, prilocaine

High potency; longer duration of action – tetracaine, bupivacaine, ropivacaine, dibucaine.

1. Surface anaesthetic

1. Soluble –cocaine, lidocaine, tetracaine & beonxinate

2. Insoluble – benzocaine, butamben, oxethazine

2. Others- propranolol, chlorpromazine, H1 anti-histaminics, quinine etc.

Amide-linked – lidocaine, bupivacaine, dibucaine, prilocaine, ropivacaine.

Ester – linked – cocaine, procaine, chloroprocaine, tetracine, benzocaine.

i. Produce more intense, rapidly acting & longer acting anesthesia.

ii. Blind to α1 acid glycoprotein.

iii. Not hydrolyzed by plasma cholinesterase.

iv. Rarely cause hypersensitivity.

Mechanism of Action : - it blocks neuronal axonal membrane sodium channels in its


inactivated states. Thus refractory period of nerve fiber is increased – conduction is blocked.
Factors affecting –

Repeatedly firing neuron is blocked more intensly than a resting nerve. The time of onset of
block depends on the pKa value of LA. Higher the pKa – slower the onset – procaine, tetracaine
& bupivacaine except for chloroprocaine.

Blockade of sodium channels is voltage & time dependent.

It is determined also by type of nerve fiber & its diameter – myelinated fibers are affected more
than unmyelinated.

Atuonomic fibers are more susceptible than somatic fibers.

Order of blockade – pain – temp – touch – deep pressure.

Effect of addition of adrenaline –

1. Prolongs duration of anesthesia.

2. Enhances intensity & reduces systemic side effects – due to reduced systemic absorption.

3. Makes injection more painful.

4. Bloodless field is available for surgery.

POSITIVE MED PG
Web: http://www.positivemedpg.com/ Score:
Email: manohrajan@yahoo.co.in 0.00 / 100
Page 110
NIMHANS - DM Neurology -
Exam Title :
MO...
Email : shubham.saha452@gmail.com
Contact : 9932160514

5. Increase tissue edema & necrosis

6. May raise BP & arrhythmia can occur.

Effects: -

1. CNS – cocaine causes stimulation & lignocaine causes depression.

2. CVS – LA are cardiac depressant – hypotension & HR. Bupivacaine produces cardiotoxicity &
Ventricular tachycardia.

3. Blood vessels – direct vasodilatation. Bupivacaine is most & prilocaine is least.

1. Lignocaine – best anesthetic with good surface & injectable anesthesia.

2. Prilocaine – larger volume of distribution. Methamoglobinemia.

3. Eutectic lidocaine / prilocaine – reduction in melting point of 2 solids when mixed. This
increases the penetration of lidocaine – IV cannulation, short dermatological procedures, split
skin graft harvesting.

4. Tetracaine – highly lipid soluble PABA ester –more potent & more toxic – topical application is
used.

5. Bupivacaine- potent, long acting amide derivative useful for nerve block, spinal or epidural
anesthesia. Useful in labor anesthesia. Prolongs QTc – produces arrhythmias. Levobupivacaine
is cardiotoxic metabolite; prone to seizures.

6. Ropivacaine – less cardiotoxic metabolite than bupivacaine.

7. Benoxinate – corneal anesthesia

8. Benzocaine & butamben – PABA derivates – antagonize sulfonamides. Long lasting anesthesia
for mucous membranes & abraded skin surfaces – poor systemic absorption.

Uses –

1. Surface anesthesia

2. Infiltration anesthesia

3. Conduction block

4. Spinal anesthesia

5. Epidural &

6. IV regional anesthesia

QUESTION 58. NDE3MCtTSFVCSEFNICBTQUhBK3NodWJoYW0uc2FoYTQ1MkBnbWFpbC5jb20rOTkzMjE2M


USU9OIDU3
Optical density values in Liley’S chart in Zone 3 indicates which of the following

a) Mild disease

POSITIVE MED PG
Web: http://www.positivemedpg.com/ Score:
Email: manohrajan@yahoo.co.in 0.00 / 100
Page 111
NIMHANS - DM Neurology -
Exam Title :
MO...
Email : shubham.saha452@gmail.com
Contact : 9932160514

b) Severe disease
c) moderate to severe disease
d) Death within 7-10 days
Correct Answer: D
Your Answer: Unanswered
Explanation

Mild
Zone 2 indicates moderate to severe disease

Zone3- severely affected fetus.Death in 7-10 daysIUT or Delivery indicated.

MORE NIMHANS FACTS-[Nimhans 2013]

REWARD PATHWAY

Neuropathology -the nucleus accumbens is associated with brain reward *****

Deep brain stimulation (DBS) of either the nucleus accumbens or subgenual area 25
elevates mood

POSITIVE MED PG
Web: http://www.positivemedpg.com/ Score:
Email: manohrajan@yahoo.co.in 0.00 / 100
Page 112
NIMHANS - DM Neurology -
Exam Title :
MO...
Email : shubham.saha452@gmail.com
Contact : 9932160514

Addictive drugs increase dopamine release in the nucleus accumbens and blockade of
dopamine in the nucleus accumbens can stop the rewarding effects of addictive drugs

QUESTION 59. NDE3MCtTSFVCSEFNICBTQUhBK3NodWJoYW0uc2FoYTQ1MkBnbWFpbC5jb20rOTkzMjE2M


USU9OIDU4
In IAP classification of malnutrition,Grade 1 malnutrition implies a weight of which of the
following

a) 71 – 80%
b) 61 – 70%
c) 51 – 60%
d) < 50%
Correct Answer: A
Your Answer: Unanswered
Explanation

Explanation

Degrees of failure to thrive

Weight for age (% of Height for age (% of Weight for height (% of


Degree
median) median) median)

Mild 75 – 90 90 - 95 81 – 90

Moderate 60 – 74 85 - 89 70 – 80

Severe < 60 < 85 < 70

Indicator Interpretation

Indicator of chronic malnutrition, the result of prolonged


Stunting Low height for age
food deprivation and/or illness

Low weight for Suggests acute malnutrition, the result of more recent food
Wasting
height deficit or illness

Combined indicator to reflect both acute and chronic


Underweight Low weight for age
malnutrition.

WHO classification of malnutrition

POSITIVE MED PG
Web: http://www.positivemedpg.com/ Score:
Email: manohrajan@yahoo.co.in 0.00 / 100
Page 113
NIMHANS - DM Neurology -
Exam Title :
MO...
Email : shubham.saha452@gmail.com
Contact : 9932160514

Moderate malnutrition Severemalnutrition

Symmetrical edema No Yes (edematous malnutrition)

Weight for height SD score between -2 to -3 SD score < -3 (severe wasting)

Height for age SD score between -2 to -3 SD score <-3 (severe stunting)

IAP classification of malnutrition

Grade of malnutrition Weight for age of the standard (median) (%)

Normal > 80%

Grade I 71 – 80% (mild malnutrition)******

Grade II 61 – 70% (moderate malnutrition)

Grade III 51 – 60% (severe malnutrition)

Grade IV < 50% (very severe malnutrition)

QUESTION 60. NDE3MCtTSFVCSEFNICBTQUhBK3NodWJoYW0uc2FoYTQ1MkBnbWFpbC5jb20rOTkzMjE2M


USU9OIDU5
Esophageal atresia with distal Tracheoesophageal fistula belongs to which of the following type

a) Type A
b) Type B
c) Type C
d) Type D
Correct Answer: C
Your Answer: Unanswered
Explanation

Explanation

POSITIVE MED PG
Web: http://www.positivemedpg.com/ Score:
Email: manohrajan@yahoo.co.in 0.00 / 100
Page 114
NIMHANS - DM Neurology -
Exam Title :
MO...
Email : shubham.saha452@gmail.com
Contact : 9932160514

In H type TEF, an esophagogram with contrast medium injected under pressure can
demonstrate the defect. Alternatively, the orifice may be detected at bronchoscopy or when
methylene blue dye injected into the endotracheal tube during endoscopy is observed in the
esophagus during inspiration.

MORE NIMHANS FACTS-NIMHANS 2013

Which one of the Interleukins is produced by Th 1 cells? [Nimhans 2013]

a. IL 1

b. IL 2

c. IL 3

c. IL 4

T H 1-type helper T cells - a central role in mediating intracellular killing- IL-2******,


IFN-, IL-3, TNF-alfa, GM-CSF, and TNF-beta

T H 2-type helper T cells - regulatory humoral immunity and isotype switching. IL-3,
-4, -5, -6, -10, and -13

Ans B

QUESTION 61. NDE3MCtTSFVCSEFNICBTQUhBK3NodWJoYW0uc2FoYTQ1MkBnbWFpbC5jb20rOTkzMjE2M


USU9OIDYw

POSITIVE MED PG
Web: http://www.positivemedpg.com/ Score:
Email: manohrajan@yahoo.co.in 0.00 / 100
Page 115
NIMHANS - DM Neurology -
Exam Title :
MO...
Email : shubham.saha452@gmail.com
Contact : 9932160514

All of the following are syndromes associated with obesity except

a) Turner syndrome
b) Carpenter syndrome
c) Prader willi syndrome
d) Klinefelter syndrome
Correct Answer: D
Your Answer: Unanswered
Explanation

Explanation

Body Mass Index (BMI) Classification of obesity in Adults

BMI (kg/m 2 ) WEIGHT STATUS

<18.5 Underweight

18.5–24.9 Normal weight

25–29.9 Overweight

30–34.9 Obese

35–39.9 Moderately obese

40–49.9 Morbid obesity

≥50 Super morbid obesity

Body Mass Index (BMI) Classification of Children and Adolescents

BMI PERCENTILE FOR AGE WEIGHT STATUS

<5th percentile Underweight

5th–84th percentile Normal weight

85th–94th percentile At risk for overweight

POSITIVE MED PG
Web: http://www.positivemedpg.com/ Score:
Email: manohrajan@yahoo.co.in 0.00 / 100
Page 116
NIMHANS - DM Neurology -
Exam Title :
MO...
Email : shubham.saha452@gmail.com
Contact : 9932160514

BMI PERCENTILE FOR AGE WEIGHT STATUS

≥95th percentile Overweight

Syndromes associated with obesity:

• Prader-Willi syndrome*****
• Carpenter syndrome*****
• Cohen syndrome
• Cushing syndrome
• Turner syndrome*****

QUESTION 62. NDE3MCtTSFVCSEFNICBTQUhBK3NodWJoYW0uc2FoYTQ1MkBnbWFpbC5jb20rOTkzMjE2M


USU9OIDYx
Ipsilateral optic atrophy with contralateral papilloedema is a feature of Which of the following
syndrome

a) Fischer syndrome
b) Foster kennedy syndrome
c) Vogt-kayanagi-Harada syndrome
d) WAGR syndrome
Correct Answer: B
Your Answer: Unanswered
Explanation

Explanation

• Papilloedema is mostly due to raised intracranial pressure. So it is usually bilateral.


Unilateral causes of papilloedema are:

Foster Kennedy Syndrome

• Olfactory or sphenoidal meningiomata & frontal lobe tumors leads to optic atrophy on the
side of lesion (I/L) and papilloedema on the other slide (C/L) due to increase ICT

POSITIVE MED PG
Web: http://www.positivemedpg.com/ Score:
Email: manohrajan@yahoo.co.in 0.00 / 100
Page 117
NIMHANS - DM Neurology -
Exam Title :
MO...
Email : shubham.saha452@gmail.com
Contact : 9932160514

Pseudo Foster Kennedy Syndrome

• U.L papilloedema associated with raise ICT (due to any cause) & preexisting optic atrophy
(d/t any other cause)

MORE NIMHANS FACTS-[Nimhans 2013]

Syndrome of inappropriate antidiuretic hormone secretion (SIADH),

* Most common cause of normovolemic hyponatremia is SIAD and there is mild volume-
expansion.

* Hypoosmotic hyponatremia occurs in the setting of an inappropriately concentrated urine


(urine osmolality >100 mosmol/kg) . [AIIMS N0V-2007***]

* It is due to nonphysiologic release of AVP from the posterior pituitary or an ectopic source.
Renal free-water excretion is impaired while the regulation of Na + balance is unaffected.

* The most common causes of SIADH is neuropsychiatric and pulmonary diseases, malignant
tumors, major surgery (postoperative pain), and pharmacologic agents. The commonest drug
causing SIAD is SSRI and the commonest tumor causing is small cell carcinoma

Investigations for confirming the diagnosis are

1) low serum osmolality,

2) high urine osmolality******

3) Urine Na + concentration usually >40 mmol/L .


4) Hypouricemia due to the uricosuric state induced by volume expansion

5) absence of hypothyroidism

QUESTION 63. NDE3MCtTSFVCSEFNICBTQUhBK3NodWJoYW0uc2FoYTQ1MkBnbWFpbC5jb20rOTkzMjE2M


USU9OIDYy
Visual pigments of peak spectral sensitivity: 560 nm is which of the following

a) Red cone
b) Blue cone
c) Green cone
d) ALL OF THE ABOVE
Correct Answer: A
Your Answer: Unanswered
Explanation

Explanation

POSITIVE MED PG
Web: http://www.positivemedpg.com/ Score:
Email: manohrajan@yahoo.co.in 0.00 / 100
Page 118
NIMHANS - DM Neurology -
Exam Title :
MO...
Email : shubham.saha452@gmail.com
Contact : 9932160514

NIMHANS FACTS

The retina contains three classes of cones, with visual pigments of differing peak spectral
sensitivity: red (560 nm), green (530 nm), and blue (430 nm). The red and green cone pigments
are encoded on

the X chromosome, and the blue cone pigment on chromosome 7. Mutations of the blue cone
pigment are exceedingly rare. Mutations of the red and green pigments cause congenital X-
linked color blindness in 8% of males.

QUESTION 64. NDE3MCtTSFVCSEFNICBTQUhBK3NodWJoYW0uc2FoYTQ1MkBnbWFpbC5jb20rOTkzMjE2M


USU9OIDYz
A patient was diagnosed to have Chondroblastoma.All of the following are true
regarding it except:

a) most frequently arise in the epiphyses of long bones


b) Femur is the most common bone affected
c) predominantly in young males (<20 years of age)
d) A joint effusion is seen in ~1/3 of patients
Correct Answer: B
Your Answer: Unanswered
Explanation

Commonest bone involved is humerus*****

Chondroblastomas (or Codman tumours )are rare benign cartilaginous neoplasms that
characteristically arise in the epiphysis or apophysis of a long bone in young patients . Despite
being rare, they are one of the most frequently encoutered benign epiphyseal neoplasms in
skeletally immature patients.

POSITIVE MED PG
Web: http://www.positivemedpg.com/ Score:
Email: manohrajan@yahoo.co.in 0.00 / 100
Page 119
NIMHANS - DM Neurology -
Exam Title :
MO...
Email : shubham.saha452@gmail.com
Contact : 9932160514

Epidemiology

Chondroblastomas represent less than 1% of all primary bone tumours, occurring


predominantly in young patients (<20 years of age) . There is a male predilection***** .

Clinical presentation

Clinical presentation is non-specific and may include joint pain, muscle wasting, tenderness,
and swelling/local mass.

Pathology

Microscopically they are composed of chondroblasts, chondroid matrix, cartilage with


occasional giant multi-nucleated cells (which may lead to the incorrect diagnosis of giant cell
tumour ).

Calcium deposition surrounding the chondroblasts, which are typically polyhedral shape,
results in typical "chicken-wire calcification" (pathognomonic)

Aneurysmal bone cysts can be seen secondarily to underlying chondroblastoma.

Distribution

Chondroblastomas most frequently arise in the epiphyses of long bones , with 70%
occurring in the humerus (most frequent) , femur and tibia 9 . ~10% are found in the
hands and feet 7 .

Radiographic features

Chondroblastomas (along with chondromyxoid fibroma ) constitute the “C” in the FEGNOMASH
IC mnemonic for benign appearing cystic lesions

POSITIVE MED PG
Web: http://www.positivemedpg.com/ Score:
Email: manohrajan@yahoo.co.in 0.00 / 100
Page 120
NIMHANS - DM Neurology -
Exam Title :
MO...
Email : shubham.saha452@gmail.com
Contact : 9932160514

Plain film

Chondroblastomas are seen as well defined lucent lesions, with either smooth or lobulated
margins and a thin sclerotic rim, arising eccentrically in the epiphysis of long tubular bone such
as the femur, humerus, or tibia or apophysis such as greater trochanter, greater tuberosity,
calcaneus or talus. Internal calcifications can be seen in up to 40-60% of cases . A joint effusion
is seen in ~1/3 of patients******. They range in size from 1-10 cm, with most being 3-4 cm at
diagnosis

CT

Solid periosteal reaction (seen in up to 50% of cases) and internal calcification (calcified matrix
seen in ~1/2 of cases) and cortical breach are also more easily appreciated Endosteal scalloping
may be seen

MRI

These lesions have signal typical of cartilage:

T1: lesion itself is of low to intermediate signal

T2/STIR: lesion is of intermediate to high signal

A.Epiphysis

Common epiphyseal lesions

POSITIVE MED PG
Web: http://www.positivemedpg.com/ Score:
Email: manohrajan@yahoo.co.in 0.00 / 100
Page 121
NIMHANS - DM Neurology -
Exam Title :
MO...
Email : shubham.saha452@gmail.com
Contact : 9932160514

Chondroblastoma

Brodies’s abscess

eosinophilic granuloma

Giant cell tumor

Aneurysmal bone cyst

Intra osseous ganglion

QUESTION 65. NDE3MCtTSFVCSEFNICBTQUhBK3NodWJoYW0uc2FoYTQ1MkBnbWFpbC5jb20rOTkzMjE2M


USU9OIDY0
One of the following is not an extra medullary intradural tumour

a) meningioma
b) neurofibroma
c) schwanoma
d) Ependymoma
Correct Answer: D
Your Answer: Unanswered
Explanation

xplanation

Ependymoma is Intramedullary******

Explanation

POSITIVE MED PG
Web: http://www.positivemedpg.com/ Score:
Email: manohrajan@yahoo.co.in 0.00 / 100
Page 122
NIMHANS - DM Neurology -
Exam Title :
MO...
Email : shubham.saha452@gmail.com
Contact : 9932160514

Commonest intradu.extramed.-neurofibroma

Astrocytoma,ependymoma,intramedullary

Extradural-mets

Myelogram

Brush border-broom stick -extradural Trouser leg - intramedullary

Meniscus,crescent - intradu. extramedullary

Intramedullary Syndromes

POSITIVE MED PG
Web: http://www.positivemedpg.com/ Score:
Email: manohrajan@yahoo.co.in 0.00 / 100
Page 123
NIMHANS - DM Neurology -
Exam Title :
MO...
Email : shubham.saha452@gmail.com
Contact : 9932160514

* Intramedullary lesions spare sensation in the perineal and sacral areas ("sacral sparing"), due
to the laminated configuration of the spinothalamic tract with sacral fibers outermost;
corticospinal tract signs appear later.

Extramedullary lesions

* Radicular pain is often prominent, and there is early sacral sensory loss (lateral spinothalamic
tract) and spastic weakness in the legs (corticospinal tract) due to the superficial location of leg
fibers in the corticospinal tract . Extradural are due to malignant lesions. Intradural lesions
are commonly due to benign lesions (neurofibroma being a common cause). Compressive
Myelopathies

* In adults, most neoplasms are epidural in origin, resulting from metastases to the adjacent
spinal bones. Ca breast, lung, prostate, kidney, lymphoma, and plasma cell dyscrasia commonly
metastasis to the vertebra. The thoracic spinal column is most commonly involved; exceptions
are metastases from prostate and ovarian cancer, which occur commonly in the sacral and
lumbar vertebrae. Pain is usually the initial symptom. Plain radiographs and radionuclide bone
scans miss 15–20% of metastatic vertebral lesions. MRI is the investigation of choice.

* Treatment : glucocorticoids, local radiotherapy (3000 cGy administered in 15 daily fractions


initiated as early as possible). Motor deficits, once established for >12 h, do not usually
improve. Intensity-modulated radiotherapy (IMRT), can deliver high doses of focused radiation.
Surgery, either decompression by laminectomy or vertebral body resection is done.

* Most intradural mass lesions are slow-growing and benign like meningiomas,
neurofibromas, chordoma, lipoma, dermoid, or sarcoma. Neurofibromas typically arise near the
posterior root.

* Primary intramedullary tumors present as central cord or hemicord syndromes, often in


the cervical region and are due to ependymomas, hemangioblastomas, or low-grade
astrocytomas.

QUESTION 66. NDE3MCtTSFVCSEFNICBTQUhBK3NodWJoYW0uc2FoYTQ1MkBnbWFpbC5jb20rOTkzMjE2M


USU9OIDY1
Which of the following gene abnormalities is associated with excess production of Beta – Globin
chains

a) Promoter region mutation of Beta chain


b) Chain Terminator mutation of Beta chain
c) Splicing mutation of beta chain
d) Deletion of three α chains
Correct Answer: D
Your Answer: Unanswered
Explanation

Explanation

When both alfa alleles on one chromosome are deleted, the locus is called alfa-thal-1; when only
a single alfa allele on one chromosome is deleted, the locus is called alfa-thal-2. AlfaThalasse
mia-2 trait is an asymptomatic, silent carrier state.

POSITIVE MED PG
Web: http://www.positivemedpg.com/ Score:
Email: manohrajan@yahoo.co.in 0.00 / 100
Page 124
NIMHANS - DM Neurology -
Exam Title :
MO...
Email : shubham.saha452@gmail.com
Contact : 9932160514

* Alfa-Thalassemia-1 trait resembles beta-thalassemia minor. It is asymptomatic hypochromia


and microcytosis. The RDW is normal.

* Homozygous state for the alfa-thalassemia-1 cis deletion (a deletion that removes both genes
from the same chromosome) (hydrops fetalis) causes total absence of alfa-globin synthesis.
(Fig.114-12). Excess globin forms tetramers called Hb Barts ( 4 ), which has a very high oxygen
affinity.

Beta thalassemia
Beta thalassemia major Beta thalassemia minor
intermedia

* Profound microcytosis and


* Severe anemia , massive
* Similar to major but can hypochromia with target
ineffective erythropoiesis:
survive without chronic cells, mild anemia, mean
hepatosplenomegaly,
hypertransfusion. corpuscular volume is rarely
microcytosis, and elevated HbF
>75 fL

* Factors aggravating anemia –


* Require chronic
infection, onset of puberty, and * Hemoglobin electrophoresis
hypertransfusion therapy-
development of splenomegaly –elevated HbA 2 (3.5–7.5%).
hematocrit of at least 27–30%
and hypersplenism

* Splenectomy if the annual


transfusion requirement
increases by >50%

* Folic acid supplements

* Vaccination with Pneumococcal


vaccine

* Endocrine evaluation – for


glucose intolerance, thyroid
dysfunction, and delayed onset
of puberty

MORE NIMHANS FACTS

Measure Conventional units

Hb 10 grams/deciliter

POSITIVE MED PG
Web: http://www.positivemedpg.com/ Score:
Email: manohrajan@yahoo.co.in 0.00 / 100
Page 125
NIMHANS - DM Neurology -
Exam Title :
MO...
Email : shubham.saha452@gmail.com
Contact : 9932160514

RBC 5E+12 cells/liter

MCV = Hct / RBC 80 femtoliters /cell

MCH = Hb / RBC 20 picograms /cell

MCHC = Hb / HCT 25 grams/deciliter

QUESTION 67. NDE3MCtTSFVCSEFNICBTQUhBK3NodWJoYW0uc2FoYTQ1MkBnbWFpbC5jb20rOTkzMjE2M


USU9OIDY2
Complications of Allogenic bone marrow Transplantation include all except

a) Chronic graft versus host disease


b) Cataract
c) Infertility
d) Glaucoma
Correct Answer: D
Your Answer: Unanswered
Explanation

Explanation

Complications Following Hematopoietic Cell Transplant

1. * High-dose cyclophosphamide can result in hemorrhagic cystitis.Patients begin


losing their hair 5–6 days posttransplant and by 1 week are pancytopenic.Keratinocyte
growth factor (palifermin) (???AI-2015***) can shorten the duration of
mucositis.Venoocclusive disease of the liver presents with Tender hepatomegaly,
ascites, Jaundice, Fluid retention.Peak on 16 th day.It is seen in 10% of BMT patients.
Mortality is 30%. Treatment is with defibrotide ,a polydeoxyribonucleotide. Most
men become azoospermic, and most postpubertal women will develop ovarian
failure.Cataracts develop in 10–20%******.Aseptic necrosis of the femoral head is seen
in 10% of patients.

Graft-versus-Host Disease

* GVHD developing within the first 3 months posttransplant is termed acute GVHD, while
GVHD developing or persisting beyond 3 months posttransplant is termed chronic GVHD. Acut
e presents Within 3 monthsof Transplant. It presents with erythematous rash, diarrhoea, and
elevated OT/PT. (AI-08♦♦♦)[AIIMS MAY 2007]. Biopsy reveasls Endothelial damage with
lymphocytic infiltration.In skin, the epidermis and hair follicles are damaged; in liver, the small
bile ducts show segmental disruption; and in intestines, destruction of the crypts and mucosal
ulceration. Prevention is by Steroids, Cyclosporine, Tacrolimus, MMF, Methotrexate,
Cyclophosphamide.

Staging

POSITIVE MED PG
Web: http://www.positivemedpg.com/ Score:
Email: manohrajan@yahoo.co.in 0.00 / 100
Page 126
NIMHANS - DM Neurology -
Exam Title :
MO...
Email : shubham.saha452@gmail.com
Contact : 9932160514

The staging of acute GVHD is listed below.

Clinical Liver—Bilirubin, (mg/


Skin Gut
Stage dL)

Diarrhea 500–1000 mL/


1 Rash <25% body surface 2–3
d

Rash 25–50% body Diarrhea 1000–1500


2 3–6
surface mL/d

Generalized
3 6–15 Diarrhea >1500 mL/d
erythroderma

4 Desquamation and bullae > 15 Ileus

* Chronic-it occurs m ore common in elderly, unrelated donor, prior acute GVHD. It r esembl
es an autoimmune disorder and presents with malar rash, sicca syndrome, Arthritis and
obliterative bronchiolitis.It is also called as Runts disease. Treatment is with Prednisolone,
cyclosporine, Thalidomide.

QUESTION 68. NDE3MCtTSFVCSEFNICBTQUhBK3NodWJoYW0uc2FoYTQ1MkBnbWFpbC5jb20rOTkzMjE2M


USU9OIDY3
During excercise Mr. Ram consumes 1.8L of 02/min. His arterial 02 content is 190ml/l and the
02 content of his mixed venous blood is 130 ml/l calculate his cardiac output.

a) 3.2 L/min
b) 16 L/min
c) 30 L/min
d) 54 L/min
Correct Answer: A
Your Answer: Unanswered
Explanation

Explanation

POSITIVE MED PG
Web: http://www.positivemedpg.com/ Score:
Email: manohrajan@yahoo.co.in 0.00 / 100
Page 127
NIMHANS - DM Neurology -
Exam Title :
MO...
Email : shubham.saha452@gmail.com
Contact : 9932160514

MORE NIMHANS FACTS-NIMHANS 2013

FALSE statement about High Anion gap [Nimhans 2013]

a. Seen in uraemia

b. Iron causes high anion gap

c. Hypercholremia present

d. Acidosis is present

An increase in the AG is often due to an increase in unmeasured anions and less commonly is
due to a decrease in unmeasured cations (calcium, magnesium, potassium). The condition
causing high anion gap acidosis are DKA, uremia****, Lactic acidosis, Methanol toxicity,
Starvation and salicylates . [AIIMS MAY-2007***]

Hyperchloremic****** (also called as Nongap) Metabolic

Causes of Non-Anion-Gap Acidosis are diarrhea , [TN-2007***] Renal tubular acidosis,


mineralocorticoid resistance (autosomal dominant PHA I), voltage defect (autosomal dominant
PHA I and PHA II), external pancreatic or small-bowel drainage, ureterosigmoidostomy ,
jejunal loop, ileal loop

Ans C

QUESTION 69. NDE3MCtTSFVCSEFNICBTQUhBK3NodWJoYW0uc2FoYTQ1MkBnbWFpbC5jb20rOTkzMjE2M


USU9OIDY4
Regarding Human poverty index,which of the following statements is false

POSITIVE MED PG
Web: http://www.positivemedpg.com/ Score:
Email: manohrajan@yahoo.co.in 0.00 / 100
Page 128
NIMHANS - DM Neurology -
Exam Title :
MO...
Email : shubham.saha452@gmail.com
Contact : 9932160514

a) It was introduced in 1987


b) HPI is expressed as percentages.
c) It includes percentage of population not using an improved water source and the
percentage of children under weight for age
d) HPI – 2 includes the following dimensions,Probability at birth of not surviving to age
60Exclusion from the world of reading and communications as measured by percentage of
adults (16 – 65) lacking functional literacy skills
Correct Answer: A
Your Answer: Unanswered
Explanation

Explanation

Human poverty Index – term was introduced in 1997.******

The dimensions for HPI – 1, while HDI measures average achievements in basic dimensions of
human development, the HPI measures deprivation in those dimensions. The dimensions used
in HPI – 1

A long and healthy life vulnerability to death at a relatively early age (probability at birth of not
surviving to age 40.

Knowledge – exclusion from the world of reading and communications.

A decent standard of living – lack of access to overall economic provisioning (percentage of


population not using an improved water source and the percentage of children under weight for
age)

HPI is expressed as percentages.

HPI – 2 measures deprivation in the same dimensions as the HPI – 1 and also captures social
exclusion (as measured by the rate of long-term unemployment 12 months or more)

HPI – 2 includes the following dimensions,

Probability at birth of not surviving to age 60

Exclusion from the world of reading and communications as measured by percentage of adults
(16 – 65) lacking functional literacy skills.

Percentage of people living below the income poverty line

Social exclusion – as measured by the rate of long term unemployment 12 months or more

Gender related development index (GDI) – introduce during the year 1995

Reflects achievements in the basic human development adjusted for gender inequalities

GEM – gender empowerment measure – measures gender inequalities in economic and political
opportunitie

QUESTION 70. NDE3MCtTSFVCSEFNICBTQUhBK3NodWJoYW0uc2FoYTQ1MkBnbWFpbC5jb20rOTkzMjE2M


USU9OIDY5

POSITIVE MED PG
Web: http://www.positivemedpg.com/ Score:
Email: manohrajan@yahoo.co.in 0.00 / 100
Page 129
NIMHANS - DM Neurology -
Exam Title :
MO...
Email : shubham.saha452@gmail.com
Contact : 9932160514

Years of potential life lost (YPLL)- is defined as one that occurs before the age to which a dying
person could have expected to survive -before an arbitrary determined age, usually taken ageis

a) 64 years
b) 68 years
c) 72 years
d) 75 years
Correct Answer: D
Your Answer: Unanswered
Explanation

Indicators OF HEALTH

1. Mortality indicators

a. Crude death rate :

*It is defined as the number of deaths per 1000 population per year in a given community.

b. Expectation of life :

* Life expectancy at birth is “the average number of years that will be lived by those born alive
into a population if the current age-specific mortality rates persist”

c. Age-specific death rates :

d. Infant mortality rate :

* Infant mortality rate is the ratio of deaths under 1 year of age in a given year to the total
number of live births.

e. Child mortality rate :

* number of deaths at ages 1-4 years in a given year, per 1000 children in that age group.

f. Under-5 proportionate mortality :

* It is the proportion of total deaths occuring in the under-5 age group.

g . Adult mortality rate :

* The adult mortality rate is defined as the probability of dying between the age of 15 and 60
years per 1000 population.

h. Proportional mortality rate:

* The simplest measure of estimating the burden of a disease in the community is proportional
mortality rate.

i. Case fatality rate :

* Number of deaths from a specific disease during a specific time period divided by number of
cases of the disease during the same time period, usually expressed as per 100.

j. Years of potential life lost (YPLL)

POSITIVE MED PG
Web: http://www.positivemedpg.com/ Score:
Email: manohrajan@yahoo.co.in 0.00 / 100
Page 130
NIMHANS - DM Neurology -
Exam Title :
MO...
Email : shubham.saha452@gmail.com
Contact : 9932160514

* It is defined as one that occurs before the age to which a dying person could have expected to
survive (before an arbitrary determined age, usually taken age 75 years.)

QUESTION 71. NDE3MCtTSFVCSEFNICBTQUhBK3NodWJoYW0uc2FoYTQ1MkBnbWFpbC5jb20rOTkzMjE2M


USU9OIDcw
The familial form of Alzheimer's disease is ______ and carried on ______

a) Autosomal dominant; chromosome 21 or chromosome 19


b) Autosomal dominant; chromosome 10 and chromosome 11
c) Recessive; chromosome 21 or chromosome 22
d) Recessive; chromosome 21 or chromosome 9
Correct Answer: A
Your Answer: Unanswered
Explanation

Explanation

Gene in Alzheimers Chromosome

* APP gene 21

* Presenilin 1 14*****

* Presenilin 2 1

* APO ‘Episilon’ 19

* a2 macroglobulin 10

The Molecular Basis for Degenerat ive Dementia

Molecular Susceptibility Pathologic


Dementia Causal Genes (Chromosome)
Basis Genes Findings

APP (21)******, PS-1 (14), PS-2 (1) Amyloid plaques,


(<2% carry neurofibrillary
AD Aβ/tau Apo ε 4 (19)****** tangles, and
these mutations, most often in PS-
1) neuropil threads

POSITIVE MED PG
Web: http://www.positivemedpg.com/ Score:
Email: manohrajan@yahoo.co.in 0.00 / 100
Page 131
NIMHANS - DM Neurology -
Exam Title :
MO...
Email : shubham.saha452@gmail.com
Contact : 9932160514

Tau neuronal and


MAPT exon and intron mutations glial inclusions
(17)
FTD Tau H1 MAPT haplotype varying in
(about 10% of familial cases) morphology and
distribution

GRN (10% of familial cases), C9O TDP-43 neuronal


RF72 and glial
inclusions varying
TDP-43 (20–30% of familial cases), rare V in
CP , very
morphology and
rare TARDBP TBK1, TIA1 distribution

FUS neuronal and


glial inclusions
varying in
FUS Very rare FUS
morphology and
distribution

α-Synuclein
α- neuronal
DLB Very rare SNCA (4) Unknown
Synuclein inclusions (Lewy
bodies)

Codon 129
PRNP (20) (up to 15% of patients homozygosity PrP SC deposition,
CJD PrP SC carry panlaminar
for methionine or spongiosiS
valine

QUESTION 72. NDE3MCtTSFVCSEFNICBTQUhBK3NodWJoYW0uc2FoYTQ1MkBnbWFpbC5jb20rOTkzMjE2M


USU9OIDcx
A patient was admitted with complaints of pain in the calf muscles which is releived by
movement.A provisional diagnosis of Restless syndrome was made.Restless leg syndrome is
characterized by all of the following except

a) Autosomal reccesive inheritance


b) More common in women
c) Also called as Willis-Ekbom Disease
d) Rotigotine is used for treatment
Correct Answer: A
Your Answer: Unanswered
Explanation

POSITIVE MED PG
Web: http://www.positivemedpg.com/ Score:
Email: manohrajan@yahoo.co.in 0.00 / 100
Page 132
NIMHANS - DM Neurology -
Exam Title :
MO...
Email : shubham.saha452@gmail.com
Contact : 9932160514

Explanation

*Restless legs syndrome, or Willis-Ekbom Disease, is a common, chronic, multifactorial,


movement disorder of the limbs in which patients have an irresistible urge to move the legs.

*There is a diurnal pattern of worsened symptoms at night.

*There are two type of Restless Legs Syndrome. Primary Restless Legs Syndrome and
Secondary Restless Legs Syndrome.

Secondary Restless Legs Syndrome can occur secondary to some disorders including:

• Iron deficiency
• End Stage Renal Disease
• Pregnancy
• Diabetes Mellitus
• Rheumatic disease
• Venous insufficiency
• Peripheral neuropathy
• Folate or magnesium deficiency
• Amyloidosis
• Lumbosacral radiculopathy
• Fibromyalgia
• Celiac disease
• Medications have been known to cause or exacerbate the symptoms of Restless Legs
Syndrome. They include antidopaminergic medications (e.g., neuroleptics),
Diphenhydramine, Tricyclic antidepressants (TCAs), Selective serotonin reuptake
inhibitors (SSRIs).

*Women are affected more than men******. . It is threefold more common in pregnant women

* There may be an autosomal dominant inheritance******

*Polymorphisms in genes including BTBD9 and MEIS1 are associated with Restless Legs
Syndrome.

A variety of factors can cause RLS. Iron deficiency is the most common treatable cause, and
iron replacement should be considered if the ferritin level is <75 ng/mL-20 th . Opioids and
benzodiazepines may also be of therapeutic value. Most patients with restless legs also
experience PLMD, although the reverse is not the case.

Medications used

• Dopaminergic agents (eg, pramipexole, ropinirole, bromocriptine, levodopa-carbidopa,


and rotigotine )
• Benzodiazepines (eg, clonazepam)
• Opioids (eg, codeine)
• Anticonvulsants (eg, gabapentin and pregabalin)
• Presynaptic alpha 2 -adrenergic agonists (eg, clonidine)
• Iron salt

POSITIVE MED PG
Web: http://www.positivemedpg.com/ Score:
Email: manohrajan@yahoo.co.in 0.00 / 100
Page 133
NIMHANS - DM Neurology -
Exam Title :
MO...
Email : shubham.saha452@gmail.com
Contact : 9932160514
QUESTION 73. NDE3MCtTSFVCSEFNICBTQUhBK3NodWJoYW0uc2FoYTQ1MkBnbWFpbC5jb20rOTkzMjE2M
NUSU9OIDcy
A 25 year old lady presented with altered sensorium,hearing loss and acute onset of visual
loss.Her MRI revealed “snowball” lesions in the corpus callosum.Which of the following could
be the possible diagnosis

a) Susac syndrome
b) Multiple sclerosis
c) SLE
d) APLA
Correct Answer: A
Your Answer: Unanswered
Explanation

Explanation

* The possibility of Susac syndrome is raised when a patient presents with one or more
components of the clinical triad (encephalopathy, BRAO, hearing loss) .

*A diagnosis of definite or probable Susac syndrome is based on presence of at least two


components of the triad and documentation of typical “snowball” lesions in the corpus callosum
upon brain MRI. ******

*Susac syndrome is characterized by three main problems: encephalopathy, partial or complete


occlusion of branch retinal artery occlusion, or BRAO and hearing loss

* Susac syndrome primarily affects young women between the ages of 20-40,

*MRI of the brain typically reveals “snowball” lesions in the corpus callosum. All forms require
immunosuppressive treatment while the disease is active.

MORE FACTS

Retinal arterial occlusion also occurs rarely in association with retinal migraine, lupus
erythematosus, anticardiolipin antibodies,

anticoagulant deficiency states (protein S, protein C, and antithrombin deficiency), Susac’s


syndrome - 20 th , pregnancy, IV drug abuse, blood

dyscrasias, dysproteinemias, and temporal arteritis

Anterior Ischemic Optic Neuropathy (AION) This is caused by insufficient blood flow
through the posterior ciliary arteries that supply the optic disc. It produces painless monocular
visual loss that is sudden in onset, followed sometimes by stuttering progression. The optic disc
appears swollen and surrounded by nerve fiber layer splinter hemorrhages AION is divided into
two forms: arteritic and nonarteritic.

The nonarteritic form is most common. .

It is urgent to recognize arteritic AION so that high doses of glucocorticoids can be instituted
immediately to prevent blindness in the second eye. Tocilizumab is an effective alternative to
glucocorticoids for sustained suppression of symptoms of giant cellArteritis-20 th

POSITIVE MED PG
Web: http://www.positivemedpg.com/ Score:
Email: manohrajan@yahoo.co.in 0.00 / 100
Page 134
NIMHANS - DM Neurology -
Exam Title :
MO...
Email : shubham.saha452@gmail.com
Contact : 9932160514

Metallosis (chromium, cobalt, nickel) from hip implant failure is a rare cause of toxic
optic neuropathy. Deficiency states induced by starvation, malabsorption, or
alcoholism can lead to insidious visual loss.20 th

Optic disc drusen are calcified, mulberry-like deposits of unknown etiology within the
optic disc, giving rise to “pseudopapilledema

QUESTION 74. NDE3MCtTSFVCSEFNICBTQUhBK3NodWJoYW0uc2FoYTQ1MkBnbWFpbC5jb20rOTkzMjE2M


NUSU9OIDcz
Which of the following diseases will NOT cause Proximal Arm/Distal Leg Weakness
(Scapuloperoneal or Humeroperonal) Weakness

a) Emery-Dreifuss muscular dystrophy


b) Inclusion body myositis
c) Facioscapulohumeral muscular dystrophy
d) Ducheene muscular dystrophy
Correct Answer: D
Your Answer: Unanswered
Explanation

Proximal Arm/Distal Leg Weakness (Scapuloperoneal or Humeroperonal)


Weakness

Facioscapulohumeral muscular dystrophy (FSHD)

Scapuloperoneal myopathy and neuropathy

Myofibrillar myopathies

Emery-Dreifuss muscular dystrophy (EDMD)

Bethlem myopathy

Distal Arm/Proximal Leg Weakness

Inclusion body myositis (usually wrist and finger flexors in arms, hip flexors and knee
extensors in legs, and asymmetric)

Myotonic dystrophy (uncommon presentation)

MORE FACTS

Muscle Sti ff ness/Decreased Ability To Relax

Myotonic dystrophy 1 and 2

Myotonia congenita

Paramyotonia congenita

POSITIVE MED PG
Web: http://www.positivemedpg.com/ Score:
Email: manohrajan@yahoo.co.in 0.00 / 100
Page 135
NIMHANS - DM Neurology -
Exam Title :
MO...
Email : shubham.saha452@gmail.com
Contact : 9932160514

Hyperkalemic periodic paralysis with myotonia

Potassium aggravated myotonia

Schwartz-Jampel syndrome

Other: rippling muscle disease (acquired and hereditary), acquired neuromyotonia (Isaacs
syndrome), sti ff -person syndrome, Brody disease

Brody myopathy is a hereditary condition that affects the skeletal muscles (muscles used for
movement). Symptoms typically begin in childhood and are characterized by muscle cramping
and stiffening (myopathy) after exercise or other strenuous activity. These symptoms can
worsen in cold temperatures and are usually painless, however, some individuals may have mild
discomfort. Some cases of Brody myopathy are caused by mutations in the ATP2A1 gene

Drug-Induced Myopathies

Drugs Major Toxic Reaction

Lipid-lowering agents Drugs belonging to all three of the

Fibric acid derivatives major classes of lipid-lowering agents

HMG-CoA reductase inhibitors can produce a spectrum of toxicity:

Niacin (nicotinic acid) asymptomatic serum creatine kinase

elevation, myalgias, exercise-induced

pain, rhabdomyolysis, and myoglobinuria.

Glucocorticoids Acute, high-dose glucocorticoid

treatment can cause acute quadriplegic

myopathy. These high doses

of steroids are often combined with

nondepolarizing neuromuscular

blocking agents but the weakness

can occur without their use. Chronic

steroid administration produces


predominantly

proximal weakness.

POSITIVE MED PG
Web: http://www.positivemedpg.com/ Score:
Email: manohrajan@yahoo.co.in 0.00 / 100
Page 136
NIMHANS - DM Neurology -
Exam Title :
MO...
Email : shubham.saha452@gmail.com
Contact : 9932160514

Nondepolarizing Acute quadriplegic myopathy can


neuromuscular
occur with or without concomitant
blocking agents
glucocorticoids.

Zidovudine Mitochondrial myopathy with ragged

red fibers

Drugs of abuse All drugs in this group can lead to

Alcohol widespread muscle breakdown,


rhabdomyolysis,
Amphetamines
and myoglobinuria.
Cocaine
Local injections cause muscle
Heroin
necrosis, skin induration, and limb
Phencyclidine
contractures.
Meperidine

Autoimmune toxic myopathy Use of this drug may cause polymyositis

d-Penicillamine and myasthenia gravis.

Amphophilic cationic drugs All amphophilic drugs have the

Amiodarone potential to produce painless,

Chloroquine proximal weakness associated with

Hydroxychloroquine autophagic vacuoles in the muscle

biopsy.

Antimicrotubular drugs This drug produces painless, proximal

Colchicine weakness especially in the setting of

renal failure. Muscle biopsy shows

autophagic vacuoles.

. In myotonic muscular dystrophy type 1 (DM1), distal weakness usually accompanies myotonia,
whereas in DM2, proximal muscles are more affected. Myotonia also occurs with myotonia
congenita (a chloride channel disorder), but in this condition muscle weakness is not prominent.
Myotonia

POSITIVE MED PG
Web: http://www.positivemedpg.com/ Score:
Email: manohrajan@yahoo.co.in 0.00 / 100
Page 137
NIMHANS - DM Neurology -
Exam Title :
MO...
Email : shubham.saha452@gmail.com
Contact : 9932160514

may also be seen in individuals with sodium channel mutations (hyperkalemic periodic paralysis
or potassium-sensitive myotonia). Another sodium channelopathy, paramyotonia congenita (PC),
also is associated with muscle stiffness. In contrast to other disorders associated with myotonia
in which the myotonia is eased by repetitive activity, PC is named for a paradoxical phenomenon
whereby the myotonia worsens with repetitive activity. Potassiumaggravated myotonia is an
allelic disorder in which myotonia is brought on by consumption of too much potassium-
containing foods

There are two painful muscle conditions of particular importance, neither of which is associated
with muscle weakness. Fibromyalgia is a common, yet poorly understood myofascial pain
syndrome in which patients complain of severe muscle pain and tenderness, severe fatigue, and
often poor sleep. Serum CK, erythrocyte sedimentation rate (ESR), EMG, and muscle biopsy are
normal . Polymyalgia rheumatica occurs mainly in patients aged >50 years and is characterized
by stiftness and pain in the shoulders, lower back, hips, and thighs . The ESR and CRP are
elevated, while serum CK, EMG, and muscle biopsy are normal .

QUESTION 75. NDE3MCtTSFVCSEFNICBTQUhBK3NodWJoYW0uc2FoYTQ1MkBnbWFpbC5jb20rOTkzMjE2M


USU9OIDc0
A patient presented with episodic weakness and was diagnosed to have Myesthenia
gravis.His serology revealed antiMusk antibodies.Which of the following is false
regarding a Myesthenic patient with antiMusk antibodies

a) typically do not experience myasthenic crisis


b) obtain less benefit from anticholinesterase agents than those with AChR antibodies and
may actually worsen
c) MuSK MG is more common in Asians
d) rituximab is not effective
Correct Answer: D
Your Answer: Unanswered
Explanation

Explanation

Myesthenia gravis

*Approximately 20% of patients with MG can be tapered off all immunotherapies

*There does not appear to be a correlation with maximal disease severity and chance for
remission.

*Thymectomy may increase the likelihood of achieving remission in anti-AChR MG, but the
large randomized

trial was too short in duration to examine this endpoint; rather, the results revealed only that
thymectomy

was efficacious and led to less use of glucocorticoids and second line agents. Mortality - 1–2%
range.

*Anti-MuSK patients typically do not experience myasthenic crises*******, but are generally
more difficult to treat than anti-AChR MG. Non-paraneoplastic LEMS is

POSITIVE MED PG
Web: http://www.positivemedpg.com/ Score:
Email: manohrajan@yahoo.co.in 0.00 / 100
Page 138
NIMHANS - DM Neurology -
Exam Title :
MO...
Email : shubham.saha452@gmail.com
Contact : 9932160514

usually responsive to immunotherapy and symptomatic treatment with pyridostigmine and 3,4
DAP.

* Juvenile onset MG is uncommon in Western populations but may represent more than half of
cases in Asians. MuSK MG appearts to be more common in the Mediterranean area of Europe
than in northern Europe and is also more common in the northern regions of East Asia ******

* There is a growing body of evidence that rituximab is effective in many MG patients,


especially those with MuSK antibody.****

*Patients with anti-MuSK MG generally obtain less benefit from anticholinesterase agents than
those with AChR antibodies and may actually worsen******

Until recently there was a debate regarding the role of thymectomy in non-thymotous MG, but a
recent large international trial of extended transternal thymectomy in non-thymomatous AChR
antibody positive, generalized MG demonstrated that participants who underwent thymectomy
had improved strength and function, required less prednisone and additions of second line
agents (e.g., azathioprine), and fewer hospitalizations for exacerbations

QUESTION 76. NDE3MCtTSFVCSEFNICBTQUhBK3NodWJoYW0uc2FoYTQ1MkBnbWFpbC5jb20rOTkzMjE2M


USU9OIDc1
A patient presented with reccurent attacks of optic neuritis and was diagnosed to have
multiple sclerosis.All of the following drugs are considered as highly effective in the
management of Multiple sclerosis except

a) Natalizumab
b) Fingolimod
c) Ocrelizumab
d) Mitoxantrone Hydrochloride
Correct Answer: B
Your Answer: Unanswered
Explanation

Drugs used in Multiple sclerosis

Highly effective Moderately effective

Natalizumab Fingolimod

Ocrelizumab Interferon β

Alemtuzumab Glatiramer Acetate

Mitoxantrone Hydrochloride Dimethyl Fumarate (DMF)

POSITIVE MED PG
Web: http://www.positivemedpg.com/ Score:
Email: manohrajan@yahoo.co.in 0.00 / 100
Page 139
NIMHANS - DM Neurology -
Exam Title :
MO...
Email : shubham.saha452@gmail.com
Contact : 9932160514

Teriflunomide

Daclizumab, a monoclonal antibody against CD25, the α subunit of the interleukin 2 receptor,
was removed from the market in 2018 because of reports of

brain inflammation; it was previously approved for patients who had failed at least two
other therapies

MORE FACTS

Drugs used in Multiple sclerosis

Intravenous I ntramuscular Subcutaneous Oral

Ocrelizumab Interferon 1-Beta Glatrimer acetate Dimethyl fumarate

Natalizumab Fingolimod

Teriflunomid

With three or moretypical T2-weighted lesions, the risk of developing MS after 20 years is
~80%. Conversely, with a normal brain MRI, the likelihood of developing MS is

<20%. Similarly, the presence of two or more Gd-enhancing lesions at baseline is highly
predictive of future MS, as is the appearance of either new T2-

weighted lesions or new Gd enhancement ≥3 months after the initial episode.

QUESTION 77. NDE3MCtTSFVCSEFNICBTQUhBK3NodWJoYW0uc2FoYTQ1MkBnbWFpbC5jb20rOTkzMjE2M


USU9OIDc2
Which of the following is incorrect regarding the drugs used in Multiple sclerosis and the
adverse effects

a) Fingolimod - QT prolongation
b) Natalizumab- PML
c) Ocrelizumab-Ca breast
d) Glatimer acetate-toxic epidermal necrolysis
Correct Answer: D
Your Answer: Unanswered
Explanation

Teriflunomide causes toxic epidermal necrolysis or Stevens-Johnson syndrome*****

Fingolimod (Moderately E ff ective)

POSITIVE MED PG
Web: http://www.positivemedpg.com/ Score:
Email: manohrajan@yahoo.co.in 0.00 / 100
Page 140
NIMHANS - DM Neurology -
Exam Title :
MO...
Email : shubham.saha452@gmail.com
Contact : 9932160514

*Fingolimod is a sphingosine-1-phosphate (S1P) inhibitor that prevents the egress of


lymphocytes from secondary lymphoid organs such as the lymph

nodes and spleen. Its mechanism of action is probably due to sequestration of lymphocytes in
the periphery, thereby inhibiting their trafficking to the

CNS.

*It is well tolerated, oraly

*First- and second-degree heart block and bradycardia can also occur . A 6-h period of
observation (including electrocardiogram

monitoring) is recommended for all patients receiving their first dose. Other side effects include
macular edema and, rarely, disseminated varicella-zoster

virus (VZV) and cryptococcal infections . Fingolimod can also cause QT prolongation .

DMF is a small molecule and is a Krebs cycle metabolite with anti-inflammatory effects in
psoriasis. Although the precise mechanisms of action of DMF

DMF is a small molecule and is a Krebs cycle metabolite with anti-inflammatory effects in
psoriasis. Although the precise mechanisms of action of DMF

are not fully understood, it seems to modulate the expression of proinflammatory and
anti-inflammatory cytokines

Glatiramer Acetate (Modestly E ff ective)

*Glatiramer acetate is a synthetic, random polypeptide composed of four amino acids (L-
glutamic acid, L-lysine, L-alanine, and L-tyrosine).

*Its mechanism of action may include (1) induction of antigen-specific suppressor T cells; (2)
binding to MHC molecules, thereby displacing bound MBP; or (3) altering the

balance between proinflammatory and regulatory cytokines. Glatiramer acetate reduces the
attack rate (whether measured clinically or by MRI) in RMS.

*glatiramer acetate should be considered as an equally effective alternative to IFN-β in RMS


patients. Its usefulness in

progressive disease is unknown.

*Glatiramer acetate is administered by subcutaneous injection**** .

Mitoxantrone Hydrochloride (Highly Effective)

*Mitoxantrone, an anthracenedione, exerts its antineoplastic action by (1) intercalating into


DNA and producing both strand breaks and interstrand crosslinks,

(2) interfering with RNA synthesis, and (3) inhibiting topoisomerase II (involved in DNA repair).
The FDA approved mitoxantrone .

* Mitoxantrone is indicated for use in patients with rapidly worsening

POSITIVE MED PG
Web: http://www.positivemedpg.com/ Score:
Email: manohrajan@yahoo.co.in 0.00 / 100
Page 141
NIMHANS - DM Neurology -
Exam Title :
MO...
Email : shubham.saha452@gmail.com
Contact : 9932160514

MS (defined as patients whose neurologic status remains significantly abnormal between MS


attacks). *

*Mitoxantrone is cardiotoxic . As a result, a

cumulative dose >140 mg/m2 is not recommended*****.

* 12 mg/m2 every 3 months

Daclizumab, a monoclonal antibody against CD25, the α subunit of the interleukin 2 receptor,
was removed from the market in 2018 because of reports of

brain inflammation; it was previously approved for patients who had failed at least two
other therapies

Alemtuzumab (Highly Effective)

*Alemtuzumab is a humanized monoclonal antibody directed against the CD52 antigen that is
expressed on both monocytes and lymphocytes. It causes

lymphocyte depletion (of both B and T cells) and a change in the composition of lymphocyte
subsets.4170

*Because of its toxicity profile, the FDA indicated

alemtuzumab only in patients who have tried and failed at least two other DMTs.

Natalizumab (Highly Effective)

*Natalizumab is a humanized monoclonal antibody directed against the α4 subunit of


α4β1 integrin, a cellular adhesion molecule expressed on the surface

of lymphocytes. It prevents lymphocytes from binding to endothelial cells, thereby preventing


lymphocytes from penetrating the BBB and entering the

CNS. Natalizumab is highly effective in reducing the attack rate and significantly improves all
measures of disease severity in MS (both clinical and MRI).

*Monthly intravenous infusions makes it very convenient for patients.

Treatment with natalizumab is, in general, well tolerated. A small percentage (<10%) of
patients experience

hypersensitivity reactions (including anaphylaxis), and ~6% develop neutralizing antibodies to


the molecule (only half of which persist).

*The major concern with long-term treatment is the risk of PML, a life-threatening condition
resulting from infection by the John Cunningham (JC) virus.

*PML has occurred in ~0.4% of patients treated with natalizumab.

Natalizumab is generally recommended only for JC antibody–negative patients,

*Ocrelizumab (Highly Effective)

*Ocrelizumab is a humanized monoclonal antibody directed against the CD20 molecule present
on the surface of mature B cells. CD20 is not expressed on

POSITIVE MED PG
Web: http://www.positivemedpg.com/ Score:
Email: manohrajan@yahoo.co.in 0.00 / 100
Page 142
NIMHANS - DM Neurology -
Exam Title :
MO...
Email : shubham.saha452@gmail.com
Contact : 9932160514

early B-cell precursors or on antibody-producing plasma cells, thus treatment with ocrelizumab
selectively depletes mature B cells while preserving

preexisting humoral immunity and the capacity for B-cell reconstitution by lymphoid stem cells.
*Ocrelizumab rapidly depletes circulating B cells through

antibody-dependent cellular toxicity and complement-dependent cytotoxicity.

*Ocrelizumab targets the same molecule as rituximab, a monoclonal antibody indicated for non-
Hodgkin’s lymphoma and

rheumatoid arthritis, and ofatumumab, indicated for treatment of chronic lymphocytic


leukemia.

* Ocrelizumab 600 mg is administered byintravenous infusion every 24 weeks

*Ocrelizumab is well tolerated with infusion-related reactions

* Rituximab is associated with a very small risk of PML (estimated at <1:25,000/year), thus it is
possible that

ocrelizumab will also carry a nonzero risk. Ocrelizumab may also carry some risk of increased
malignancies including breast cancer, although rituximab is

not associated with an increased risk of malignancy.

Teriflunomide (Modestly E ff ective)

*Teriflunomide inhibits the mitochondrial enzyme dihydro-orotate dehydrogenase, which is a


key part of the pathway for de novo pyrimidine biosynthesis

from carbamoyl phosphate and aspartate. It is the active metabolite of the drug leflunomide
(FDA-approved for rheumatoid arthritis), and it exerts its

anti-inflammatory effects by limiting the proliferation of rapidly dividing T and B cells.

*Consequently, teriflunomide is considered to be cytostatic rather than cytotoxic. Teriflunomide


reduces the attack rate and significantly improves all measures of disease severity in MS

patients.

*It is well tolerated, and its daily oral dosing *****

*treatment with teriflunomide was well tolerated. Teriflunomide rarely causes toxic epidermal
necrolysis or Stevens-Johnson syndrome. *****

*pregnancy category X

* teriflunomide can remain in the bloodstream for 2 years due tohepatobiliary reabsorption.

QUESTION 78. NDE3MCtTSFVCSEFNICBTQUhBK3NodWJoYW0uc2FoYTQ1MkBnbWFpbC5jb20rOTkzMjE2M


USU9OIDc3
Motor Nerve Conduction Studies in an axonal degeneration will show all the following except

POSITIVE MED PG
Web: http://www.positivemedpg.com/ Score:
Email: manohrajan@yahoo.co.in 0.00 / 100
Page 143
NIMHANS - DM Neurology -
Exam Title :
MO...
Email : shubham.saha452@gmail.com
Contact : 9932160514

a) F wave can be absent


b) Conduction block is absent
c) Distal latency is normal
d) Conduction velocity is markedly prolonged
Correct Answer: D
Your Answer: Unanswered
Explanation

El ectrophysiologic Features: Axonal Degeneration Versus Segmental Demyelination

Axonal Segmental

Degeneration Demyelination

Motor Nerve Conduction Studies

Normal (except with CB


CMAP amplitude Decreased
or distal dispersion)

Distal latency Normal Prolonged

Conduction velocity Normal Slow

Conduction block Absent Present

Temporal dispersion Absent Present

F wave Normal or absent Prolonged or absent

H reflex Normal or absent Prolonged or absent

Sensory Nerve Conduction Studies

SNAP amplitude Decreased Normal or decreased

Distal latency Normal Prolonged

Conduction velocity Normal Slow

POSITIVE MED PG
Web: http://www.positivemedpg.com/ Score:
Email: manohrajan@yahoo.co.in 0.00 / 100
Page 144
NIMHANS - DM Neurology -
Exam Title :
MO...
Email : shubham.saha452@gmail.com
Contact : 9932160514

Needle EMG

Spontaneous activity

Fibrillations Present Absent

Fasciculations Present Absent

Motor unit potentials

Recruitment Decreased Decreased

Long duration/
Morphology Normal
polyphasic

In general, low-amplitude potentials with relatively preserved distal latencies, conduction


velocities, and late potentials, along with fibrillations on needle EMG, suggest an axonal
neuropathy. On the other hand, slow conduction velocities, prolonged distal latencies and late
potentials, relatively preserved amplitudes, and the absence of fibrillations on needle EMG
imply a primary demyelinating neuropathy. The presence of nonuniform slowing of conduction
velocity, conduction block, or temporal dispersion further suggests an acquired demyelinating
neuropathy (e.g., GBS or CIDP) as opposed to a hereditary demyelinating neuropathy (e.g.,
CMT type 1).

QUESTION 79. NDE3MCtTSFVCSEFNICBTQUhBK3NodWJoYW0uc2FoYTQ1MkBnbWFpbC5jb20rOTkzMjE2M


USU9OIDc4
A patient was suspected to have porphyria.Which of the following porphyria is not associated
with peripheral neuropathy

a) acute intermittent porphyria


b) hereditary coproporphyria
c) variegate porphyria
d) Erythropoitic porphyria
Correct Answer: D
Your Answer: Unanswered
Explanation

Porphyria is a group of inherited disorders caused by defects in heme biosynthesis Three forms
of porphyria are associated with peripheral

neuropathy: acute intermittent porphyria (AIP), hereditary coproporphyria (HCP), and


variegate porphyria (VP).*******

POSITIVE MED PG
Web: http://www.positivemedpg.com/ Score:
Email: manohrajan@yahoo.co.in 0.00 / 100
Page 145
NIMHANS - DM Neurology -
Exam Title :
MO...
Email : shubham.saha452@gmail.com
Contact : 9932160514

Rare Hereditary Neuropathies

Hereditary Disorders of Lipid Metabolism

Metachromatic leukodystrophy

Krabbe’s disease (globoid cell leukodystrophy)

Fabry’s disease

Adrenoleukodystrophy/adrenomyeloneuropathy

Refsum’s disease

Tangier disease

Cerebrotendinous xanthomatosis

Hereditary Ataxias with Neuropathy

Friedreich’s ataxia

Vitamin E deficiency

Spinocerebellar ataxia

Abetalipoproteinemia (Bassen-Kornzweig disease)

Disorders of Defective DNA Repair

Ataxia-telangiectasia

Cockayne’s syndrome

Giant Axonal Neuropathy

Porphyria

Acute intermittent porphyria (AIP)

Hereditary coproporphyria (HCP)

Variegate porphyria (VP)

POSITIVE MED PG
Web: http://www.positivemedpg.com/ Score:
Email: manohrajan@yahoo.co.in 0.00 / 100
Page 146
NIMHANS - DM Neurology -
Exam Title :
MO...
Email : shubham.saha452@gmail.com
Contact : 9932160514

Familial Amyloid Polyneuropathy (FAP)

Transthyretin-related

Gelsolin-related

Apolipoprotein A1-related

MORE FACTS

Adrenomyeloneuropathy

* It is an X - linked dominant disorders caused by mutations in adenosine triphosphate -


binding cassette (ABC) transporter gene. It is a primary axonopathy with secondary
demyelination. Nerve biopsies demonstrate lamellar inclusions in the cytoplasm of Schwann
cells. Very long chain fatty acid (VLCFA) levels (C24, C25, and C26) are increased in the
urine. Adrenal insufficiency is present in two - thirds of patients. Diets low in VLCFAs and
supplemented with Lorenzo's oil (erucic and oleic acids) reduce the levels of VLCFAs and
increase the levels of C22.

Refsum Disease

* The tetrad is (1) peripheral neuropathy, (2) retinitis pigmentosa [AIIMS NOV - 2010***], (3)
cerebellar ataxia, and (4) elevated CSF protein concentration. Other associations are
sensorineural hearing loss, cardiac arrythmias, ichthyosis, and anosmia. It is an autosomal
recessive disease due to mutations in the gene that encodes for phytanoyl - CoA alfa –
hydroxylase [AIIMS NOV - 2008***] (PAHX) or peroxin 7 receptor protein (PRX 7) - serum
phytanic acid levels are elevated. Nerve biopsy demonstrates has onion bulb formation. It is
treated by avoiding phytanic precursors (phytols: fish oils, dairy products) from the diet.

Tangier Disease

* It is due to mutations(AR) in the ATP - binding cassette transporter 1 (ABC1) gene, which
leads to decreased levels of high - density lipoprotein (HDL) cholesterol. Clinical
manifestations are asymmetric multiple mononeuropathies, a slowly progressive symmetric
polyneuropathy predominantly in the legs, a pseudo - syringomyelia pattern with dissociated
sensory loss (abnormal pain/temperature perception but preserved position/vibration in the
arms. There is yellowish - orange tonsil, with splenomegaly and lymphadenopathy. Biochemical
abnormality is increased triacylglycerol levels. Nerve biopsies reveal abnormal accumulation of
lipid in Schwann cells..

Porphyria(AD)

* Three forms of porphyria are associated with peripheral neuropathy: acute


intermittent porphyria (AIP), hereditary coproporphyria (HCP), and variegate
porphyria (VP).

QUESTION 80. NDE3MCtTSFVCSEFNICBTQUhBK3NodWJoYW0uc2FoYTQ1MkBnbWFpbC5jb20rOTkzMjE2M


USU9OIDc5

POSITIVE MED PG
Web: http://www.positivemedpg.com/ Score:
Email: manohrajan@yahoo.co.in 0.00 / 100
Page 147
NIMHANS - DM Neurology -
Exam Title :
MO...
Email : shubham.saha452@gmail.com
Contact : 9932160514

Immune-mediated brachial plexus neuropathy (IBPN) or Parsonage-Turner syndrome is


associated with all of the following except

a) usually presents with an acute onset of severe pain in the shoulder region
b) weakness and often sensory loss occurs
c) primarily involves the suprascapular, long thoracic, or axillary nerves
d) It commonly affects the lower trunk of brachial plexsus
Correct Answer: D
Your Answer: Unanswered
Explanation

Explanation

Immune-mediated brachial plexus neuropathy (IBPN) goes by various terms, including acute
brachial plexitis, neuralgic amyotrophy, and Parsonage-

Turner syndrome. IBPN usually presents with an acute onset of severe pain in the shoulder
region.

However, as the pain dissipates, weakness and often sensory loss are appreciated..

* The most common pattern of

IBPN involves the upper trunk***** or a single or multiple mononeuropathies primarily


involving the suprascapular, long thoracic, or axillary nerves.

Additionally, the phrenic and anterior interosseous nerves may be concomitantly affected

MORE FACTS

* Brachial plexopathies occur in 5% of patients following a median sternotomy and typically


affect the lower trunk*****

Approximately 25% of affected patients have continued pain (postherpetic neuralgia

Standard doses of INH (3–5 mg/kg per day) are associated with a 2%

incidence of neuropathy, whereas neuropathy develops in at least 17% of patients taking in


excess of 6 mg/kg per d. The elderly, malnourished, and “slow

acetylators” are at increased risk for developing the neuropathy. INH inhibits pyridoxal
phosphokinase, resulting in pyridoxine deficiency and the

neuropathy.

[PHN]).

The nucleoside analogues zalcitabine (dideoxycytidine or ddC), didanosine (dideoxyinosine or


ddI), stavudine (d4T), lamivudine (3TC), and antiretroviral

nucleoside reverse transcriptase inhibitor (NRTI) are used to treat HIV infection. One of the
major dose-limiting side effects of these medications is a

predominantly sensory, length-dependent, symmetrically painful neuropathy

The predominant motor neuropathies are listed below

POSITIVE MED PG
Web: http://www.positivemedpg.com/ Score:
Email: manohrajan@yahoo.co.in 0.00 / 100
Page 148
NIMHANS - DM Neurology -
Exam Title :
MO...
Email : shubham.saha452@gmail.com
Contact : 9932160514

Motor - predominant neuropathies

Immune neuropathies: acute (Guillain - Barré syndrome); relapsing

Heritable motor - sensory neuropathies

Acute intermittent porphyria

Diphtheritic neuropathy

Lead neuropathy [AIIMS NOV - 2006***]

Brachial neuritis

Diabetic lumbosacralplexus neuropathy (diabetic amyotrophy)

Toxic Neuropathies Secondary to Chemotherapy

Mechanism of Clinical Nerve


Drug EMG/NCS
Neurotoxicity Features Histopathology

Symmetric, S-M,
Axonal degeneration Axonal
large-/
Interfere with of sensorimotor PN;
Vinca alkaloids distal
axonal small-fiber PN;
(vincristine, myelinated and
autonomic
microtubule unmyelinated denervation on
vinblastine, EMG; abnormal
assembly; symptoms
vindesine, fibers; regenerating
common;
impairs axonal clusters, QST, particularly
vinorelbine) vibratory
transport infrequent cranial
minimal segmental
neuropathies demyelination Perception

POSITIVE MED PG
Web: http://www.positivemedpg.com/ Score:
Email: manohrajan@yahoo.co.in 0.00 / 100
Page 149
NIMHANS - DM Neurology -
Exam Title :
MO...
Email : shubham.saha452@gmail.com
Contact : 9932160514

Loss of large >


Preferential small myelinated
Low-amplitude or
damage to dorsal unobtainable
and unmyelinated
root ganglia: Predominant fibers;
SNAPs with
large-fiber
? binds to and axonal degeneration normal
Cisplatin cross-links DNA sensory with
CMAPs and EMG;
neuronopathy; abnormal
? inhibits protein small clusters of
synthesis sensory ataxia regenerating
QST, particularly
fibers; secondary vibratory
? impairs axonal
transport segmental
Perception
demyelination

Loss of large >


small myelinated
Symmetric, Axonal
and unmyelinated
Promotes axonal predominantly sensorimotor PN;
fibers;
microtubule distal
sensory PN;
Taxanes axonal degeneration
assembly; large-fiber denervation on
(paclitaxel, with
interferes with EMG; abnormal
modalities
docetaxel) axonal small clusters of
affected more QST, particularly
regenerating
transport than vibratory
fibers; secondary
small-fiber Perception
segmental

demyelination

Abnormalities
consistent with

Length- an axonal sensory


dependent, neuropathy
Bortezomib sensory,
Unknown Not reported with early small-
(Velcade)
predominantly fiber involvement
small-fiber PN
(abnormal
autonomic

studies

QUESTION 81. NDE3MCtTSFVCSEFNICBTQUhBK3NodWJoYW0uc2FoYTQ1MkBnbWFpbC5jb20rOTkzMjE2M


USU9OIDgw

POSITIVE MED PG
Web: http://www.positivemedpg.com/ Score:
Email: manohrajan@yahoo.co.in 0.00 / 100
Page 150
NIMHANS - DM Neurology -
Exam Title :
MO...
Email : shubham.saha452@gmail.com
Contact : 9932160514

A patient presented with recurrent episodes of optic neuritis and bladder incontinence and was
diagnosed to have multiple sclerosis.Which of the following is the commonest type of Multiple
sclerosis

a) Relapsing or bout onset MS (RMS)


b) Secondary progressive MS (SPMS)
c) Relapsing/remitting MS (RRMS)
d) Primary progressive MS (PPMS)
Correct Answer: A
Your Answer: Unanswered
Explanation

Explanation

Risk factors are Vitamin D deficiency,EBV exposure in childhood and cigarette smoking****

Three clinical types[19 th edition-4] of MS exist

1)Relapsing or bout onset MS (RMS) accounts for 90% of MS cases and is characterized by
discrete attacks of neurological dysfunction that generally

evolve over days to weeks (rarely over hours). . Between attacks, patients are neurologically
stable.

2. Secondary progressive MS (SPMS) always begins as RMS . . SPMS produces a greater


amount of fixed neurologic disability than RMS. For a

patient with RMS, the risk of developing SPMS is ~2% each year.

3. Primary progressive MS (PPMS) accounts for ~10% of cases. These patients do not
experience attacks but rather steadily decline in function from

disease onset . Compared to RMS, the sex distribution is more even, the disease begins later in
life (mean age ~40 years), and disability

develops faster (relative to the onset of the first clinical symptom).

19 TH EDITION

Four clinical types of MS are Relapsing/remitting MS (RRMS - 85%); Secondary progressive


MS (SPMS) always begins as RRMS - risk of developing SPMS is 2% each year;Primary
progressive MS (PPMS) has a steady functional decline, the sex incidence is equal and the
disease begins later in life; Progressive/relapsing MS (PRMS - 5%)

Risk of Developing Multiple Sclerosis (MS)

1 in 3 If an identical twin has MS

1 in 15 If a fraternal twin has MS

POSITIVE MED PG
Web: http://www.positivemedpg.com/ Score:
Email: manohrajan@yahoo.co.in 0.00 / 100
Page 151
NIMHANS - DM Neurology -
Exam Title :
MO...
Email : shubham.saha452@gmail.com
Contact : 9932160514

1 in 25 If a sibling has MS

1 in 50 If a parent or half-sibling has MS

1 in 100 If a first cousin has MS

1 in 1000 If a spouse has MS

1 in 1000 If no one in the family has M

QUESTION 82. NDE3MCtTSFVCSEFNICBTQUhBK3NodWJoYW0uc2FoYTQ1MkBnbWFpbC5jb20rOTkzMjE2M


USU9OIDgx
The most common nerve involvement in the peripheral neuropathy associated with Glycogen
storage disorder type III-GSDIII- Deficient debranching enzyme activity

a) Ulnar nerve
b) Sural nerve
c) Lateral cutaneous nerve of thigh
d) Median nerve
Correct Answer: D
Your Answer: Unanswered
Explanation

Explanation

GSD III-debrancher enzyme deficiency

*Patients with GSD III present with hepatomegaly, hypoglycemia, short stature, variable
skeletal myopathy, and cardiomyopathy.

*GSD type IIIa involves both liver and muscle. 15% of patients have only liverinvolvement and is
classified as type IIIb.

*Hypoglycemia and hyperlipidemia occur in children.

*Serum creatine kinase (CK) levels - normal levels do not rule outmuscle enzyme deficiency.

* In most patients with type III disease, hepatomegaly improves with age; however,liver fibrosis,
cirrhosis progressing to liver failure, and hepatocellular carcinoma, are noted in many in late
adulthood.*****

*Hepatic adenomas may occur, although less commonly than in GSD I. Left ventricular
hypertrophy, significant scarring of the myocardium, and life-threatening arrhythmias can occur

*The pattern of muscle weakness is variable and both proximal and distalmuscle weakness are
seen.

POSITIVE MED PG
Web: http://www.positivemedpg.com/ Score:
Email: manohrajan@yahoo.co.in 0.00 / 100
Page 152
NIMHANS - DM Neurology -
Exam Title :
MO...
Email : shubham.saha452@gmail.com
Contact : 9932160514

*Peripheral neuropathy may become discernible later in life with preferentialmedian nerve
involvement*******.-20 th

* Polycystic ovaries are common .

* Fertility is normal.

More FACTS

Type IX GSD (Liver Phosphorylase Kinase Deficiency)

* The most common subtype is GSD IXa, an X-linked liver PhK deficiency caused by mutations in
the PHKA2 gene, which is also one of the most common liver glycogenoses.

Typically, a child between the ages of 1 year and 5 years presents with

growth retardation and hepatomegaly.

Liver fibrosis has been identified in some patients, including children. Levels of cholesterol,
triglycerides, and liver enzymes are mildly elevated. Fasting ketosis is a feature of the disease.
Lactic and uric acid levels are usually normal. Hypoglycemia may be mild in some but severe
and recurrent in others[20 th ].

*Liver transplantation may be considered in those with severe hepatic involvement

*Other subtypes of type IX GSD include GSD IXb, an autosomal recessive form of liver and
muscle PhK deficiency caused by PHKB mutations.

* GSD IXc, an autosomal recessive form of liver PhK deficiency that often develops into liver
cirrhosis, is due to PHKG2 mutations.

*GSD IXd, a muscle-specific PhK deficiency that causes cramps and myoglobinuria with
exercise, is caused by PHKA1 mutations. The previous reports of cardiac-specific PhK deficiency
is now considered to be a secondary phenomenon, as these patients have mutations in
the PRKAG2 gene. Patients with cardiac PRKAG2 syndrome often present with cardiomyopathy
during infancy. The condition is lethal because of massive glycogen deposition in the
myocardium.

Type IV GSD (Branching enzyme deficiency, Amylopectinosis, Polyglucosan disease or Andersen


disease) is caused by deficiency of branching enzyme activity leading to accumulation of an
abnormal glycogen with poor solubility. The disease is clinically heterogeneous. Individuals
typically present in the first 18 months of life with failure to thrive, hepatosplenomegaly, and
progressive liver cirrhosis leading to death in early childhood. Some patients may develop
hepatic adenomas and hepatocellular carcinoma. GSD IV has extrahepatic manifestations
involving the central and peripheral nervous system as well as cardiac and skeletal muscles.
The adult form is known as adult polyglucosan body disease (APBD) and may present as an
isolated myopathy or with systemic involvement of the central and peripheral nervous system
characterized by neurogenic bladder, peripheral neuropathy, leukodystrophy, and mild cognitive
impairment. Definitive

diagnosis requires demonstration of branching enzyme deficiency in liver, muscle, cultured skin
fibroblasts or leukocytes, or genetic testing of the GBE1 gene. It is likely that life expectancy is
shortened in APBD

POSITIVE MED PG
Web: http://www.positivemedpg.com/ Score:
Email: manohrajan@yahoo.co.in 0.00 / 100
Page 153
NIMHANS - DM Neurology -
Exam Title :
MO...
Email : shubham.saha452@gmail.com
Contact : 9932160514

patients though it is yet to be confirmed by long-term natural history studies. Good supportive
care is crucial to improve clinical outcomes.

Liver transplantation may be performed for progressive hepatic failure.

pompes

The disorder encompasses a range of phenotypes. Each includes myopathy but differs in the
age of onset, extent of organ involvement, and clinical severity. The most severe is the infantile
th edition-1] years of age. Infants
form, with cardiomegaly, hypotonia, and death before 2 [19
often present with cardiomyopathy at birth, and develop

a generalized muscle weakness with feeding difficulties, macroglossia, hepatomegaly, and


congestive heart failure due to the rapidly progressive hypertrophic cardiomyopathy.

Adults typically present between the second and seventh decades with slowly progressive
myopathy without overt cardiac involvement. The clinical picture is dominated by slowly
progressive, predominantly proximal limb girdle muscle weakness. The pelvic girdle, paraspinal
muscles, and diaphragm are most seriously affected. Respiratory symptoms include
somnolence, morning headache, orthopnea, and exertional dyspnea. Respiratory failure causes
significant morbidity and mortality in the late-onset form. In rare instances, patients present
with respiratory insufficiency as the initial symptom. Basilar artery aneurysms and dilation of
the ascending aorta have been observed in patients with Pompe disease. Ptosis, lingual
weakness, gastrointestinal dysmotility, and incontinence due to poor sphincter tone are now
being recognized as part of the clinical spectrum. Small-fiber neuropathy, which presents with
painful paresthesia or pins-and-needles sensations, is also seen in some patients with LOPD.
Individuals with advanced disease often require some form of ventilatory support and are
dependent on a walking aid or wheelchair

DIAGNOSIS

The confirmatory step for a diagnosis of Pompe disease is enzyme assay demonstrating deficient
acid α- glucosidase or a gene sequence with two pathogenic mutations in the GAA gene.
Enzyme activity can be measured in muscle, cultured skin fibroblasts, or blood. The latter is
increasingly being used and is very reliable when performed in laboratories with
experience. Prenatal diagnosis using mutation analysis of DNA extracted from fetal cells
obtained by amniocentesis or by measuring GAA enzyme activity in chorionic villi or amniocytes
is available. Carrier detection and prenatal diagnosis, using DNA-based targeted mutation

analysis, are also possible if disease-causing family mutations are already known

The approval of enzyme replacement therapy with alglucosidase alfa in 2006 has changed the
natural history and clinical course of Pompe disease. Other adjunctive treatment options
include dietary modifications, submaximal aerobic exercise, and respiratory muscle strength
training. Gene therapy is under study as another treatment modality.

Pompe disease is now part of the recommended uniform screening panel

Late-Onset Polyglucosan Body Myopathy due to GYG1 Mutations

This is an autosomal recessive, slowly progressive skeletal myopathy caused by mutations in


the GYG1 gene blocking glycogenin-1 biosynthesis. GYG1 mutations results in a reduced or
complete absence of glyogenin-1 which is necessary for glycogen synthesis in muscles. Affected
individuals commonly present with adultonset proximal muscle weakness prominently affecting
the hip and shoulder girdles. Cardiomyopathy and cardiac failure necessitating cardiac

POSITIVE MED PG
Web: http://www.positivemedpg.com/ Score:
Email: manohrajan@yahoo.co.in 0.00 / 100
Page 154
NIMHANS - DM Neurology -
Exam Title :
MO...
Email : shubham.saha452@gmail.com
Contact : 9932160514
transplantation is seen. Compared to GSD IV APBD, nervous system involvement has not been
reported although both disorders cause polyglucosan deposition

QUESTION 83. NDE3MCtTSFVCSEFNICBTQUhBK3NodWJoYW0uc2FoYTQ1MkBnbWFpbC5jb20rOTkzMjE2M


USU9OIDgy
A 5 day old patient presented with refusal to eat and lethargy progressing to coma.On
evaluation there was acidosis, Increased plasma ammonia and Increased plasma
glutamine. Plasma citrulline is markedly increased.Which of the following enzyme
deficiency could have caused this disease

a) N-acetylglutamate synthase
b) carbamylphosphate synthase 1
c) ornithine transcarbamylase deficiency
d) argininosuccinic acid synthase
Correct Answer: D
Your Answer: Unanswered
Explanation

Explanation

UREA CYCLE DISORDERS

*Deficiencies in urea cycle enzymes are individually rare, but as a group, they affect about 1:3
5,000 [19 th edition- 1:25,000 ] individuals.

*They are all transmitted as autosomal recessive traits, with the exception of ornithine
transcarbamylase deficiency, which is X-linked and the most frequent urea cycle defect*****.

*Infants with classic urea cycle defects present at 1–4 days of life with refusal to eat and
lethargy progressing to coma and death. Milder enzyme deficiencies present with protein
avoidance, recurrent vomiting, migraine, mood swings, chronic fatigue, irritability, and
disorientation that can progress to coma.

*Administration of systemic corticosteroids can precipitate hyperammonemia and can be fatal


in previously asymptomatic individuals.

*The diagnosis requires measurement of plasma ammonia, plasma amino acids, and urine orotic
Acid

*Increased plasma glutamine is seen with all urea cycle defects since ammonia not removed by
the urea cycle in periportal hepatocytes is conjugated to glutamate by glutamine synthase in
perivenous hepatocytes.

* Citrulline is low or undetectable in proximal defects of the urea cycle (N-acetylglutamate


synthase, carbamylphosphate synthase 1, and ornithine transcarbamylase deficiency), with
urine orotic acid being increased only in ornithinetranscarbamylase deficiency.

*Plasma citrulline is markedly increased in argininosuccinic acid synthase deficiency


(citrullinemia type 1), with a milder elevation in argininosuccinic acid lyase deficiency in the
presence of argininosuccinic acid (argininosuccinic aciduria).*******

POSITIVE MED PG
Web: http://www.positivemedpg.com/ Score:
Email: manohrajan@yahoo.co.in 0.00 / 100
Page 155
NIMHANS - DM Neurology -
Exam Title :
MO...
Email : shubham.saha452@gmail.com
Contact : 9932160514

Inherited Disorders of Amino Acid Metabolism

Amino
Condition Enzyme Defect Clinical Findings Inheritance
Acid(s)

Dystonia,
DNAJC12 Hydroxylase Co- parkinsonism,
Phenylalanine AR
Deficiency Chaperone intellectual
disability

Intellectual disability,
microcephaly,
Phenylalanine hypopigmented
Phenylketonuria AR
hydroxylase
skin and hairs,
eczema, “mousy” odor

Liver failure, cirrhosis,


Tyrosinemia type rickets, failure to
I Fumarylacetoacetate thrive, peripheral
Tyrosine AR
hydrolase
(hepatorenal) neuropathy, “boiled
cabbage” odor

Palmoplantar
keratosis, painful
Tyrosinemia type
corneal erosions with
II Tyrosine transaminase AR
photophobia,
(oculocutaneous)
intellectual disability
(?)

Hypertyrosinemia with
4-
Tyrosinemia type Hydroxyphenylpyruvate normal liver function,
occasional AR
III
dioxygenase
mental delay

4-
Transient failure to
Hydroxyphenylpyruvate
Hawkinsinuria thrive, metabolic AD
dioxygenase acidosis in infancy

POSITIVE MED PG
Web: http://www.positivemedpg.com/ Score:
Email: manohrajan@yahoo.co.in 0.00 / 100
Page 156
NIMHANS - DM Neurology -
Exam Title :
MO...
Email : shubham.saha452@gmail.com
Contact : 9932160514

Ochronosis, arthritis,
Homogentisic acid cardiac valve
Alkaptonuria involvement, coronary AR
oxidase
artery calcification

Hypopigmentation of
Albinism hair, skin, and optic
Tyrosinase fundus; visual AR
(oculocutaneous)
loss; photophobia

Hypopigmentation of
Different enzymes or
Albinism (ocular) optic fundus, visual AR,XL
transporters
loss

Seizures,
ABAT deficiency GABA transaminase intellectual AR
disability, hypotonia

Mild intellectual
Prolidase
Prolidase disability, chronic AR
deficiency
dermatitis

Lens dislocation,
thrombotic vascular
Homocystine Homocystinuria Cystathionine β-synthase disease, intellectual AR

disability, osteoporosis

QUESTION 84. NDE3MCtTSFVCSEFNICBTQUhBK3NodWJoYW0uc2FoYTQ1MkBnbWFpbC5jb20rOTkzMjE2M


USU9OIDgz
Budapest consensus criteria is used for clinical diagnosis of which of the following
condition

a) COMPLEX REGIONAL PAIN SYNDROMES (CRPS)


b) Postprandial orthostatic hypotension
c) Postural Orthostatic Tachycardia Syndrome (POTS)
d) Guillian barrie syndrome
Correct Answer: A
Your Answer: Unanswered
Explanation

limb trauma, fractures, myocardial infarction, or stroke can cause CRPS type I*****

Reflex Sympathetic Dystrophy and Causalgia

POSITIVE MED PG
Web: http://www.positivemedpg.com/ Score:
Email: manohrajan@yahoo.co.in 0.00 / 100
Page 157
NIMHANS - DM Neurology -
Exam Title :
MO...
Email : shubham.saha452@gmail.com
Contact : 9932160514

* Complex regional pain syndrome (CRPS) types I and II are now called reflex sympathetic
dystrophy (RSD) and causalgia, respectively. CRPS type I(reflex sympathetic dystrophy (RSD)
(AI - 2011***) is a regional pain syndrome that develops after tissue trauma. Examples of
associated trauma include myocardial infarction, minor shoulder or limb injury, and stroke. All
odynia (is the perception of a nonpainful stimulus as painful), hyperpathia (is an exaggerated
pain response to a painful stimulus), and spontaneous pain can occur. CRPS type II
(causalgia) is a regional pain syndrome that develops after injury to a specific peripheral
nerve. CRPS type I (RSD) has three clinical phases. Medicines used in management are
adrenergic blockers, nonsteroidal anti - inflammatory drugs, calcium channel blockers,
phenytoin, opioids, and calcitonin. Stellate ganglion gives temporary pain relief. Symptomati
c Treatment is with Midodrine, a directly acting alfa 1 - agonist that does not cross the blood
- brain barrier. It has a duration of action of 2–4 h.

MORE FACTS-20 th

*Although CRPS type I (RSD) has been classically divided into three

clinical phases, there is little evidence that CRPS “progresses” from one stage to another.

* Currently, the Budapest consensus criteria for clinical diagnosis of CRPS delete staging and
require at least three symptoms and two signs in the following four categories: (1) sensory, (2)
vasomotor, (3) sudomotor/edema, and (4) motor/trophic. Pain (usually burning or electrical in
quality) is the primary clinical feature of CRPS. Limb pain syndromes that do not meet these
criteria are best classified as “limb pain—not otherwise specified.” In CRPS, localized sweating
(increased resting sweat output) and changes in blood flow may produe temperature
differences between affected and unaffected limbs.

The natural history of typical CRPS may be more benign and more variable than previously
recognized. A variety of surgical and medical treatments have been developed, with conflicting
reports of efficacy. Clinical trials suggest that early mobilization with physical therapy or a brief
course of glucocorticoids may be helpful for early CRPS type I or II. Chronic glucocorticoid
treatment is not recommended*****.

Initial Treatment of Orthostatic Hypotension (OH)

Patient education: mechanisms and stressors of OH

High-salt diet (10–20 g/d)

High-fluid intake (2 L/d)

Elevate head of bed 10 cm (4 in.) to minimize supine hypertension

Maintain postural stimuli

Learn physical counter-maneuvers

Compression garments

Correct anemia

POSITIVE MED PG
Web: http://www.positivemedpg.com/ Score:
Email: manohrajan@yahoo.co.in 0.00 / 100
Page 158
NIMHANS - DM Neurology -
Exam Title :
MO...
Email : shubham.saha452@gmail.com
Contact : 9932160514

QUESTION 85. NDE3MCtTSFVCSEFNICBTQUhBK3NodWJoYW0uc2FoYTQ1MkBnbWFpbC5jb20rOTkzMjE2M


USU9OIDg0
A person sustains spinal cord injury following a road traffic accident. X-ray films indicate a
fracture of T8 vertebra. Which of the following spinal segments is likely to be involved, if you
suspect a lesion of the spinal cord at this level?

a) T7
b) T9
c) T10
d) T11
Correct Answer: D
Your Answer: Unanswered
Explanation

Spinal Cord Levels Relative to the Vertebral Bodies

Spinal Cord Level Corresponding Vertebral Body

Upper cervical Same as cord level

Lower cervical 1 level higher

Upper thoracic 2 levels higher

Lower thoracic 2 to 3 levels higher

Lumbar T10-T12

Sacral T12-L1

QUESTION 86. NDE3MCtTSFVCSEFNICBTQUhBK3NodWJoYW0uc2FoYTQ1MkBnbWFpbC5jb20rOTkzMjE2M


USU9OIDg1
According to Revised GCS (2014)-Glascow coma scale, a motor scor of 4 indicates which of the
following

a) Normal flexion
b) Withdraws
c) Localizes
d) obeys
Correct Answer: A

POSITIVE MED PG
Web: http://www.positivemedpg.com/ Score:
Email: manohrajan@yahoo.co.in 0.00 / 100
Page 159
NIMHANS - DM Neurology -
Exam Title :
MO...
Email : shubham.saha452@gmail.com
Contact : 9932160514
Your Answer: Unanswered
Explanation

TBI is commonly defined as an alteration in brain function, or other evidence of brain pathology,
caused by an external force, and characterized by the following: (1) any period of loss or
decreased level of consciousness (LOC), (2) any loss of memory for events immediately before
(retrograde) or after (posttraumatic) the injury, (3) any neurological deficits, and/or (4) any
alteration in mental state at the time of injury .-20 th

Glasgow Coma Scale for Head Injury

Eye Opening (E) Verbal Response (V)

Spontaneous 4 Oriented 5

To loud voice 3 Confused, disoriented 4

To Pain 2 Inappropriate words 3

Nil 1 Incomprehensible sounds 2

Nil 1

Best Motor Response (M)

Obeys 6

Localizes 5

Normal flexion-20 th [19 th edition- 4


Withdraws (flexion)]]

Abnormal flexion-20 th [19 th - 3


Abnormal flexion Posturing]

Extension posturing 2

Nil 1

POSITIVE MED PG
Web: http://www.positivemedpg.com/ Score:
Email: manohrajan@yahoo.co.in 0.00 / 100
Page 160
NIMHANS - DM Neurology -
Exam Title :
MO...
Email : shubham.saha452@gmail.com
Contact : 9932160514

Note : Coma score = E + M + V. Patients scoring 3 or 4 have an 85% chance of dying or


remaining vegetative, whereas scores >11 indicate only a 5–10% likelihood of death

Note: Revised GCS (2014).

QUESTION 87. NDE3MCtTSFVCSEFNICBTQUhBK3NodWJoYW0uc2FoYTQ1MkBnbWFpbC5jb20rOTkzMjE2M


USU9OIDg2
A patient met with a road traffic accident and was diagnosed to have severe brain injury.He did
not have convulsions.Which of the following statement regarding prophylactic antiepileptic
medications is true

a) have no role
b) are given for 48 hrs
c) are given for 1 week
d) are given for 2 weeks
Correct Answer: C
Your Answer: Unanswered
Explanation

SEVER BRAIN INJURY

19 th edition

The use of prophylactic antiepileptic medications has been recommended, but there is
little supportive data

20 TH EDITION

Prophylactic antiepileptic medications are recommended for 7 days and should be discontinued
unless there are multiple seizures postinjury

MORE NIMHANS FACTS-NIMHANS 2014

Bilateral lesions in the fusiform and lingual gyri***** of the occipitotemporal cortex
leads to inability to recognize familiar faces called as prosopagnosia and visual object
agnosi a . The common cause is bilateral infarctions in the territory of the posterior
cerebral arteries

Prosopagnosia- Bilateral lesion in fusiform and lingual gyri of occipito temporal cortex****** is
the cause and it presents with face and object recognition defects

QUESTION 88. NDE3MCtTSFVCSEFNICBTQUhBK3NodWJoYW0uc2FoYTQ1MkBnbWFpbC5jb20rOTkzMjE2M


USU9OIDg3
Location of the indicated arrow in internal capsule is

POSITIVE MED PG
Web: http://www.positivemedpg.com/ Score:
Email: manohrajan@yahoo.co.in 0.00 / 100
Page 161
NIMHANS - DM Neurology -
Exam Title :
MO...
Email : shubham.saha452@gmail.com
Contact : 9932160514

a) Rubrospinal tract
b) Lateral corticospinal tract
c) Reticulospinal tract
d) Vestibulospinal tract
Correct Answer: A
Your Answer: Unanswered
Explanation

MORE NIMHANS FACTS-NIMHANS 2014

PARINAUDS SYNDROME

Parinaud's Syndrome is also called as dorsal midbrain syndrome . It is a


supranuclear****** vertical gaze disorder from damage to the posterior commissure. It
is a classic sign of hydrocephalus from aqueductal stenosis*****.

Features include loss of upgaze (and sometimes downgaze), convergence-retraction


nystagmus on attempted upgaze, downwards ocular deviation ("setting sun" sign), lid
retraction (Collier's sign*****), skew deviation, pseudoabducens palsy, and light-near
dissociation of the pupils*****.

Pineal Region Tumors

* These typically present with headache, visual symptoms, and hydrocephalus.*****


Patients may have Parinaud's syndrome characterized by impaired upgaze and
accommodation

POSITIVE MED PG
Web: http://www.positivemedpg.com/ Score:
Email: manohrajan@yahoo.co.in 0.00 / 100
Page 162
NIMHANS - DM Neurology -
Exam Title :
MO...
Email : shubham.saha452@gmail.com
Contact : 9932160514
QUESTION 89. NDE3MCtTSFVCSEFNICBTQUhBK3NodWJoYW0uc2FoYTQ1MkBnbWFpbC5jb20rOTkzMjE2M
USU9OIDg4
Recent work has identified two fundamentally different types of reactive astrocytes that appear
to have

countervailing functions; the terms A1 and A2 astrocytes.A1s might be harmful. In contrast, A2s
up-regulated many neurotrophic factors are protective. Which of the secreted products of
activated microglia, specifically WIIL NOT induce astrocytes to transform to the A1 type

a) IL-1a
b) TGF Beta
c) TNF
d) C1q
Correct Answer: B
Your Answer: Unanswered
Explanation

Explanation

POSITIVE MED PG
Web: http://www.positivemedpg.com/ Score:
Email: manohrajan@yahoo.co.in 0.00 / 100
Page 163
NIMHANS - DM Neurology -
Exam Title :
MO...
Email : shubham.saha452@gmail.com
Contact : 9932160514

One characteristic of the response to many types of brain injury is reactive astrocytosis, or the
formation of a glial scar

Recent work has identified two fundamentally different types of reactive astrocytes that appear
to have

countervailing functions; the terms A1 and A2 astrocytes have been proposed, by analogy to
brain

macrophage/microglia M1 and M2 designations

*A1s might be harmful. In contrast, A2s up-regulated many neurotrophic factors are
protective*****

*. A2 astrocytes are induced by ischemia and may serve beneficial functions, including a
contribution to tissue repair after injury.

POSITIVE MED PG
Web: http://www.positivemedpg.com/ Score:
Email: manohrajan@yahoo.co.in 0.00 / 100
Page 164
NIMHANS - DM Neurology -
Exam Title :
MO...
Email : shubham.saha452@gmail.com
Contact : 9932160514

*A1 astrocytes are induced in inflammatory and degenerative states, and actively participate in
the injury process.

*Secreted products of activated microglia, specifically IL-1a, TNF, and C1q, induce astrocytes to

transform to the A1 type. *****

*Functionally, A1 astrocytes lose the capacity to phagocytose synapses and myelin debris, and
are strikingly toxic in vitro to various populations of neurons and to mature oligodendrocytes,

Interestingly, OPCs, abundant in active lesions of

multiple sclerosis despite the inflammatory milieu, are resistant to A1 mediated death.

*There is speculation that products of A1 astrocytes could promote damage in

conditions as varied as MS, AD , Parkinson’s disease (PD) , and amyotrophic lateral sclerosis

(ALS) , despite their distinct etiologies and pathologies.

Subject

QUESTION 90. NDE3MCtTSFVCSEFNICBTQUhBK3NodWJoYW0uc2FoYTQ1MkBnbWFpbC5jb20rOTkzMjE2M


USU9OIDg5
A patient presented with Head injury and a CTscan revealed a subarachnoid
hemorrhage and was operated upon..He developed increased urine output in the 2 nd
POD.The paramaters were- Bl.urea -20mg;Sr.creatinine-1mg;Sr.sodium-125
mq;sr.K-5mq.What could be the diagnosis

a) Central Diabetes Insipidus


b) Nephrogenic Diabetes Insipidus
c) cerebral salt wasting
d) SIADH
Correct Answer: C
Your Answer: Unanswered
Explanation

*The syndrome of “cerebral salt wasting” is a rare cause of hypovolemic hyponatremia,


encompassing hyponatremia with clinical hypovolemia

and inappropriate natriuresis in association with intracranial disease; associated disorders


include subarachnoid hemorrhage******, traumatic brain injury,

craniotomy, encephalitis, and meningitis

MORE NIMHANS FACTS-NIMHANS 2014

SLEEP FACTS

Polysomnographic profiles define two states of sleep: (1) rapid-eye-movement (REM) sleep and
(2) non-rapid-eye-movement (NREM) sleep. NREM sleep is subdivided into 3 stages ,
characterized by increasing arousal threshold and slowing of the cortical EEG.

POSITIVE MED PG
Web: http://www.positivemedpg.com/ Score:
Email: manohrajan@yahoo.co.in 0.00 / 100
Page 165
NIMHANS - DM Neurology -
Exam Title :
MO...
Email : shubham.saha452@gmail.com
Contact : 9932160514

Overall, REM sleep constitutes 20–25% of total sleep****, and NREM stages 1 and 2 are 50–
60%. In infancy, REM sleep may comprise 50% of total sleep time*****, and the percentage is
inversely proportional to developmental age.

The first REM sleep episode occurs in the second hour of sleep*****. More rapid onset of REM
sleep (if <30 min) occurs in endogenous depression, narcolepsy, circadian rhythm disorders, or
drug withdrawal

Characteristic features of sleep in older people include reduction of slow-wave sleep*****,


frequent spontaneous awakenings, early sleep onset, and early morning awakening

* REM sleep is characterized by a low-amplitude, mixed-frequency EEG similar to that


of NREM stage 1 sleep. The EOG shows bursts of REM similar to those seen during
eyes-open wakefulness. Chin EMG activity is absent [AIIMS NOV-2006***] , reflecting
the brainstem-mediated muscle atonia that is characteristic of that state

QUESTION 91. NDE3MCtTSFVCSEFNICBTQUhBK3NodWJoYW0uc2FoYTQ1MkBnbWFpbC5jb20rOTkzMjE2M


USU9OIDkw
GLymphatics are associated with which of the follwoing organ

a) Brain
b) Gastrointestinal system
c) Genitourinary system
d) Tongue
Correct Answer: A
Your Answer: Unanswered
Explanation

POSITIVE MED PG
Web: http://www.positivemedpg.com/ Score:
Email: manohrajan@yahoo.co.in 0.00 / 100
Page 166
NIMHANS - DM Neurology -
Exam Title :
MO...
Email : shubham.saha452@gmail.com
Contact : 9932160514

Two newly identified lymphatic structures of the CNS are the glymphatic and deep dural
lymphoid systems,

responsible for clearance of debris in the CNS.

*The brain has traditionally been considered to lack a classical lymphatic system, and immune
responses against antigens are

less effectively generated in the CNS than in other organ systems, a concept termed “immune
privilege.” However,

immune privilege status of the brain is only relative and not absolute.

Glymphatic pathways are also likely to represent an important egress pathway for lymphocytes
in the CNS and a route for

lymphocyte encounter with CNS antigens in cervical lymph nodes. In this regard, recent data
indicate that deep

cervical lymph nodes might be a site for antigen-specific stimulation of B-cells in MS .

POSITIVE MED PG
Web: http://www.positivemedpg.com/ Score:
Email: manohrajan@yahoo.co.in 0.00 / 100
Page 167
NIMHANS - DM Neurology -
Exam Title :
MO...
Email : shubham.saha452@gmail.com
Contact : 9932160514

*The glymphatic system derives its name from a distinctive architecture involving lymphoid-like
structures and astroglial cells [recognition of its dependence upon glial cells and the similarity
of its functions to those of the peripheral lymphatic system ]

*glymphatic system that likely functions to remove waste products from the brain interstitium

Glymphatic pathways are also likely to represent an important egress pathway for lymphocytes
in the CNS and a route for

lymphocyte encounter with CNS antigens in cervical lymph nodes

*Another fascinating aspect of the glymphatic system is that the transport of fluids and solutes
accelerates with sleep

*Interstitial flow in the CNS is also impaired with aging, possibly related to changes in
astrocytic aquaporin-4

expression

A second recently identified pathway consists of a plexus of small lymphatic-like vessels located
on the external

surface of meningeal arteries and deep dural sinuses (including the sagittal and transverse
sinuses), structures

that exit the brain along the surface of veins and arteries and drain to the deep cervical lymph
nodes

QUESTION 92. NDE3MCtTSFVCSEFNICBTQUhBK3NodWJoYW0uc2FoYTQ1MkBnbWFpbC5jb20rOTkzMjE2M


USU9OIDkx
Histopathologic microscopic picture of muscle belongs to which of the following

POSITIVE MED PG
Web: http://www.positivemedpg.com/ Score:
Email: manohrajan@yahoo.co.in 0.00 / 100
Page 168
NIMHANS - DM Neurology -
Exam Title :
MO...
Email : shubham.saha452@gmail.com
Contact : 9932160514

a) Polymyositis
b) Myasthenia gravis
c) Scleroderma
d) Dermatomyositis
Correct Answer: D
Your Answer: Unanswered
Explanation

Explanation

POSITIVE MED PG
Web: http://www.positivemedpg.com/ Score:
Email: manohrajan@yahoo.co.in 0.00 / 100
Page 169
NIMHANS - DM Neurology -
Exam Title :
MO...
Email : shubham.saha452@gmail.com
Contact : 9932160514

Dermatomyositis

* The cutaneous signs of dermatomyositis may precede or follow the development of myositis
by weeks to years.

* Cases lacking muscle involvement is called as dermatomyositis sine myositis .

* The clinical signs are ,"Heliotrope" erythema, violaceous, flat-topped papules over the dorsal
interphalangeal joints called Gottron's sign or Gottron's papules ,Periungual telangiectasia.

* Sclerodactyly and Raynaud's phenomenon can occur.

* Areas of hypopigmentation, hyperpig-mentation, mild atrophy, and telangiectasia known as


poikiloderma can occur .

* Skin biopsy reveals epidermal atrophy, hydropic degeneration of basal keratinocytes,


edema of the upper dermis, and a mild mononuclear cell infiltrate.

* Direct immunofluorescence microscopy of lesional skin is usually negative.

POSITIVE MED PG
Web: http://www.positivemedpg.com/ Score:
Email: manohrajan@yahoo.co.in 0.00 / 100
Page 170
NIMHANS - DM Neurology -
Exam Title :
MO...
Email : shubham.saha452@gmail.com
Contact : 9932160514

* Topical glucocorticoids are used and patients should avoid exposure to ultraviolet
irradiation.

Dermatomyositis presents with Periorbital violaceous erythema is called heliotrope rash. It o


ften involves the hands as erythematous flat-topped papules over the knuckles ( Gottron's sign
) and has periungual telangiectasias.

QUESTION 93. NDE3MCtTSFVCSEFNICBTQUhBK3NodWJoYW0uc2FoYTQ1MkBnbWFpbC5jb20rOTkzMjE2M


USU9OIDky
There are some mutations that may be considered ecogenetic, as they typically remain
silent, meaning they do not cause disease, unless an external event occurs.Which of
the followingmutation in mutation in the mitochondrial 12S rRNA gene is associated
with hearing loss but is rapidly exacerbated by exposure to normal dosages of an
aminoglycoside.

a) m.A1555G
b) m.B1555G
c) m.C1555G
d) m.D1555G
Correct Answer: A
Your Answer: Unanswered
Explanation

MITOCHONDRIAL DNA STRUCTURE AND FUNCTION

*mtDNA itself replicates independently of cellular replication.

* Another important feature of the mtDNA replication process is a reduced stringency of


proofreading and replication error correction

* There are some mutations that may be considered ecogenetic, as they typically remain silent,
meaning they do not cause disease, unless an external event occurs.

* One classic example is seen in a common (1:800) mutation in the mitochondrial 12S rRNA
gene, m.A1555G., which is associated with hearing loss but is rapidly exacerbated by exposure
to normal dosages of an aminoglycoside.

Point mutations in the mtDNA gene encoding the 12S rRNA (m.A1555G) result in heritable
nonsyndromic hearing loss. One such mutation causes heritable ototoxic susceptibility to
aminoglycoside antibiotics, which opens a pathway for a simple pharmacogenetic test in the
appropriate clinical settings. This is an example of an eco-genetic disorder in that most people
with this mutation do not develop any symptoms until exposed to an external agent

Another example is strict avoidance of aminoglycosides in the familial syndrome of ototoxic


susceptibility to aminoglycosides in the presence of the mtDNA m.1555A>G mutation of the
12SrRNA encoding gene

MORE FACTS

The mtDNA sequence contains a total of 37 genes, of which 13 encode mitochondrial protein

POSITIVE MED PG
Web: http://www.positivemedpg.com/ Score:
Email: manohrajan@yahoo.co.in 0.00 / 100
Page 171
NIMHANS - DM Neurology -
Exam Title :
MO...
Email : shubham.saha452@gmail.com
Contact : 9932160514

components of the ETC . The remaining 22 tRNA- and 2 rRNA-encoding genes are
mitochondria-specific and dedicated to the process of

translating the 13 mtDNA-encoded proteins. The mtDNA itself replicates constantly,


independent of cell division, and requires its own unique

polymerase, referred to as polymerase gamma (polγ), which is encoded by the nuclear


protein POLG

QUESTION 94. NDE3MCtTSFVCSEFNICBTQUhBK3NodWJoYW0uc2FoYTQ1MkBnbWFpbC5jb20rOTkzMjE2M


USU9OIDkz
An elevated CSF lactate is a more specific test for mitochondrial disease if there is
central nervous system involvement. Which of the following is a recent marker for the
diagnosis of mitochondrial myopathy

a) Growth Differentiating Factor 15 (GDF15)


b) Glial fribrillary protein
c) Brain derived natriuretic peptide
d) ANP
Correct Answer: A
Your Answer: Unanswered
Explanation

An elevated CSF lactate is a more specific test for mitochondrial disease if there is central
nervous system involvement. The serum creatine kinase may be elevated but is often normal,
even in the presence of a proximal myopathy. Recently, testing for elevated levels of Growth
Differentiating Factor 15 (GDF15) has shown a high degree of sensitivity and specificity in those
with a mitochondrial myopathy, but it is not known yet if the degree of elevation for an
individual patient reflects the severity of the illness or is in any way a marker of disease activity.
Urinary organic and amino acids may also be abnormal, reflecting metabolic and kidney
proximal tubule dysfunction 95% of patients with LHON harbor one of the three mtDNA point
mutations (m.11778A>G, m.A3460A>G, or m.14484T>C).

It is a protein belonging to the transforming growth factor beta superfamily . Under normal
conditions, GDF-15 is expressed in low concentrations in most organs and upregulated because
of injury of organs such as such as liver , kidney , heart and lung

MORE NIMHANS FACTS-2014

Mitral cells are the principal neurons in the olfactory bulb****. There are about 50,
000 in the adult human. They have a primary apical dendrite which extends into a
spherical bundle of neuropil called a glomerulus which receives the input from the
olfactory receptor neurons

Glomeruli

* It is seen in in olfactory bulb . It is a spherical mass of neuropil and recieves input


from unmyelinated axons of receptor cells from olfactory N. The second order neurons
are mitral cells* Axons of mitral cells project into anterior olfactory nucleus and

POSITIVE MED PG
Web: http://www.positivemedpg.com/ Score:
Email: manohrajan@yahoo.co.in 0.00 / 100
Page 172
NIMHANS - DM Neurology -
Exam Title :
MO...
Email : shubham.saha452@gmail.com
Contact : 9932160514
amygdala, Prepyriform cortex. Second site for olfactory chemoreception is in
epithelium of vomeronasal organ, which d etect phermones.

QUESTION 95. NDE3MCtTSFVCSEFNICBTQUhBK3NodWJoYW0uc2FoYTQ1MkBnbWFpbC5jb20rOTkzMjE2M


USU9OIDk0
Which among the following antiplatelet drugs is a Reversible inhibitor of P2Y12

a) Clopidrogel
b) Prasugrel
c) Ticagrelor
d) Cangrelor
Correct Answer: C
Your Answer: Unanswered
Explanation

The drugs clopidogrel , prasugrel , ticagrelor , and cangrelor bind to this receptor and are
called as antiplatelet agents

ADP RECEPTOR ANTAGONISTS

There are several distinct ADP receptors, classified as P2X1, P2Y1, andP2Y12.*****

The ADP receptor antagonists include the thienopyridines (clopidogrel and prasugrel) and
ticagrelor. All of these drugs target P2Y12, the key ADP receptor on platelets.

Thienopyridines • Mechanism of action The thienopyridines are structurallyrelated drugs


that selectively inhibit ADP-induced plateletaggregation by irreversibly blocking P2Y12*******

Ticagrelor As an orally active inhibitor of P2Y12, ticagrelor differs from the thienopyridines in
that ticagrelor does not require metabolic activation********[NOT A PRODRUG] and it produces
reversible inhibition of the ADP receptor

New agents in advanced stages of development include cangrelor, aparenteral, rapidly acting,
reversible inhibitor of P2Y12. Cangrelor An adenosine analogue, cangrelor binds reversibly to
P2Y12and inhibits its activity. The drug has a half-life of 3–6 min and is givenIV as a bolus
followed by an infusion. When stopped, platelet functionrecovers within 60 min

MORE NIMHANS FACT

NEW ANTIPLATELETS-Vorapaxar An orally active PAR-1 antagonist, vorapaxar is slowly


eliminated with a half-life of about 200 h. ******

QUESTION 96. NDE3MCtTSFVCSEFNICBTQUhBK3NodWJoYW0uc2FoYTQ1MkBnbWFpbC5jb20rOTkzMjE2M


USU9OIDk1
Which of the following drug should be stopped 14 days before lumbar puncture

a) Aspirin
b) Heparin
c) Clopidrogel

POSITIVE MED PG
Web: http://www.positivemedpg.com/ Score:
Email: manohrajan@yahoo.co.in 0.00 / 100
Page 173
NIMHANS - DM Neurology -
Exam Title :
MO...
Email : shubham.saha452@gmail.com
Contact : 9932160514

d) Ticlopidine
Correct Answer: D
Your Answer: Unanswered
Explanation

GUIDELINES FOR PATIENTS RECEIVING ANTICOAGULANT OR ANTIPLATELT


MEDICATIONS

* Platelet counts; a count of <20,000/Μl is considered to be a contraindication to LP.

Some institutions recommend that the platelet count be >40,000 prior to LP.

* The most common site of bleeding is the epidural space.

Guidelines for some commonly used anticoagulants are summarized below.

Time of stoppage before a


Drug
lumbar puncture

Unfractionated Heparin 2–4 h prior

At least 24 h before the


Low-Molecular-Weight Heparin
procedure.

Spinal puncture is
Warfarin
contraindicated

Aspirin and Nonsteroidal Anti-inflammatory Drugs 10 days for aspirin and for 48 h
(NSAIDs) for NSAIDs.

Ticlopidine 14 days

clopidogrel 7 days prior

Glycoprotein Iib/IIIa Inhibitors, Direct Thrombin


LP avoided
Inhibitors, Oral Factor Xa Inhibitor

* Unfractionated Heparin, Therapeutic Dosing The ASRA 2010 Practice Advisory


recommends discontinuing unfractionated heparin (UFH) 2–4 h prior to removal of spinal or
epidural catheters to minimize risk of hematoma. The half-life of heparin is 60–90 min.

*Low-Molecular-Weight Heparin, should be held for at least 24 h before the procedure.

*Warfarin- Spinal puncture is contraindicated during warfarin therapy.

POSITIVE MED PG
Web: http://www.positivemedpg.com/ Score:
Email: manohrajan@yahoo.co.in 0.00 / 100
Page 174
NIMHANS - DM Neurology -
Exam Title :
MO...
Email : shubham.saha452@gmail.com
Contact : 9932160514

*Aspirin and Nonsteroidal Anti-inflammatory Drugs (NSAIDs) . Reversal of drug effect on


platelet function requires stopping the drug for approximately 10 days for aspirin and for 48 h
for NSAIDs.

Ticlopidine/Clopidogrel ASRA guidelines suggest discontinuing ticlopidine 14 days prior to a


spinal or epidural procedure and discontinuing clopidogrel 7 days prior to the procedure.

Abciximab, Eptifibatide, and Other Platelet Glycoprotein Iib/IIIa Inhibitors

ASRA guidelines recommend avoiding spinal or epidural procedures until platelet function is
normal. Similar guidelines are reasonable for performing LP.

Direct Thrombin Inhibitors (e.g., Argatroban, Bivalirudin) ASRA guidelines recommend


against performing spinal or epidural anesthesia in patients receiving thrombin inhibitors.

Oral Factor Xa Inhibitor (e.g., Rivaroxaban)

LP should be avoided in patients receiving this drug

QUESTION 97. NDE3MCtTSFVCSEFNICBTQUhBK3NodWJoYW0uc2FoYTQ1MkBnbWFpbC5jb20rOTkzMjE2M


USU9OIDk2
Which among the following combination regarding the channelopathies is not correct

a) Paramyotonia congenital-Sodium Channel


b) Hyperkalemic periodic paralysis- Sodium channel
c) Benign neonatal familial convulsions –K channel
d) Familial hemiplegic migraine 2- Calcium channel
Correct Answer: D
Your Answer: Unanswered
Explanation

Explanation

Examples of Neurologic Channelopathies

Channel
Category Disorder
Type

Episodicataxia-1 K

Ataxias Episodicataxia-2 Ca

Spinocerebellar ataxia-6 Ca

Familial hemiplegic migraine 1 Ca


Migraine
Familial hemiplegic migraine 2 Na

POSITIVE MED PG
Web: http://www.positivemedpg.com/ Score:
Email: manohrajan@yahoo.co.in 0.00 / 100
Page 175
NIMHANS - DM Neurology -
Exam Title :
MO...
Email : shubham.saha452@gmail.com
Contact : 9932160514

Benign neonatal familial convulsions K


Epilepsy
Generalized epilepsy with febrile convulsions plus Na

Hyperkalemic periodic paralysis Na


Periodic
paralysis Hypokalemic periodic paralysis Ca

Myotonia ncludec Cl
Myotonia
Paramyotonia congenita Na

Jervell and Lange-Nielsen syndrome (deafness, prolonged QT


Deafness K
interval, and arrhythmia)

Autosomal dominant progressive deafness K

Autoimmune

Kv1
Limbic encephalitis
Kv1
Acquired neuromyotonia
Paraneoplastic Ca (P/Q
Cerebellar ataxia type)

Lambert-Eaton syndrome Ca (P/Q


type)

QUESTION 98. NDE3MCtTSFVCSEFNICBTQUhBK3NodWJoYW0uc2FoYTQ1MkBnbWFpbC5jb20rOTkzMjE2M


USU9OIDk3
Which of the following antiepileptic drug acts through opening of K+ Channels

a) lamotrigine
b) topiramate
c) zonisamide
d) ezogabine
Correct Answer: D
Your Answer: Unanswered
Explanation

Antiepileptic drugs appear to act primarily by blocking the initiation or spread The mechanisms
include inhibition of Na+-dependent action potentials in a frequency-dependent manner (e.g.,
phenytoin, carbamazepine, lamotrigine, topiramate, zonisamide, lacosamide, rufinamide),
inhibition of voltage-gated Ca2+ channels (phenytoin, gabapentin, pregabalin), facilitating the

POSITIVE MED PG
Web: http://www.positivemedpg.com/ Score:
Email: manohrajan@yahoo.co.in 0.00 / 100
Page 176
NIMHANS - DM Neurology -
Exam Title :
MO...
Email : shubham.saha452@gmail.com
Contact : 9932160514
opening of potassium channels (ezogabine), attenuation of glutamate activity (lamotrigine,
topiramate, felbamate), potentiation of GABA receptor function (benzodiazepines and
barbiturates), increase in the availability of GABA (valproic acid, gabapentin, tiagabine), and
modulation of release of synaptic vesicles (levetiracetam).

Mechanisms of Action of Antiepileptic Drugs

* The mechanism of action of action of various antiepileptic drugs are listed below

* Inhibition of Na + -dependent * Phenytoin, carbamazepine, lamotrigine, topiramate,


action potentials zonisamide, lacosamide, rufinamide

* Inhibition of voltage-gated Ca 2
* Phenytoin, gabapentin, pregabalin
+ channels

* Attenuation of glutamate
* Lamotrigine, topiramate, felbamate
activity

* Potentiation of GABA receptor


* Benzodiazepines and barbiturates
function

* Increase in the availability of * Valproic acid, gabapentin, tiagabine [AIIMS


GABA NOV-2005***]

* Modulation of release of
* Levetiracetam.
synaptic vesicles

Drug adverse effect

* Diplopia

* Phenytoin –Level increased by isoniazid, sulfonamides, * Ataxia


fluoxetine. Half life is 24 h*
* Hirsuitism

* Gumhypertrophy

POSITIVE MED PG
Web: http://www.positivemedpg.com/ Score:
Email: manohrajan@yahoo.co.in 0.00 / 100
Page 177
NIMHANS - DM Neurology -
Exam Title :
MO...
Email : shubham.saha452@gmail.com
Contact : 9932160514

* Ataxia

* Diplopia
* Carbama zepine- Level increased by erythromycin,
propoxyphene, isoniazid, cimetidine, fluoxetine. Half life is 10– * Aplastic Anemia
17 h
* Hepatotoxicity

* Hyponatremia

* Tremor, Ataxia

* Wt. Gain

* Valproic acid -. Half life is 15 hors * Hepatotoxicity

* Alopecia

* Hyperammonemia*

* Lamotrigine. –Half life is25 h

* 14 h (with enzyme-inducers) * Steven. Johnson syndrome

* 59 h (with valproic acid)

* Gabapentin -. Half life is 5-9 hours * Wt. Gain

* Wt. loss* [AIIMS


* Topiramate –also used in Lennox-Gastaut syndrome. Half life MAY-2006***], glaucoma
is 20–30 h
* Renal stones, hypohydrosis

* Wt. loss*
* Felbamate – Increases phenytoin, valproic acid, active
* Aplastic Anemia
carbamazepine. Half life is 16-20 hours
* Hepatic failure

* Zonisamide-. Half life is 50-68 hours * Renal stones, hypohydrosis

QUESTION 99. NDE3MCtTSFVCSEFNICBTQUhBK3NodWJoYW0uc2FoYTQ1MkBnbWFpbC5jb20rOTkzMjE2M


USU9OIDk4
A patient presented with sudden onset of quadriparesis.His MRI brain revealed
features suggestive of Central Pontine Myelinolysis.Which of the following is false
regarding CPM

POSITIVE MED PG
Web: http://www.positivemedpg.com/ Score:
Email: manohrajan@yahoo.co.in 0.00 / 100
Page 178
NIMHANS - DM Neurology -
Exam Title :
MO...
Email : shubham.saha452@gmail.com
Contact : 9932160514

a) quadriplegia
b) pseudobulbar palsy
c) locked in state
d) associated with rapid correction of hypocalcemia
Correct Answer: D
Your Answer: Unanswered
Explanation

Central Pontine Myelinolysis

* This disorder typically presents in a devastating fashion as quadriplegia and pseudobulbar


palsy-locked in state most cases are associated with rapid correction of hyponatremia or with
hyperosmolar states

MORE NIMHANS FACTS-NIMHANS 2014

ANTERIOR CHOROIDAL ARTERY OCCLUSION

Anterior Choroidal Artery (AIIMS NOV-2012***) arises from the internal carotid artery and
supplies the posterior limb of the internal. The complete syndrome of anterior choroidal artery
occlusion consists of contralateral hemiplegia, hemianesthesia (hypesthesia), and homonymous
hemianopia. Anterior choroidal strokes are usually due to situ thrombosis. It is prone for
iatrogenic occlusion during surgical clipping of aneurysms.

QUESTION 100. NDE3MCtTSFVCSEFNICBTQUhBK3NodWJoYW0uc2FoYTQ1MkBnbWFpbC5jb20rOTkzMjE2M


USU9OIDk5
Flibanserin, originally developed as an antidepressant, has been approved by the FDA as a
treatment for low sexual desire in

premenopausal women.What is the mode of action of this drug

a) a postsynaptic agonist of serotonin receptor 1A and antagonist of serotonin receptor 2A,


b) a postsynaptic antagonist of serotonin receptor 1A and antagonist of serotonin receptor 2A,
c) a presynaptic agonist of serotonin receptor 1A and antagonist of serotonin receptor 2A,
d) a presynapticantagonist of serotonin receptor 1A and antagonist of serotonin receptor 2A,
Correct Answer: A
Your Answer: Unanswered
Explanation

Flibanserin, originally developed as an antidepressant, has been approved by the FDA as a


treatment for low sexual desire in

premenopausal women. Flibanserin, a postsynaptic agonist of serotonin receptor 1A and


antagonist of serotonin receptor 2A,

increases sexual desire and reduces resultant stress in women with HypoSexual Desire
Disorder (HSDD) with few adverse effects.

Flibanserin may boost sex drive in women who experience low sexual desire and who find the
experience distressing. The drug

POSITIVE MED PG
Web: http://www.positivemedpg.com/ Score:
Email: manohrajan@yahoo.co.in 0.00 / 100
Page 179
NIMHANS - DM Neurology -
Exam Title :
MO...
Email : shubham.saha452@gmail.com
Contact : 9932160514

should be discontinued if there is no improvement in sex drive after 8 weeks. Potentially serious
side effects include low blood

pressure, dizziness, and fainting, particularly if it is mixed with alcohol. Other common adverse
events include dizziness, nausea,

fatigue, sleepiness, and insomnia. Health care professionals and pharmacies dealing with
flibanserin have to undergo a

certification process, and patients need to submit a written agreement to abstain from alcohol.

MORE FACTS-NIMHANS 2014

George Phillip Cammann -Famous physician: In 1852 he invented the modern binaural
stethoscope. He never patented it, as he believed the instrument's design should be
freely available

POSITIVE MED PG
Web: http://www.positivemedpg.com/ Score:
Email: manohrajan@yahoo.co.in 0.00 / 100
Page 180
NIMHANS - DM Neurology -
Exam Title :
MO...
Email : shubham.saha452@gmail.com
Contact : 9932160514
Review in

POSITIVE MED PG
Web: http://www.positivemedpg.com/ Score:
Email: manohrajan@yahoo.co.in 0.00 / 100
Page 181

Вам также может понравиться